Download Document

Document related concepts

Dental avulsion wikipedia , lookup

Disease wikipedia , lookup

Syndemic wikipedia , lookup

Hygiene hypothesis wikipedia , lookup

Dental emergency wikipedia , lookup

Organ-on-a-chip wikipedia , lookup

Tissue engineering wikipedia , lookup

Sjögren syndrome wikipedia , lookup

List of medical mnemonics wikipedia , lookup

Transcript
DENTISTRY FACULTY
Pathological anatomy include the pathology of head and neck
Module 2. Special Pathomorphology.
TEXT TESTS
1. Acute endocarditis warty rheumatism accompanied by:
A. fibrinoid changes of connective tissue and endothelium of the
postponement of the masses in the form of warts
B. mucoid changes of connective tissue with postponement of thrombotic
masses in the form of warts
C. fibrinoid changes of cardiomyocells and desquamation of the
postponement of the epicardium warts
D. mucoid changes of connective tissue and desquamation of the
postponement of the epicardium warts
E. * fibrinoid changes of connective tissue and desquamation of the
postponement of endothelial damage in parts of thrombotic masses in the
form of warts
2. Acute ischemic heart disease leads to ...
A. Arrhythmias
B. Acute aneurysm
C. Chronic aneurysm
D. Brown atrophy of myocardium
E. * Sudden coronary death
3. Adverse effect of myocardial infarction.
A. Cardiosclerosis
B. Obesity Heart
C. Heart disease
D. Atrophy heart
E. * Cardiorrhexis
4. Phase of rheumatic granuloma is characterized by the accumulation of:
A. Neutrophils, lymphocytes, histiocells
B. Lymphocytes and follicles formation
C. Lymphocytes and follicles formation in breeding centers
D. All that described above
E. * Macrophages around the fibrinoid necrosis
5. Rheumatic granuloma phase is characterized by granuloma formation and the
first period of this formation is called as :
A. "Flagging granuloma"
B. "Acute granuloma"
C. "Fading granuloma"
D. "Blooming granuloma"
E. * "Flowering granuloma"
6. Brain hematoma most often develops as a complication of …
A. Atherosclerosis
B. Posthaemorrhagic anemia
C. Malignant anemia
D. Myelomic disease
E. * Hypertension disease
7. Who is authors of alimentary and metabolic theory of atherosclerosis
pathogenesis?
A. Asshof
B. Goldstein, Braun
C. M'yasnykov
D. Dyuged
E. * Anichkov, Halatov
8. What is a background disease for atherosclerosis development most often?
A. Rheumatism
B. Diabetes
C. Symptomatic hypertension
D. Cerebrovascular disease
E. * Hypertension disease
9. What is the cardiovascular risk factors for atherosclerosis development?
A. Hyperlipidemia
B. Sedentary lifestyle
C. Excessive body weight
D. Arterial hypertension
E. * All listed above
10.Causes of death in acute ischemic heart disease.
A. Cardiogenic shock
B. Ventricular fibrillation
C. Hemoperykard
D. Acute heart failure
E. * All of the above
11.Name the changes in brain tissue which forming during hypertensive crisis.
A. Plasmocells impregnating of the arterioles wall
B. Cerebral ischemia
C. Softening of brain tissue
D. Brain cysts
E. * Diapedetic brain hemorrhage
12.Changes in cardiomyocytes in focal ischemic myocardium dystrophy.
A. Lysis of nuclei
B. Lysis of cytoplasm
C. Increasing the number of glycogen
D. Coagulation of cytoplasm
E. * Disappearance of glycogen
13.Changes in the aorta of developing hypertension
A. Gummy infiltration, sclerosis
B. Amyloidosis
C. Divided aneurysm
D. Caseous necrosis
E. * Atherosclerosis, hyperelastozis
14.What changes of coronary arteries lead to myocardial infarction most often?
A. Hiperelastoz
B. Elastofibroz
C. Hyalinosis
D. Coronaritis
E. * Atherosclerosis
15.Characteristic changes at liposklerosis stage of atherosclerosis.
A. Ulceration in the area of intima plaques
B. Vascularization in the area of intima plaques
C. Formation of necrotic cells in the intima
D. Ulceration of the wall surface thrombosis
E. * Fibrous tissue growth in intima plaques
16.Characteristic changes at lipoidosis stage of atherosclerosis.
A. The appearance of cells apoptoses
B. Sclerosis
C. Vascularization intimate
D. Petrification
E. * Focal infiltration of intima by cholesterol
17.Characteristic changes at prelipid stage of atherosclerosis.
A. Accumulation of erythrocytes in the intima
B. The appearance of foam cells
C. Accumulation of glycogen in intimate
D. Metabolic calcification
E. * Loss of glycocalix, expansion of intraendothelial cracks, activation of
endocytosis in endothelial cells
18.Name the risk factors for 2nd order of ischemic heart diseases.
A. Hypertension
B. Sedentary lifestyle
C. Excessive body weight
D. Protein starvation
E. * Violation of tolerance to carbohydrates
19.Clinical and morphological form of hypertension.
A. Suprarenal
B. Hepatic
C. Mesenteric
D. Aortic
E. * Brain
20.Clinical expression of arteriolosclerotic nephrocirrhosis at hypertension
disease.
A. Acute heart failure
B. Chronic heart failure
C. Acute renal failure
D. Amyloidosis
E. * Chronic renal failure
21.What type of vessels are affected by atherosclerosis mainly?
A. Veins
B. Venules
C. Arterioles
D. Capillaries
E. * Arteries
22.Definition of hypertension.
A.
B.
C.
D.
E.
Hypertension, as a secondary manifestation of the disease
Metabolic disease
Contagion
Disease impaired absorption
* Disease, the main feature of which is sustainable increase arterial
pressure
23.Depending on the prevailing alternatives or regenerative processes distinguish
the type of rheumatic valvular endocarditis:
A. intramural endocarditis
B. transmural endocarditis
C. epikardial endocarditis
D. mixed
E. * diffuse endocarditis
24. Rheumatic diseases - a group of chronic diseases characterized by systemic
lesion:
A. *connective tissue and blood vessels
B. joints
C. the circulatory system
D. cell
E. Kidney
25. What is rheumatic disease - (a group of chronic diseases characterized by
systemic lesion):
A. *connective tissue and blood vessels
B. joints
C. the circulatory system
D. cell
E. kidney
26. In the etiology of rheumatic diseases is essential clinically expressed or latent:
A. *streptococcal infection
B. Meningococcal infection
C. diplokoccal infection
D. fungal infections
E. all infections
27. Pathogenic mechanisms of rheumatic diseases:
A. *allergic reactions of immediate and delayed-type
B. streptococcal septicemia
C. shock
D. staphylococcus septicopyemia
E. gipertermical reactions
28. To belong to pathogenetic mechanisms of rheumatic diseases:
A. *allergic reactions of immediate and delayed-type
B. streptococcal septicemia
C. shock
D. staphylococcus septicopyemia
E. gipertermical reactions
29. In rheumatic diseases the main pathogenetic plot are:
A. *progressive disorganization of the connective tissue
B. stagnant around the plethora of small joints
C. ischemia
D. Hypertrophy of left heart
E. atrophy
30. In the pathogenesis of rheumatic diseases is:
A. atrophy
B. *progressive disorganization of the connective tissue
C. stagnant around the plethora of small joints
D. ischemia
E. Hypertrophy of left heart
31. In the pathogenesis of rheumatic diseases is:
A. atrophy
B. *progressive disorganization of the connective tissue
C. stagnant around the plethora of small joints
D. ischemia
E. hypertrophy of left heart
32. Disturbance of connective tissue in rheumatic diseases includes:
A. atrophy
B. hyaline droplets dystrophy
C. *mucoid swelling
D. amyloidosis
E. hydropic dystrophy
33. What includes the disturbance of connective tissue in rheumatic diseases:
A. atrophy
B. hyaline droplets dystrophy
C. *mucoid swelling
D. amyloidosis
E. hydropic dystrophy
34. Disturbance of connective tissue in rheumatic diseases includes:
A. *fibrinoid swelling and necrosis
B. hyaline droplets dystrophy
C. amyloidosis
D. hydropic dystrophy
E. atrophy
35. What type of dystrophy involves the disruption of connective tissue in
rheumatic diseases:
A. necrosis
B. *fibrinoid edema and necrosis
C. hyaline droplets dystrophy
D. amyloidosis
E. hydropic dystrophy
36. Disturbance of connective tissue in rheumatic diseases includes:
A. *Hyalinosis
B. cell reaction (granulematosis) and sclerosis
C. hyaline droplets dystrophy
D. amyloidosis
E. hydropic dystrophy
37. In rheumatism sensitizing factor antibodies:
A. *B-hemolytic streptococcus typeА
B. B-hemolytic streptococcus, type B
C. B-hemolytic streptococcus type AB
D. B-hemolytic staphylococcus type D
E. B-hemolytic staphylococcus type C
38. What antibodies are sensitizing factor in rheumatic conditions:
A. B-hemolytic streptococcus, type B
B. B-hemolytic streptococcus type AB
C. *B-hemolytic staphylococcus type A
D. B-hemolytic streptococcus type D
E. B-hemolytic staphylococcus type C
39. B-hemolytic streptococci type A has affinity with antigens аntibodies against
A. *connective tissue of heart
B. liver
C. kidney
D. tonsils
E. connective tissue of mediastinum
40. What kind of relationship with the antigens are B-hemolytic streptococci type
A:
A. kidney
B. tonsils
C. connective tissue of mediastinum
D. *connective tissue of heart
E. liver
41. Kinship antibodies against B-hemolytic streptococci type A antigens from
connective tissue of the heart explains the predominant lesion in patients with
rheumatism:
A. *heart
B. liver
C. only epikard
D. only endocard
E. more pericardiocentesis
42. What explains the predominant lesion in patients with rheumatism affinity
antibodies against B-hemolytic streptococci type A antigens from connective
tissue of the heart:
A. liver
B. *heart
C. only epikard
D. only endocard
E. more pericardiocentesis
43. In rheumatoid arthritis in patients with damage mainly to the connective
tissue:
A. *capsule of the joints
B. heart
C. kidney
D. renal capsule
E. liver
44. Connective tissue structures are damaged in rheumatoid arthritis:
A. heart
B. kidney
C. renal capsule
D. liver
E. *capsule of the joints
45. In the pathogenesis of rheumatoid arthritis are important:
A. *immune complexes, where the antibodies are different classes of
immunoglobulins (Ig M, Ig G, Ig A)
B. -C. immune complexes, where there are different classes of
immunoglobulins (Ig N, Ig Q, Ig B)
D. complexes, where there are different classes of immunoglobulins (Ig Y,
Ig H, Ig L)
E. immune complexes, where the antigen is a globulin of different classes
(Ig M, Ig G, Ig A)
46. What have an important role in the pathogenesis of rheumatoid arthritis:
A. immune complexes, where the antigen is a globulin of different classes
(Ig M, Ig G, Ig A)
B. *immune complexes, where the antibodies are different classes of
immunoglobulin (Ig M, Ig G, Ig A)
C. immune complexes, where the antigen is a globule of different classes
(Ig M, Ig G, Ig A) -D. immune complexes, where there are different classes of
immunoglobulins (Ig N, Ig Q, Ig B)
E. complexes, where there are different classes of immunoglobulins (Ig Y,
Ig H, Ig L)
47. In systemic lupus erythematosus is violated:
A. exchange of DNA and antigens to components formed the nucleus and
cytoplasm
B. *exchange of DNA and formation of antibodies to components of the
nucleus and cytoplasm
C. exchange of RNA and formation of antibodies to components of the
nucleus and cytoplasm
D. exchange of RNA and formation of antibodies
E. exchange of DNA and antigens to components formed the nucleus and
cytoplasm
48. Violations in systemic lupus erythematosus:
A. exchange of DNA and antigens to components formed the nucleus and
cytoplasm
B. *exchange of DNA and formation of antibodies to components of the
nucleus and cytoplasm
C. exchange of RNA and formation of antibodies to components of the
nucleus and cytoplasm
D. exchange of RNA and formation of antibodies
E. exchange of DNA and antigens to components formed the nucleus and
cytoplasm
49. Systemic scleroderma is characterized in the skin:
A. violation of vascularization
B. *sclerotic and atrophic changes
C. inflammation
D. calcification
E. violation of vascularization
50. What changes in the skin are identified with systemic scleroderma:
A. violation of vascularization
B. *sclerotic and atrophic changes
C. inflammation
D. calcification
E. violation of vascularization
51. The decisive factor for the development of scleroderma is a violation of the
synthesis:
A. keloid
B. *collagen
C. fibrin
D. fibrinogen
E. keloid
52. What is the decisive factor for the development of scleroderma - a violation of
the synthesis:
A. keloid
B. *collagen
C. fibrin
D. fibrinogen
E. keloid
53. For nodular periarteritis characterized mechanism immunokomplex vascular
lesions such as:
A. arterioles- capillaries anastomosis
B. * small artery
C. aorta
D. large artery
E. arterioles-capillary anastomosis
54. What affects the blood vessels in the nodular periarteritis:
A. large-caliber artery
B. arterioles-capillaries anastomosis
C. *artery and small-medium caliber
D. aorta
E. large-caliber artery
55. It is believed that with nodular periarteritis due to fibrinous necrosis secondary
hull vessels develop proliferative response of cells in the outer shell, followed
by transfer to:
A. granuloma
B. *sclerosis and the formation of knots
C. hyalinosis
D. amyloidosis
E. inflammation
56. In nodular periarteritis due to fibrinous necrosis secondary hull vessels
develop proliferative response of cells in the outer shell, followed by transfer
to:
A. granuloma
B. *sclerosis and the formation of knots
C. hyalinosis
D. amyloidosis
E. inflammation
57. Bekhterev disease - a chronic rheumatic disease with the damage:
A. *glenoid spine-connected device
B. articular apparatus
C. large joints
D. small and large joints
E. articular apparatus
58. What kind of surprised when the disease Bekhterev -A. *glenoid spine-connected device
B. articular apparatus
C. large joints
D. small and large joints
E. articular apparatus
59. Bekhterev disease - a chronic rheumatic disease damaged articular-connected
device the spine, which leads to:
A. *bone ankylosis
B. bone amyloidosis
C. osteopetrosis
D. osteoporosis
E. articular apparatus
60. As a result, the disease develops Bekhterev:
A. Osteoporosis
B. *bone ankylosis
C. bone amyloidosis
D. osteopetrosis
E. Osteoporosis
61. Complications of the disease Bekhterev -A. *bone ankylosis
B. bone amyloidosis
C. osteonecrosis
D. osteoporosis
E. necrosis
62. Dermatomyositis - rheumatic diseases, leading to the clinical and
morphological expression of which is a system failure:
A. *skeletal muscles
B. skin
C. dermis
D. only the smooth muscle
E. kidney
63. What is leading to the clinical and morphological expression, with
dermatomyositis:
A. System failure only of smooth muscle
B. *System failure skeletal muscles
C. Systemic dermal
D. System failure dermis
E. System failure only of smooth muscle
64. What kind of surprised when dermatomyositis:
A. *skeletal musculature
B. skin
C. derm
D. only the smooth muscle
E. kidney
65. Indicate which affects the structure with dermatomyositis:
A. *skeletal musculature
B. skin
C. dermis
D. only the smooth muscle
E. kindey
66. Rheumatism - a chronic disease with predominant lesion:
A. *heart and vascular
B. kidney and liver
C. arteries of medium caliber
D. muscular arteries, elastic-type
E. liver
67. In rheumatism predominantly affects:
A. *heart and blood vessels
B. kidney and liver
C. medium-sized artery
D. artery muscle-elastic type of
E. liver
68. Determination of rheumatism - a chronic disease with predominant lesion:
A. *heart and vascular
B. kidney and liver
C. arteries of medium caliber
D. muscular arteries, elastic-type
E. kidney and liver
69. In rheumatism periods alternate with periods of exacerbation of extinction
(remission), so during rheumatism:
A. remitting
B. nonhomogeneous
C. homogeneous
D. *wave
E. repeatedly
70. In rheumatism streptococci penetrate the body through the tonsils, release
toxins and cause destruction in the invasion of cells and inflammation, which
usually manifests itself:
A. *angina
B. tonzilyar abscesses
C. retropharyngeal abscesses
D. Qatar nasopharyngeal
E. abscesses
71. Because rheumatic conditions streptococci penetrate the body through the
tonsils, release toxins and cause destruction in the invasion of cells and
inflammation, which usually manifests itself:
A. *angina
B. tonzilyar abscesses
C. retropharyngeal abscesses
D. Qatar nasopharyngeal
E. abscesses
72. In rheumatism streptococci penetrate the body through the tonsils, release
toxins, which are products of the destruction of cells is:
A. *activated antibodies
B. immune complexes
C. antibodies, which produce antigens
D. аntigens, that produce antibodies
E. leukocytes
73. What structures are formed by the interaction of streptococcus toxins and
products of destruction of cells in the amygdala rheumatic conditions:
A. activated antibodies
B. immune complexes
C. antibodies, which produce antigens
D. *antigens, that produce antibodies
E. activated antibodies
74. In the development of rheumatism occurs mukoid edema, which is:
A. *Surface and reverse disruption of connective tissue
B. irreversible disruption of connective tissue
C. Surface restructuring of the connective tissue
D. reverse proliferation of connective tissue
E. Surface restructuring of the connective tissue
75. Which involves mukoid swelling that develops rheumatic conditions:
A. *surface reverse disruption of connective tissue
B. irreversible disruption of connective tissue
C. surface reorganization of connective tissue
D. reverse the proliferation of connective tissue
E. irreversible disruption of connective tissue
76. Early diagnosis and early treatment of rheumatism:
A. *can lead to complete recovery
B. can never lead to a cure
C. can not lead to a reorganization of the connective tissue
D. can not lead to the restitution of connective tissue
E. can not lead to a reorganization of the connective tissue
77. As a result of early diagnosis and early treatment of rheumatism are:
A. *Full recovery
B. can never lead to recovery
C. can not lead to a reorganization of the connective tissue
D. can not lead to the restitution of connective tissue
E. can never lead to a cure
78. When the development of rheumatism among the stages of disorganization of
connective tissue are fibrinoid changes as swelling and necrosis, refer them to:
A. *irreversible
B. reversible
C. partially reversible
D. border
E. reversible
79. Phase of disorganization of connective tissue such as fibrinoid changes,
swelling and necrosis, refer them to:
A. *irreversible
B. reversible
C. partially reversible
D. border
E. reversible
80. Fibrinoidnye changes (swelling and necrosis) in the rheumatic conditions are
characterized by:
A. *homogenization of collagen fibers with infiltration of plasma proteins
B. homogenization of nerve fibers with infiltration their proteins
C. homogenization keloidnyh fibers with infiltration their proteins
D. granulyatsionnoy homogenization of tissue with infiltration plasma
proteins
E. homogenization keloid fibers with infiltration their proteins
81. What are fibrinoid changes (swelling and necrosis), rheumatic conditions:
A. *homogenization of collagen fibers with prosyakannyam of plasma
proteins
B. homogenization of nerve fibers with prosyakannyam their proteins
C. homogenization keloidnyh fibers with prosyakannyam their proteins
D. granulyatsionnoy homogenization of tissue with prosyakannyam plasma
proteins
E. homogenization keloidnyh fibers with prosyakannyam their proteins
82. Stage granulematous rheumatic conditions morphologically evident:
A. *Cell incendiary reaction
B. Monokin reaction
C. cytokine response
D. polymorphonuclear cell inflammation of the reorganization
E. -83. What stage is morphologically manifested granulematous rheumatic
conditions:
A. *Cell incendiary reaction
B. Monokin reaction
C. cytokine response
D. polymorphonuclear cell inflammation of the reorganization
E. -84. The first stage granulematous in the form of nodular formations in the stroma
of heart described:
A. *Ashof
B. Bischoff
C. Amosov
D. Langerhans
E. Amosov
85. Who first described the stage granulematous in the form of nodular formations
in the stroma of heart:
A. *Ashof
B. Bischoff
C. Amosov
D. Langerhans
E. Amosov
86. Three phases of development of rheumatic granulomas – alteration , exudation
proliferations and sclerotic identified:
A. *Talalaev
B. Bischoff
C. Amosov
D. Ashof
E. Amosov
87. Who first identified three phases of development of rheumatic granulomas –
alteration? Exudation and proliferation and sclerotic:
A. Amosov
B. *Talalaev
C. Bischoff
D. Amosov
E. Ashtof
88. The entire development cycle granuloma rheumatic conditions are:
A. *4-6 months
B. 1 year
C. 1-2 years
D. 9-12 months
E. 2 years
89. The entire development cycle granuloma rheumatic conditions are:
A. 9-12 Months
B. 3 years
C. 4-6 months
D. 1 year
E. 1-2 years
90. Specify how much of the entire development cycle granuloma rheumatic
conditions:
A. 9-12 Months
B. 5 years
C. *4-6 months
D. 1 year
E. 1-2 years
91. Alterative and exudative phase of development characterized by the
accumulation of rheumatic granuloma:
A. * macrophages around the focus fibrinoid necrosis
B. lymphocytes and the formation of follicles
C. lymphocytes and the formation of follicles in breeding centers
D. neutrophils, lymphocytes, histiocytes
E. neutrophils, lymphocytes, histiocytes
92. What is alteration, axudative phase development of rheumatoid granuloma:
A. accumulation of neutrophils, lymphocytes, histiocytes
B. accumulation of lymphocytes and the formation of follicles in breeding
centers
C. *a macrophages ccumulation around the focus fibrinoid necrosis
D. accumulation of lymphocytes and the formation of follicles
E. accumulation of lymphocytes and the formation of follicles in breeding
centers
93. Indicate what is alteration and exudation phase development of rheumatoid
granulomy:
A. accumulation of neutrophils, lymphocytes, histiocytes
B. accumulation of neutrophils,
C. *accumulation around the campfire fibrinoidnogo necrosis of
macrophages
D. accumulation of lymphocytes and the formation of follicles
E. accumulation of lymphocytes and the formation of follicles in breeding
centers
94. The presence of «sclerosing granuloma» refers to:
A. *fading process
B. exacerbation of the process after remission
C. active attack of rheumatism
D. recovery process after an active attack of rheumatism
E. slight slowdown in the pace of development
95. At that indicates the existence of «sclerosing granuloma»:
A. active attack of rheumatism
B. *fading process
C. exacerbation of the process after remission
D. recovery process after an active attack of rheumatism
E. slight slowdown in the pace of development
96. With the development of rheumatoid granulomy phase sclerosis fibroblasts
replace the necrotic zone fibrinoid are synthesized:
A. * argyrophil and collagen fibers
B. mukoid swelling
C. fibrinoid swelling
D. hyalinosis
E. amyloidosis
97. With the development of rheumatoid granuloma phase sclerosis fibroblasts
replace the necrotic zone fibrinoid, are synthesized:
A. * argyrophil and collagen fibers
B. Mukoid swelling
C. Fibrinoid swelling
D. Hyalinosis
E. amyloidosis
98. With the development of rheumatoid granuloma phase sclerosis granuloma
recruits properties scar, a sign indicates:
A. *remission of disease
B. Acute period
C. escalation process after remission
D. rebuilding process after the strong attack of rheumatism
E. slight slowdown in the pace of development
99. What period of the disease shows the development of rheumatoid granuloma
in the sclerosis phase:
A. acute period
B. *remission of disease
C. escalation process after remission
D. rebuilding process after the strong attack of rheumatism
E. slight slowdown in the pace of development
100.
In a typical course of rheumatism in the first place is damaged:
A. *heart
B. only the large joints
C. only small joints
D. Only kidney
E. liver
101.
Which body is damaged in the first place, with a typical current of
rheumatism:
A. *heart
B. only the large joints
C. only small joints
D. Only kidney
E. liver
102.
In a typical course of rheumatism in the first damaged heart, which
develops a pathological process, such as:
A. *endocarditis and myocarditis
B. endovaskulitis coronary arteries
C. pericarditis and pankarditis
D. perivaskulitis coronary arteries
E. cardiosclerosis
103.
What pathologic process develops in the heart, with a typical current of
rheumatism:
A. *endocarditis and myocarditis
B. endovaskulitis coronary arteries
C. pericarditis and pankarditis
D. perivasculitis of coronary arteries
E. cardiosclerosis
104.
Endocarditis rheumatic conditions on the localization can be:
A. *valvular
B. mural
C. transmural
D. epicardial
E. pericardial
105.
What endocarditis rheumatic conditions may be to localize:
A. *valvular
B. mural
C. transmural
D. epicardial
E. pericardial
106.
Endocarditis (inflammation endocardium) rheumatic conditions on the
localization can be:
A. *chordal
B. mural
C. transmural
D. epicardial
E. pericardial
107.
How to localize endocarditis (inflammation of endocardium) rheumatic
conditions:
A. *chordal
B. mural
C. transmural
D. epicardial
E. pericardial
108.
Endocarditis (inflammation of endocardium) rheumatic conditions on
the localization can be:
A. *subendocardial
B. mural
C. transmural
D. epicardial
E. pericardial
109.
How to localize endocarditis (inflammation of endocardium) may be
rheumatic conditions:
A. *subendocardial
B. mural
C. transmural
D. epicardial
E. pericardial
110.
Often the rheumatoid process affects:
A. *mitral and aortic valves
B. tricuspid valve
C. valve pulmonary artery
D. valve and the pulmonary artery and tricuspid valve
E. --
111.
What most amazes rheumatic process:
A. *mitral and aortic valves
B. tricuspid valve
C. valve pulmonary artery
D. valve and the pulmonary artery valve tricuspid
E. -112.
What kind-hearted often surprised when rheumatic process:
A. *mitral and aortic valve
B. tricuspid valve
C. valve pulmonary artery
D. valve and the pulmonary artery tricuspid valve
E. -113.
Depending on the prevalence alterative or regenerative processes
distinguish the type of rheumatic valve endocarditis:
A. *diffuse endocarditis
B. mural endocarditis
C. transmural endocarditis
D. epicardial endocarditis
E. pericarditis
114.
Depending on the prevalence alterative or regenerative processes
distinguish the type of rheumatic valve endocarditis:
A. *diffuse endocarditis
B. mural endocarditis
C. transmural endocarditis
D. epicardial endocarditis
E. pericarditis
115.
Diffuse endocarditis as a kind of rheumatic valve endocarditis is
characterized by diffuse:
A. *mucoid swelling of connective tissue without damaging the
endothelium
B. fibrinoid necrosis of connective tissue and endothelial
C. fibrinoid edema of connective tissue and endothelial damage
D. hyalinosis of connective tissue and endothelial
E. amyloidosis
116.
What is diffuse endocarditis as a kind of rheumatic valve endocarditis:
A. -B. *mukoid edema of connective tissue without damaging the endothelium
C. fibrinoid necrosis of connective tissue and endothelial
D. fibrinoid edema of connective tissue and endothelial damage
E. hyalinosis of connective tissue and endothelial
117.
Depending on the prevalence alterative or regenerative processes
distinguish the type of rheumatic valve endocarditis:
A. *warty acute endocarditis
B. Acute mural endocarditis
C. transmural subacute endocarditis
D. Acute endocarditis
E. --
118.
What types of rheumatic valve endocarditis differ depending on the
prevalence alterative or regenerative processes:
A. *warty acute endocarditis
B. Acute endocarditis
C. Acute mural endocarditis
D. transmural subacute endocarditis
E. Acute endocarditis
119.
Acute papillose endocarditis rheumatic conditions is accompanied by:
A. *fibrinoid changes in connective tissue and endothelial desquamation
with delay in the sites of injury trombotic masses in the form of warts
B. mucoid changes in connective tissue with a delay trombotic mass in the
form of warts
C. -D. fibrinotic changes in connective tissue and endothelium of the
postponement of the masses of warts
E. fibrinoid changes in cardiomyocytes and desquamation epicardium with
postponement of warts
120.
What is accompanied by a sharp papillose endocarditis rheumatic
conditions:
A. *fibrinoid changes in connective tissue and endothelial desquamation
with delay in the sites of injury trombotic masses in the form of warts
B. mukoid changes in connective tissue with a delay trombotic mass in the
form of warts
C. -D. fibrinoid changes in connective tissue and endothelium of the masses in
the form of postponement of warts
E. fibrinoid changes in cardiomyocytes and desquamation epicardium with
postponement of warts
121.
Depending on the prevalence alterative or regenerative processes
distinguish the type of rheumatic valve endocarditis:
A. *fibroplastic endocarditis
B. intramural endocarditis
C. -D. transmural endocarditis
E. epicardial endocarditis
122.
Specify the type of rheumatic valve endocarditis depending on the
prevalence alterative or regenerative processes:
A. -B. *fibroplastic endocarditis
C. mural endocarditis
D. transmural endocarditis
E. epicardial endocarditis
123.
Depending on the prevalence alterative or regenerative processes
distinguish the type of rheumatic valve endocarditis - fibroplastichny
endocarditis, characterized by:
A. *growth of new tissue, scar revisions and valve insufficiency (acquired
heart disease)
B. -C. mucoid changes in connective tissue with a delay trombotic mass in the
form of warts on the valves
D. fibrinogen changes in connective tissue valves
E. fibrinoid changes in cardiomyocytes and endothelial
124.
Depending on the prevalence alterative or regenerative processes
distinguish the type of rheumatic valve endocarditis:
A. *rotary-papillose endocarditis
B. re endocarditis
C. re-trombotic endocarditis
D. diffuse-papillose endocarditis
E. -125.
Rotary-papillose endocarditis is characterized by repeated disruptions:
A. *newly formed connective tissue, endothelial injury and fibrin
accumulation on a background of sclerosis and hyalinosis valve
B. collagen tissue in the middle of the valve with the proliferation of
endothelium
C. -D. fibrous tissue valve with diffuse deposition of fibrin
E. intact endothelium and the postponement of fibrinogen
126.
Indicate the degree of disorganization of connective tissue in rotarypapillose endocarditis:
A. -B. *neoformation of connective tissue, endothelial damage and fibrin
deposition on the background of sclerosis and hyalinosis valve
C. collagen tissue in the middle of the valve with the proliferation of
endothelial
D. fibrous tissue valve with diffuse deposition of fibrin
E. intact endothelium and delays fibrinogen
127.
Myocarditis (inflammation of the myocardium) - permanent
manifestation of rheumatic disease, a form of it are:
A. *granulematozny
B. -C. focal parenchymatous
D. diffuse ekssudative parenchymatous
E. focal interstitial
128.
Specify the form myocarditis (inflammation of the myocardium) permanent manifestation of rheumatism:
A. -B. *granulematozny
C. focal parenchymatous
D. diffuse exudative parenchymatous
E. focal interstitial
129.
Granulematous myocarditis rheumatic conditions characterized by the
presence of "blooming", and sclerotic rheumatic granulomas:
A. *in the perivascular connective tissue of the myocardium
B. in cardiomyocytes
C. vascular endothelium in myocardial
D. in epycardium
E. -130.
Specify the localization of "blooming", sclerotic granulomas rheumatic
conditions:
A. -B. *in the perivascular connective tissue of the myocardium
C. in cardiomyocytes
D. vascular endothelium in myocardial
E. in epicardium
131.
Exudative diffuse interstitial myocarditis is characterized by:
A. *interstitial significant infiltration of lymphocytes, gistiocyts,
neutrophils and eosinophils in the presence of single granulomas AshofTalalaev
B. -C. slight infiltration of lymphocytes intersticial, gistiocyts, neutrophils and
eosinophils in the presence of single granulomas Ashof-Talalaev
D. interstitsiya significant infiltration of lymphocytes in the presence of
single granulomas Ashof-Talalaev
E. slight infiltration of neutrophils in the presence of interstitial isolated
granulomas Ashof-Talalaev
132.
What is exudative diffuse interstitial myocarditis:
A. *interstitial significant infiltration of lymphocytes, gistiocites,
neutrophils and eosinophils in the presence of single granulomas AshofTalalaev
B. slight infiltration of lymphocytes interstitial, gistiocyts, neutrophils and
eosinophils in the presence of single granulomas Ashof-Talalaev
C. interstitsiey significant infiltration of lymphocytes in the presence of
single granulomas Ashof-Talalaev
D. -E. slight infiltration of neutrophils in the presence of interstitial isolated
granulomas Ashof-Talalaev
133.
Development of myocarditis cardiosclerosis ends with:
A. *favorable course of rheumatism
B. -C. disadvantaged during rheumatism
D. in the frequent recurrence of rheumatism
E. in rheumatism with high activity indices of blood
134.
Cardiosclerosis is the result of miocarditis with:
A. -B. *favorable course of rheumatism
C. disadvantaged during rheumatism
D. During the frequent recurrence of rheumatism
E. During rheumatism with high activity indices of blood
135.
Prior to surgery, which of the following is the best medical therapy for a
newborn infant with transposition of the pulmonary artery and aorta?
A. *Give prostaglandin E2 to keep the ductus arteriosus open
B.
C.
D.
E.
Give prostaglandin F2 to close the ductus arteriosus
Give oxygen to keep the ductus arteriosus open
Give indomethacin to keep the ductus arteriosus open
Give indomethacin to close the ductus arteriosus
136.
Name the stage of myocardial infarction.
A. Dystrophic
B. Metabolic
C. Coronary
D. All listed above
E. * Necrotic
137.
Name the stage of the course of hypertension.
A. Metabolic
B. Discirculatory
C. Dystrophic
D. Dystonic
E. * Morphological changes in vessels
138.
Name the type of heart attack, according to localization of necrosis foci.
A. Second
B. Mixed
C. Focal
D. Palindromic
E. * Transmural
139.
Name the type of macroscopic changes in cardiac blood vessels at 1st
stage of atherosclerosis
A. Liposclerosis
B. Hyalinosis
C. Atheromatosis
D. Lipoidosis
E. * Lipid spots and stripes
140.
Name the type of macroscopic changes in large vessels at
atherosclerosis.
A. Arteriolosclerosis
B. Lipoidosis
C. Sclerosis
D. Petrification
E. * Fatty spots and stripes
141.
The nature of foam cells in atherosclerotic plaques
A. Granulocytes
B. Lymphocytes
C. Plasmocells
D. Eosinophils
E. * Macrophages
142.
Pathogenetic factors of ischemic heart disease.
A. Hyperlipidemia, smoking
B. Arterial hypertension
C. Excessive body weight
D. Sedentary lifestyle
E. * All listed above
143.
Possible causes of death at myocardial infarction.
A. Ventricular fibrillation
B. Asystolia
C. Cardiogenic shock
D. Acute heart failure
E. * All listed above
144.
Name the possible extracardiac complications which are associated with
acute heart aneurysm.
A. Pneumonia
B. Cirrhosis
C. Pulmonary gangrene
D. Hemorrhage in the brain
E. * Ischemic infarct of brain
145.
Presence of "flowering granuloma" refers to:
A. escalation process after remission
B. recovery process after an active attack of rheumatism
C. some slowdown of development
D. * active phase of process
E. process of remission
146.
Atherosclerosis of abdominal aorta can be complicated by development
of:
A. Brain infarction
B. Myocardial infarction
C. Pulmonary heart
D. Pulmonary gangrene
E. * Intestinal gangrene
147.
Specify the factor that is important in the pathogenesis of atherosclerosis.
A. Predominance of high density lipoproteins in plasma
B. Predominance of very high density lipoproteins in plasma
C. Hypercalciumaemia
D. Dysproteinaemia
E. * Hypercholesterolaemia
148.
Specify the type of heart attack (myocardial infarct) which developed
after first signs of ischemia.
A. Necrotic infarct
B. Ischemic infarct
C. Transmural infarct
D. Focal infarct
E. * Primary infarct
149.
Stages in the course of benign hypertension.
A. Prelipid
B. Clinical
C. Vascular endothelial ulceration
D. Cirrhotic
E. * Changes in organs due to changes in arteries
150.
The reason of death at malignant nephrosclerosis.
A.
B.
C.
D.
E.
Acute heart failure
Acute liver failure
Chronic renal failure
Chronic heart failure
* Acute renal failure
151.
What is the main factor in pathogenesis of hypertension.
A. Age factor
B. Metabolic dysfunction factor
C. Intake of lipoproteins
D. Liver factor
E. * Kidney factor
152.
The modern theory of the pathogenesis of atherosclerosis.
A. Theory of alimentary infiltration
B. Emotional theory
C. Trombogenic theory
D. Golgi metabolic theory
E. * Receptor theory
153.
The modern theory of the pathogenesis of hypertension.
A. Metabolic theory
B. Receptor theory
C. * Genetic pathology of cell membranes
D. Golgi metabolic theory
E. Emotional theory
154.
Time of necrotizing stage at myocardial ischemia after the first signs.
A. To 6-8 hours
B. To 10-12 hour
C. To 6 hours
D. To 2:00
E. * More than 12 hours
155.
What type of macroscopic complication is developed in arterial intima at
the atherosclerosis.
A. Fat and protein accumulation
B. Hyalinosis
C. Fibrous plaque
D. Fatty strips
E. * Mural hematoma
156.
Type of calcification in atherosclerotic "plaque" according to
pathogenesis.
A. Metabolic
B. Metastatic
C. Secondary
D. Primary
E. * Dystrophic
157.
Type of myocardial infarction depending on the timing of development
since the first signs of ischemia.
A. Hemorrhagic, ischemic
B. Acute
158.
159.
160.
161.
162.
163.
164.
165.
C. Subacute
D. Chronic
E. * Recurrent, primary
Types of hypertension depending on the activity pressure renal systems.
A. Primary
B. Secondary
C. Hormonal
D. Hypovolemic
E. * Vasoconstruction
Types of hypertension depending to nature of its passing.
A. Primary
B. Secondary
C. Idiopathic
D. Subacute
E. * Malignant
Typical changes of arterioles at hypertension.
A. Atherosclerosis
B. Liposclerosis
C. Atheromatosis
D. Atherocalcinosis
E. * Hyalinosis
What type of blood vessels does atherosclerosis affect?
A. * Aorta
B. Hollow vein
C. Venules
D. Thoracic duct
E. Capillaries
What are clinical and anatomic form of hypertension.
A. Functional
B. Escherichia
C. Changes in vessels
D. Changes in extremities
E. * Renal
What are clinical and anatomic form of hypertension.
A. Mesenteric
B. Portal
C. Mediastinal
D. Pulmonary
E. * Cerebral
What are complications of myocardial infarction?
A. Chronic aneurysm
B. Heart disease
C. Diffuse cardiosclerosis
D. Coronary artery spasm
E. * Ventricular fibrillation
What are synonyms of hypertension disease?
A. Symptomatic hypertension
B.
C.
D.
E.
166.
167.
168.
169.
170.
171.
172.
173.
Secondary hypertension
Idiopathic hypertension
Neurogenic hypertension
* Primary hypertension
What factor is most important at atherosclerosis development?
A. Salt factor
B. Protein starvation factor
C. Carbohydrate rich foods factor
D. Alcohol factor
E. * Age factor
What are the main factors in the hypertension development?
A. Virus infection
B. Protein starvation
C. Food rich of carbohydrates
D. Alcohol
E. * Stress, Salt
To what change of heart does acute ischemic heart disease lead?
A. Metabolic myocardial damage
B. Cardiomyoliposis
C. Granulematous myocarditis
D. Cardiomyopathy
E. * Myocardial infarction
To what change of heart does acute ischemic heart disease lead?
A. Cardiomyoliposis
B. Diffuse small focal cardiosclerosis
C. Tiger heart
D. Necrotic cardiomyopathy
E. * Myocardial infarction
What complications can develop in patients with acute aneurysm?
A. Wall aneurysm rupture
B. Cavity pericardial tamponade
C. Renal infarction
D. Myocardial ischemic brain
E. * All listed above
What is acute aneurysm of heart?
A. Acute ischemic heart disease
B. Forms of chronic ischemic heart disease
C. Cardiomegaly complications
D. Inherited defect of heart
E. * Balloon-like bulge in the wall of heart at the infarction.
What is characteristic for acute aneurysm?
A. Situates in the wall of the right ventricle most often
B. Wall aneurysms before scar tissue
C. Wall aneurysms presented thrombotic masses
D. Calcified aneurysm wall
E. * Situates in the wall of the left ventricle most often
What is characteristic for primary wrinkled kidney at atherosclerosis?
A.
B.
C.
D.
E.
Grained surface, reducing the size
Expansion of sinuses and cups
Expansion cortex
The presence of small scars in the parenchyma
* Reducing the size and formed large nodular surface
174.
What is characteristic of transmural myocardial infarction?
A. Focal necrosis in papillary muscles only
B. Fibrinous endocarditis
C. Only subendocardial focal necrosis
D. Warty endocarditis
E. * Myocardial necrosis through all layers
175.
What is myocardial infarction?
A. Expressions of cardiac arrhythmias
B. Forms of chronic ischemic heart diseases
C. Hemorrhagic necrosis
D. Cardiomyopathy
E. * Vascular necrosis of heart
176.
What is the relationship between arteriosclerosis and atherosclerosis?
A. Arteriosclerosis - a kind of atherosclerosis
B. Atherosclerosis - a kind elastofibrozu
C. Atherosclerosis - senile arteriosclerosis
D. Atherosclerosis - allergic arteriosclerosis
E. * Atherosclerosis - a metabolic arteriosclerosis
177.
What layer of arterial wall does atherosclerosis injure?
A. Adventitial
B. All layers
C. Media
D. Perivascular tissue
E. * Internal
178.
What blood vessels does hypertension disease injure mostly?
A. Venules
B. Capillaries
C. Muscular type arteries
D. Arteries of muscle-elastic type
E. * Arterioles
179.
What pathological conditions may lead to symptomatic hypertension?
A. CNS Diseases
B. Vascular Diseases
C. Kidney
D. Endocrine system diseases
E. * All of the above
180.
With the development of rheumatic granuloma at the sclerosis phase
fibroblasts replaces fibrynoid necrosis zones, and synthesized:
A. mucoid fibers
B. fibrynoid fibers
C. hyaline fibers and colloid
D. All the above
E. * Argyrophil and collagen fibers
181.
One of the stage of granuloma development is the scar formation, this
moment shows about ...
A. Escalation process after remission
B. Recovery process after an active attack of rheumatism
C. Some slowdown in development
D. Some activation process
E. * Phaze of remission
182.
Acute rheumatic warty endocarditis is accompanied by:
A. * fibrinoid changes and endothelial desquamation with thrombus
formation in the warts forms
B. mucoid changes of connective tissue with thrombus formation in the
warts forms
C. -D. fibrinoid changes of connective tissue and endothelium growthing in the
warts forms
E. fibrinoid changes of cardiomyocytes and epicardium growthing in the
warts forms
183.
Alterativ-exudativ phase of rheumatism characterized by rheumatic
granuloma formation with accumulation of:
A. * macrophages with hypertrophic nuclei around the fibrinoid necrosis in
fan-like manner form
B. lymphocytes and the formation of follicles in center
C. neutrophils, lymphocytes, histiocytes near the
D. neutrophils, lymphocytes, histiocytes
E. -184.
A clinical syndrome is characteristic for the renal amiloidisis ...
A. Hepatonephrotic
B. Lowe's syndrome
C. Alport's syndrome
D. * Nephrotic syndrome
E. Nephritic syndrome
185.
A gouty kidney develops as a result of abnormality of metabolism of
A. Chromoproteins
B. Lipids
C. * Purines
D. Albumins
E. Carbonhydrates
186.
Advanced cervical carcinoma can extend by direct continuity to all of the
following, EXCEPT:
A. Urinary bladder
B. Ureters
C. * Colon transversum
D. Rectum
E. Peritoneum
187.
All of the following clinical features are likely to be found in nephrotic
syndrome, EXCEPT:
A.
B.
C.
D.
E.
Proteinuria
Hypoalbuminemia
* Hematuria
Hyperlipidemia
Edema
188.
All of the following conditions predispose to urolithiasis, EXCEPT:
A. * Sickle cell nephropathy
B. Hyperparathyroidism
C. Gout
D. Proteus pyelonephritis
E. Enteric hyperoxaluria
189.
All of the following morphologic features characterize the ovary
adenocarcinoma cells, EXCEPT:
A. Enlarged nucleoli
B. Atypical mitoses
C. Variation in size and shape
D. Hyperchromatic nuclei
E. * Nucleus inclusions
190.
All of the following statements correctly describe chronic pyelonephritis,
EXCEPT:
A. It causes asymmetrically scarred kidneys
B. It is associated with vesicoureteral reflux in most cases
C. * It spares the calyces and pelvis
D. It may produce thyroidization of tubules
E. It is an important cause of secondary nephrosclerosis
191.
All of the following statements correctly describe analgesic abuse
nephropathy, EXCEPT:
A. It is characterized by tubulo-interstitial component
B. It is often caused by phenacetin
C. It causes inability to concentrate urine
D. It often improves with drug withdrawal
E. * It predisposes to the development of renal cell carcinoma
192.
All of the following statements correctly describe renal artery stenosis,
EXCEPT:
A. It is an uncommon form of hypertension
B. It is the most common curable form of hypertension
C. It is usually caused by atherosclerotic plaque
D. It produces high renin levels in the venous blood of the ischemic kidney
E. * It is treated by hemodialysis
193.
All of the following statements regarding Goodpasture's syndrome are
true, EXCEPT:
A. Patients present with hemoptysis and hematuria
B. Death occurs due to uremia and pulmonary hemorrhage
C. Electron microscopy shows the absence of electron-dense deposits
D. * Immunofluorescence reveals granular deposits of IgG in the glomeruli
E. Immunofluorescence reveals linear deposits of IgG in the glomeruli
194.
All of the following statements regarding postinfectious
glomerulonephritis are true, EXCEPT:
A. The disease follows streptococcus infection
B. Electron microscopy shows large subendothelial immune-type deposits
C. The histologic picture is that of diffuse proliferative glomerulonephritis
D. The clinical picture is characteristic of acute nephritis
E. * Most affected children develop chronic renal failure
195.
All statements concerning chorion-carcinoma are true, EXCEPT:
A. The tumor has a dimorphic pattern
B. Hemorrhages and necrosis are present
C. The tumor is malignant
D. The tumor is composed of cytotrophoblast and syncytiotrophoblast cells
E. * The tumor produces chorionic villi
196.
All statements concerning leiomyomas are true, EXCEPT:
A. They regress or calcify after castration or menopause
B. * They do not respond to estrogens
C. They may undergo rapid increase in size during pregnancy
D. Their cause is unknown
E. They are found in 25% of reproductive women
197.
An immunofluorescence-stained kidney specimen from a patient with
poststreptococcal glomerulonephritis is likely to show which of the following:
A. * Granular deposits of Ig G
B. Linear deposits of Ig G
C. Granular deposits of IgA
D. Linear deposits of streptococcal antigen
E. Granular deposits of streptococcal antigen
198.
An important factor in cervical oncogenesis is which of the following:
A. * HPV-infection
B. Herpes simplex
C. Herpes zoster
D. Human immunodeficiency virus
E. Respiratory syncytial virus
199.
At extracapillar productive glomerulonephritis half moons are formed by
A. Red corpuscles
B. Neutrophilic leucocytes
C. Red corpuscles and neutrophilic leucocytes
D. Exfoliated endothelium and fibrin
E. * Exfoliated renal epithelium and fibrin
200.
Benign nephrosclerosis is characterized by all of the following,
EXCEPT:
A. Narrowing of the lumen of the arterioles and small arteries
B. Thickening and hyalinization of the vessels' walls
C. * Deposition of amyloid within the Bowman space
D. Foci of tubular atrophy
E. Deposition of collagen within the Bowman space
201.
Changes in a colon at uremia are
A. Granulomatous inflammation
B.
C.
D.
E.
Stricture
Polyposis
Ulcerous colitis
* Fibrinous [diphtheritic] colitis
202.
Chorioncarcinoma is commonly associated with which of the following:
A. Systemic hypertension
B. * Pregnancy
C. Obesity
D. Oral contraceptive steroid use
E. Diabetes mellitus
203.
Conditions leading to endometrial hyperplasia include all of the
following, EXCEPT:
A. Polycystic ovarian disease
B. Functioning granulosa cell tumors of the ovary
C. Excessive cortical function (cortical stroma hyperplasia)
D. * Endometriosis
E. Estrogen replacement therapy
204.
Criteria to differentiate between benign and malignant tumors are all of
the following, EXCEPT:
A. Maturity
B. Rate and character of growth
C. * Edema
D. Metastases
E. Local invasion
205.
Diabetes mellitus is associated with all of the following renal disorders,
EXCEPT:
A. Diffuse glomerulosclerosis
B. Nodular glomerulosclerosis
C. Benigh nephrosclerosis
D. * Urate nephropathy
E. Acute pyelonephritis
206.
Distant hematogenic metastases of invasive cervical carcinoma occur in
all of the following, EXCEPT:
A. Liver
B. Lungs
C. Bone marrow
D. * Lymph nodes
E. Kidney
207.
Endometrial hyperplasia is associated with which of the following:
A. High estrogenic stimulation with normal progestational activity
B. Normal estrogenic stimulation with increased progestational activity
C. * High estrogenic stimulation with diminished progestational activity
D. High estrogenic stimulation and progestational activity
E. Normal estrogenic stimulation and progestational activity
208.
First hematogenous metastases of the endometrial carcinoma can be
found in which of the following organs:
A. Bones
B.
C.
D.
E.
Regional lymph nodes
* Lungs
Liver
Central nervous system
209.
First lymphogenous metastases of the papillary mucinous
cystadnocarcinoma of the ovary can be found in which of the following organs:
A. * Pelvic lymph nodes
B. Aortic lymph nodes
C. Liver
D. Lungs
E. Bones
210.
First metastases of the Chorioncarcinoma can be found in which of the
following organs:
A. Liver
B. * Lung
C. Kidney
D. Bones
E. Brain
211.
Give the determination of gynecomastia
A. Non-cancerous cystous dysplasia of mammary gland
B. Non-cancerous hyperplasia of mammary gland
C. Malignant dysplasia of mammary gland
D. Non-cancerous sclerotic dysplasia of mammary gland
E. * Non-cancerous dysplasia of breast in men
212.
Glomerular injury caused by circulating complexes occurs in all of the
following disorders, EXCEPT:
A. Syphilis
B. Goodpasture's syndrome
C. Hepatitis B
D. Systemic lupus erythematosus
E. * Lung cancer
213.
Hematuria is a characteristic clinical feature of all of the following
diseases, EXCEPT:
A. Glomerulonephritis
B. * Malakoplakia
C. Nephrolithiasis
D. Renal cell carcinoma
E. Bladder papilloma
214.
Higher frequency of carcinoma of the endometrium is commonly
associated with all of the following, EXCEPT:
A. * Young age
B. Obesity
C. Diabetes mellitus
D. Hypertension
E. Infertility (anovulatory cycles)
215.
Histologic features of malignant nephrosclerosis include all of the
following, EXCEPT:
A.
B.
C.
D.
E.
Fibrinoid necrosis of arterioles
Medial thickening of arterioles
* Fibromuscular dysplasia of the renal artery
Renal artery thrombosis
Focal renal parenchymal infarction
216.
Histological type of of high quality hyperplasia of prostatic gland
A. Muscular type
B. Stromal vascular type
C. Amyloid type
D. Fibroid cystous type
E. * Mixed type
217.
Hydronephrosis is caused by all of the following, EXCEPT:
A. * Chronic renal vein thrombosis
B. Large uterine leiomyoma
C. Renal calculi
D. Benign prostatic hypertrophy
E. Papillary transitional cell carcinoma of the ureter
218.
Hydronephrosis is characterized by all of the following, EXCEPT:
A. * Kidney infarct
B. Thinning of the renal parenchyma
C. Dilatation of the renal pelvis
D. Dilatation of the renal calyces
E. Progressive atrophy of the kidney
219.
Immunologicaly mediated glomerulonephritis all of the following cells
contribute to the glomerular injury, EXCEPT:
A. * Mast cells
B. Macrophages
C. Platelets
D. Neutrophils
E. Mesangial cells
220.
In case of poisoning by mercuric chloride there is a necrosis of
A. Cardiac hystiocytes
B. * Epithelium of renal tubules
C. Glomerular system of kidneys
D. Renal stroma
E. Wall of ureter
221.
In case of poisoning by mercuric chloride there is a necrosis of
A. Brain cells
B. * Epithelium of renal tubules
C. Glomerular system of kidneys
D. Liver stroma
E. Wall of ureter
222.
In nonobstructive chronic pyelonephritis the most common way for
bacteria to gain entrance into the kidney is which of the following:
A. Arterial bloodstream
B. The lymphatics
C. Venous bloodstream
D. * Vesicoureteral reflux
E. Aberrant arteriovenous shunts
223.
In pathogeny of glomerulonephritis a considerable role plays:
A. * Sensitizing of an organism
B. Reinfection
C. Toxicness of microbes
D. A presence of the inflammatory diseases of kidneys in anamnesis
E. Superinfection
224.
In the urinary tract obstruction all pathologic processes can be found.
EXCEPT:
A. Dilatation of the pelvis and calyces
B. * Ischemic tubular necrosis
C. Interstitial inflammation
D. Interstitial fibrosis
E. Glomerular and tubular atrophy
225.
Invasive cervical carcinoma can manifest in which of the following
patterns:
A. * Ulcerating tumor
B. Endocervical polyp
C. Grapelike projections
D. Paget's cancer
E. Flattened plaque cancer
226.
Mesangial cells can be characterized by all of the following properties,
EXCEPT:
A. Ingestion of macromolecules
B. Connection with Lacis cells
C. Ability to contract
D. * Production of renin
E. Production of basement membrane proteins
227.
Microscopically, simple hyperplasia of endometrium is characterized by
all of the following, EXCEPT:
A. * Pressed endometrial glands
B. Dilated endometrial glands
C. Increased gland-to-stroma ratio
D. Abundant stroma
E. Complex ramified endometrial glands
228.
Morphological form of subacute glomerulonephritis is:
A. Mesangial membranous glomerulonephritis
B. * Extracapillar proliferative glomerulonephritis
C. Mesangial proliferative glomerulonephritis
D. Minimum changes
E. Intracapillar proliferative glomerulonephritis
229.
Most forms of chronic renal failure produce increased serum levels of all
of the following substances, EXCEPT:
A. * Calcium
B. Aldosterone
C. Phosphate
D. Parathormone
E. Renin
230.
Most frequent complication of glomerulonephritis is
A. Amiloidosis
B. Increase of arterial pressure
C. * Arteriolosclerotic kidney
D. Renal abscesses
E. Hematuria
231.
Name the complication of dishormonal hypertrophic prostatopathy?
A. Formation of the prostatic cysts.
B. Purulent melting of prostate
C. Sclerosis of prostate
D. Appearance of polypous excrescences
E. * Difficulty in the urine outflow
232.
Name a process, which is regarded as an inflammatory disease of
mucous membrane of uterus
A. Cystophorous hyperplasia
B. Pseudo erosion
C. Polypous excrescence
D. Endometriosis
E. * Endometritis
233.
Obstetrically related renal disease includes all of the following disorders,
EXCEPT:
A. * Nephrocalcinosis
B. Diffuse cortical renal necrosis
C. Acute ischemic tubular necrosis
D. Acute glomerulonephritis
E. Hydronephrosis
234.
Poor prognosis in chorioncarcinoma is associated with which of the
following features of the tumor...
A. Lack of the capsule
B. Absence of estrogen receptors
C. Presence of secondary necrosis and hemorrhages
D. Extensive angiogenesis
E. * Early spread of hematogenous metastases
235.
On gross examination, leiomyoma of the uterus is characterized by all of
the following, EXCEPT:
A. Intramural, submucosal or subserosal localization
B. Discrete multiple nodules
C. Firm consistence
D. Fibrous capsule
E. * Dark brown color
236.
On gross examination, leiomyoma of the uterus may be characterized by
which of the following:
A. Indistinct margins
B. Brown color
C. Soft consistence
D. * Fibrillated cut surface
E. Multicolored
237.
On gross examination, the hyperplasia of endometrium is characterized
by all of the following, EXCEPT:
A. Enlarged uterus
B. Thickened uterus wall
C. Foci of hemorrhages into endometrium
D. * Suppurative endometritis
E. Irregular thickened endometrium
238.
Patogenetic basis of glomerulonephritis is formed by:
A. * Immunoreactions on the basic membranes of glomerular capillaries
B. Exsudates reactions on the basic membranes of glomerular capillaries
C. Necrotic reactions on the basic membranes of glomerular capillaries
D. Exsudative necrotic reactions on the basic membranes of glomerular
capillaries
E. Proliferative reactions on the basic membranes of glomerular capillaries
239.
Picrofuchsin by van Gieson staining of leiomyoma of the uterus results
in which of the following:
A. Red color of tumor parenchyma
B. Blue color of tumor parenchyma
C. Orange color of tumor parenchyma
D. Yellow color of the tumor stroma
E. * Red color of the tumor stroma
240.
Proteinuria at glomerulonephritis is caused by
A. Purulent exsudate permeating into the urine
B. Hyperproteinemia
C. Purulent infiltration of stroma
D. * Damage of wall of glomerular capillares
E. Damage of tubular wall
241.
Renal diseases producing systemic hypertension include all of the
following, EXCEPT:
A. Acute glomerulonephritis
B. * Renal amyloidosis
C. Chronic glomerulonephritis
D. Chronic pyelonephritis
E. Renal vasculitis
242.
Risk factors for cervical cancer development include all of the following,
EXCEPT:
A. Early age at first intercourse
B. Multiple sexual partners
C. Oral contraceptive use
D. * Alcohol abuse
E. HPV- infection
243.
Structurally-functional unit of kidney is:
A. Kidney pyramid;
B. Kidney little body;
C. Kidney lobule;
D. Kidney bowl;
E. * Nephron
244.
Subendothelial granular electron-dense deposits can be found in which
of the following diseases:
A. Rapidly progressive glomerulonephritis
B. Side cell nephropathy
C. Membranous glomerulonephritis
D. * Systemic lupus erythematosus
E. Gouty nephropathy
245.
Symptomatic hypertension evolves most frequently from:
A. * Glomerulonephritis
B. Atherosclerosis of renal arteries
C. Thyrotoxicosis
D. Aldosteronism
E. Pheochromocytoma
246.
Systemic lupus erythematosus gives rise to all of the following patterns
of glomerular injury, EXCEPT:
A. Focal proliferative glomerulonephritis
B. Diffuse membranous glomerulonephritis
C. Diffuse proliferative glomerulonephritis
D. * Lipoid nephrosis
E. Mesangial proliferative glomerulonephritis
247.
The benign tumor characterized by the development of cysts lined by tall
columnar epithelial cells and formation of regular papillary structures is
referred to as:
A. Chorioncarcinoma
B. Adenoma
C. Adenocarcinoma
D. * Cystadenoma
E. Fibroadenoma
248.
The changes of epithelium of tubulis are in the shock stage of acute
kidney insufficiency
A. Metaplasia of epithelium
B. * Dystrophy of epithelium
C. Necrosis of epithelium
D. The changes are absent
E. Removing a layer by the layer of epithelium
249.
The changes of tubular epithelium in the oliguric stage of acute renal
insufficiency
A. Metaplasia of epithelium
B. Dystrophy of epithelium
C. * Necrosis of epithelium
D. Changes are absent
E. Exfoliation of epithelium
250.
The characteristic feature of chorioepithelioma is
A. Sclerosis of stroma
B. * Absence of stroma
C. Atrophy of chorionic epithelium
D. Presence of lymphatic vessels
E. Petrification
251.
The dishormonal disease of the sexual system of men is
A. * Gynecomastia
B. Prostatitis
C. Orchitis
D. Epididymitis
E. Cancer of testicles
252.
The dishormonal disease of the sexual system of men is
A. Prostatitis
B. * Adenoma of prostatic gland
C. Orchitis
D. Cancer of prostatic
E. Urethritis
253.
The dishormonal disease of the sexual system of women is
A. Cervicitis
B. Endometritis
C. * Mastopathy
D. Salpingitis
E. Cancer of the body of uterus
254.
The dishormonal disease of the sexual system of women is
A. * Endocervicosis
B. Oophoritis
C. Cystitis
D. Sarcoma of uterus
E. Endometritis
255.
The essence of pyelonephritis is
A. Autoimmune aggression of renal tissue
B. Non suppurative inflammation of glomerules
C. Necrosis of renal cortical layer of kidney
D. * Suppurative inflammation of kidney
E. Hyalinosis of glomerular vessels
256.
The factor least likely to cause acute pyelonephritis is which of the
following:
A. Pregnancy
B. Nephrolithiasis
C. Catheterization of the bladder
D. Prostatic hypertrophy
E. * Septicemia
257.
The leiomyoma parenchyma is characterized by all of the following,
EXCEPT:
A. Haphazardly situated cells
B. Uniformed in size and shape cells
C. * Small vessels
D. Oval nuclei of smooth muscle tumor cells
E. Long cytoplasmic processus of tumor cells
258.
The main complication of nephrolithiasis is
A. Hypertrophy of the left ventricle
B. Hypertension
C. * Hydronephrosis
D. Contracted [granular] kidney
E. Cancer of kidney
259.
The malignant tumor characterized by easily recognizable glandular
tubular pattern is referred to as:
A. Chorioncarcinoma
B. Adenoma
C. * Adenocarcinoma
D. Cystadenoma
E. Fibroadenoma
260.
The most common benign tumor of the ovary is which of the following:
A. * Cystadenoma
B. Adenocarcinoma
C. Cystadenocarcinoma
D. Papilloma
E. Fibroadenoma
261.
The most frequent clinical display of fibroid hyperplasia of mucous
membrane of uterus is:
A. Pain syndrome
B. Masculinization
C. * Metrorrhagia
D. Enlargement of uterus determined by ultrasonography
E. The characteristic clinical displays are absent
262.
The Russian scientist who was the first to describe the histogenesis of
chorioncarcinoma was:
A. R.Virchov
B. * M.N.Nikiforov
C. N.I.Pirogov
D. I.M.Setchenov
E. A.I.Abrikosov
263.
The stain that helps to detect precancer and cancer of the cervix in
smears is which of the following:
A. Weigert's
B. * Papanicolaou
C. Sudan III
D. Toluidin blue
E. Hematoxylin and eosin
264.
The stain used to differentiate leiomyoma and fibroma is which of the
following:
A. * Picrofuchsin
B. Congo red
C. Sudan III
D. Toluidin blue
E. Fuchselin
265.
The tubular epithelial cells in acute tubular necrosis are characterized by
all of the following pathologic features, EXCEPT:
A. Karyolysis
B. Plasmolysis
C. Plasmorrhexis
D. Plasmocoagulation
E. * Tubulorrhexis
266.
The tumor composed of clusters of cuboid cytotrophoblast cells
separated by streaming masses of syncytiotrophoblast giant cells with marked
atypia is referred to as:
A. * Chorioncarcinoma
B. Adenoma
C. Adenocarcinoma
D. Cystadenoma
E. Fibroadenoma
267.
Ultrastructural changes in children primary nephrotic syndrome involve
which of the following glomerular elements:
A. Endothelium
B. * Podocytes
C. Mesangium
D. Blood vessels
E. Basement membrane
268.
Uremia is associated with all of the following abnormalities, EXCEPT:
A. Peripheral neuropathy
B. Gastritis
C. * Polycythemia
D. Pericarditis
E. Diffuse alveolar damage
269.
What illness from the listed below relates tothe large mottled kidney
A. Amiloidosis of kidney
B. Chronic pyelonephritis
C. Nephrolithiasis
D. * Subacute glomerulonephritis
E. Chronic glomerulonephritis
270.
What abnormality develops at shock
A. * Acute kidney insufficiency
B. Chronic kidney insufficiency
C. Glomerulonephritis
D. Renal amiloidisis
E. Renal cirrhosis
271.
What belongs to the inherited diseases from these tubular patologies
A. Pyelonephritis
B. Necrotic nephrosis
C. * Tubular enzimatic pathology
D. “Myelome” kidney
E. “Shield-shaped” kidney
272.
What develops in case of prolong obstruction of ureters by stone in a
kidney
A. Heart attack
B. Gangrene
C. Glomerulonephritis
D. Amiloidosis
E. * Hydronephrosis
273.
What disease can be complicated by amyloidosis:
A. Atherosclerosis
B. Heart ischemic disease
C. * Fibrous cavernous tuberculosis of lungs
D. Croupous pneumonia
E. Hypertension
274.
What pathologic condition of the kidneys is caused by mercury
poisoning?
A. * Acute tubular necrosis
B. Renal papillary necrosis
C. Crescentic glomerulonephritis
D. Acute interstitial nephritis
E. Renal cell carcinoma
275.
What pathological process the hypertrophy of prostatic gland is related
to?
A. Chronic prostatitis
B. Acute prostatitis
C. Hyperfunction of sexual glands
D. Disorder of the urine outflow
E. * Hypofunction of sexual glands
276.
What process occurs in the wall of urinary bladder at prostatopathy?
A. Sclerosis
B. Fibrosis
C. Atrophy
D. The changes are absent
E. * Compensatory hypertrophy
277.
What type of epithelium covers the mucous membrane of urinoexcretory
ways?
A. Monolayer pavement epithelium;
B. * Multi-layered transitional
C. Multi-layered ciliated
D. Monolayer cylindrical
E. Multi-layered pavement unonkeratinizing.
278.
Which statement correctly characterizes membranous glomerulopathy?
A. It is the most common cause of nephrotie syndrome in children
B. Patients usually present with acute renal failure
C. It is characterized by diffuse proliferative glomerulonephritis
D. * Electron microscopy demonstrates numerous subepithelial
immunetype deposits
E. It is characterized by mesangial interposition phenomenon
279.
"Black measles" is characterized by which of the following:
A. * Hemorrhages
B. Hypermelanosis
C. Icterus
D. Hyperchromatosis
E. Multiple nevi
280.
A calcified focus (fibrocalcific scar) forming in the lung parenchyma and
in the hilar lymph node after the primary tuberculosis infection is also referred
to as:
A. Keloid
B. Granuloma
C. * Ghon focus
D. Aschoff-Pule focus
E. Simon focus
281.
Acute adrenal insufficiency syndrome is also referred to as:
A. * Waterhouse — Friderichsen syndrome
B. Kimmelstill-Wilson syndrome
C. Hamman-Rich syndrome
D. Zollinger-Ellison syndrome
E. Budd-Chiari syndrome
282.
All of the following pathologic processes can be found in the second
period of scarlet fever, EXCEPT:
A. Vasculitis
B. Acute glomerulonephritis
C. Arthritis
D. Endocarditis
E. * Amyloidosis
283.
All of these cells participate in immune response in primary lung
tuberculosis, EXCEPT:
A. Type I pneumocytes
B. * Alveolar macrophages
C. CD4+ helper T-cells
D. CD8+ suppressor T-cells
E. Double negative T-cells
284.
An infectious agent causing diphtheria is which of the following:
A. * Corynebacterium
B. Staphylococcus
C. Treponema
D. Bordetella
E. Esherichia
285.
Caseating destructive secondary tuberculosis includes all of the
following lesions, EXCEPT:
A. Caseation in the lung
B. Cavities in the lung
C. Caseation in the lung lymph nodes
D. * Miliary extrapulmonary lesions
E. Extrapulmonary caseation
286.
Complications of pyogenic meningitis include all of the following,
EXCEPT:
A. Leptomeningeal fibrosis
B. Hydrocephalus
C. Adhesive arachnoiditis
D. * Intracerebral hemorrhage
E. Focal encephalitis
287.
Complications of tuberculosis osteomyelitis include all of the following,
EXCEPT:
A. Tuberculosis arthritis
B. Sinus tract formation
C. Cold abscess formation
D. * Caseation in the lung
E. Amyloidosis
288.
Consequences of diphtheria include all of the following, EXCEPT:
A. Hyperplasia of the spleen
B. Fatty myocardial changes
C. * Systemic hemosiderosis
D. Polyneuritis
E. Focal necroses of the parenchyma of the organs
289.
Diphtheria is characterized by the derangement of all of the following
organs, EXCEPT:
A. Nasopharynx
B. Oropharynx
C. Larynx
D. Trachea
E. * Esophagus
290.
Erythematous rash in scarlet fever can be found on which of the
following parts of the body:
A. The trunk and inner aspects of the arms and legs
B. * The upper aspects of the arms and legs
C. The area about the mouth
D. The knees
E. The head
291.
Exotoxin in diphtheria damages all of the following organs, EXCEPT:
A. * Heart
B. Nervous system
C. Adrenals
D. Kidneys
E. Thyroid gland
292.
Favored targets for miliary extrapulmonary seeding are all of the
following organs, EXCEPT:
A. Bone marrow
B. Kidneys
C. Liver
D. * Uterus
E. Spleen
293.
Figuratively the influensa pneumonia is often referred to as:
A. Hemosiderosis of the lung
B. * Multicolored lung
C. Silicosis
D. Lung infarct
E. Farmer's lung
294.
Growth and multiplication of the tubercle bacilli in cavitary fibrocaseous
tuberculosis are favored by which of the following pathologic conditions:
A. Lymphatic drainage
B. Progressive hypoxia
C. Increased perfusion
D. * Increased oxygen tension
E. Sludging of blood in alveolar capillaries
295.
Healed lesions in primary tuberculosis include all of the following,
EXCEPT:
A. Fibrous incapsulation
B. * Caseous pneumonia
C. Fibrocalcific scar
D. Foci of ossification
E. Focal pleural adhesions
296.
Impairment of bronchociliary function in influenza results in which of
the following:
A. Fungal superinfection
B. * Bacterial superinfection
C. Protozoal superinfection
D. Mycoplasmal superinfection
E. Chlamydial superinfection
297.
The common morphologic feature seen in the skin vessels in epidemic
typhus fever is which of the following:
A. Abscess
B. Hyaline thrombus
C. Caseous necrosis
D. * VascuIitis
E. Hemangioma
298.
Influenza tracheitis is characterized by all of the following, EXCEPT:
A. Hyperemic and swollen mucosa
B. * Purulent exudate
C. Narrowing of trachea lumen
D. Pinpoint hemorrhages
E. Grey-yellow mucosa covering films within trachea
299.
Influenza virus of type A infects all of the following, EXCEPT:
A. Horses
B. Humans
C. Pigs
D. * Dogs
E. Birds
300.
Renal pathology that can be found in patients with scarlet fever is which
of the following;
A. * Glomerulonephritis
B. Pyelonephritis
C. Amyloidosis
D. Glomerulosclerosis
E. Lipoid nephrosis
301.
Macrophages in tuberculous granulomatous inflammation can transform
into which of the following cells:
A. Monocytes
B. Epithelial cells
C. * Epithelioid cells
D. Plasma cells
E. Lymphocytes
302.
Measles virus is transmitted by which of the following:
A. Milk
B. Food
C. * Respiratory droplets
D. Feces
E. Blood
303.
Measles virus multiplies inside all of the following cells, EXCEPT:
A. Upper respiratory epithelial cells
B. B lymphocytes
C. Macrophages
D. T lymphocytes
E. * Hepatocytes
304.
Meningococcal meningitis is most commonly characterized by which of
the following types of inflammation:
A. Serous
B. * Purulent
C. Fibrinous
D. Granulomatous
E. Catarrhal
305.
Meningococcal nasopharyngitis is characterized by which of the
following types of inflammation:
A. * Catarrhal
B. Serous
C. Fibrinous
D. Purulent
E. Diphtherial
306.
Microscopic features in diphtheria include all of the following,
EXCEPT:
A. Neutrophilic infiltration
B. Vascular congestion
C. Interstitial edema
D. * Hyaline deposition
E. Fibrin exudation
307.
Miliary tuberculosis is associated with which of the following pathologic
conditions:
A. Reinfection
B. Localized caseation in the lungs
C. Localized caseation in the lymph node
D. Primary infection
E. * Hematogenous tuberculosis
308.
Morphologically, measles pneumonia is characterized by all of the
following, EXCEPT:
A. Diffuse neutrophil infiltration of the bronchiolar walls
B. Destruction of some fragments of bronchiolar walls and squamous cell
metaplasia
C. Exudate with giant cells in the adjacent alveoli
D. Interstitial lung inflammation
E. * Large areas of hemorrhage in lung tissue
309.
Morphology of influenza pneumonia is characterized by all of the
following features, EXCEPT:
A. Diffuse neutrophil infiltration of the bronchiolar walls
B. Dense fibrin-rich neutrophil exudate into adjacent alveoli
C. Foci of necrosis and hemorrhages in lung tissue
D. * Granuloma formation in bronchiolar walls
E. Foci of necrosis and ulceration in bronchiolar mucosa
310.
Mycobacterium tuberculosis is characterized by all of the following
features, EXCEPT:
A. * Pili-forming
B. Aerobic
C. Non-spore-forming
D. Nonmotile
E. Red colored in acid-fast staining
311.
Nervous system pathology in diphtheria includes which of the following:
A. Intracerebral hemorrhage
B. Ischemic necrosis
C. Hydrocephalus
D. * Polyneuritis
E. Encephalitis
312.
Caseous focus in tuberculosis may progress into a cavity in which of the
following pathologic conditions:
A. Reactivation of dormant disease
B. Reinfection
C. * Erosion into the bronchiole (drainage)
D. Lobar exudate consolidation
E. Caseous pneumonia
313.
Diphtheria damage of the larynx is characterized by which of the
following types of inflammation:
A. Serous
B. Catarrhal
C. * Fibrinous
D. Granulomatous
E. Hemorrhagic
314.
The type of necrosis that can be found in tuberculous granuloma is
which of the following:
A. Coagulation necrosis
B. Liquefactive necrosis
C. * Caseous necrosis
D. Enzymatic fat necrosis
E. Fibrinoid necrosis
315.
The liver may be affected in which of the following forms of
tuberculosis:
A. Secondary tuberculosis
B. Cavitary fibrocaseous tuberculosis
C. Dormant tuberculosis
D. Primary tuberculosis complex
E. * Miliary tuberculosis
316.
On gross examination, measles pneumonia is characterized by which of
the following:
A. Cavities
B. Foci of necrosis
C. Microbe colonies
D. * Leukocyte infiltration
E. Lymphocyte infiltration
F. Edema of the tissue
317.
On gross examination, measles pneumonia is characterized by all of the
following, EXCEPT:
A. Small grey-white foci in the lung
B. Peribronchial localization of foci
C. * Subpleural localization of foci
D. Firm consistence of inflammatory foci
E. Pus containing foci
318.
On intracutaneous injection of tuberculin in an individual previously
exposed to tubercle bacilli arise all immunopathologic processes, EXCEPT:
A. * Release of CCb, the opsonin that promotes phagocytosis of
microorganisms
B. Interaction of memory THA cells with the antigen on the surface
antigen- presenting cells
C. Activation of THA cells
D. Blast transformation of THA cells
E. Proliferation of THA cells
319.
Pathogenetic mechanisms of cell injury in infectious diseases are all of
the following, EXCEPT:
A. Direct cell damage
B. Indirect cell damage by release of toxins
C. Indirect cell damage by release of enzymes
D. Indirect cell damage by induction of killer effect
E. * Direct cell damage by reduction of host responses
320.
Pathological feature in the skin found in meningococcemia is which of
the following:
A. Erythematous rash
B. * Hemorrhagic rash
C. Furuncle
D. Impetigo
E. Koplic spots
321.
Pathological feature that can be found in the heart in diphtheria is which
of the following:
A. Fibrinous pericarditis
B. * Toxic myocarditis
C. Bacterial endocarditis
D. Myocardial infarction
E. Heart aneurysm
322.
Peribronchial pneumonia in measles often results in which of the
following complications:
A. Peripheral lung tumor
B. Central lung tumor
C. Hemosiderosis of the lung
D. * Bronchiectasis
E. Lung gangrene
323.
Primary complex in tuberculosis may directly transform in all of the
following pathologic conditions, EXCEPT:
A. Fibrocalcific scars
B. Latent pulmonary lesions
C. * Miliary tuberculosis
D. Latent extrapulmonary lesions
E. Progressive primary tuberculosis
324.
Prolonged bronchiectasis may result in which of the following
derangements of protein metabolism:
A. Primary amyloidosis
B. Systemic hyalinosis
C. * Secondary amyloidosis
D. Mucoid changes
E. Fibrinoid changes
325.
Secondary tuberculosis is characterized by all of the following features,
EXCEPT:
A. Caseous necrosis and cavities in the lung
B. * Primary focus in the lung
C. Reinfection with Mycobacteria
D. Reactivation of dormant disease
E. Progression directly into the disseminated disease
326.
Severe destruction of vertebrae in spine tuberculosis may result in all of
the following pathologic conditions, EXCEPT:
A. Permanent compression fractures
B. Scoliotic deformities
C. * Drainage tract (sequestrum) forming
D. Kyphotic deformities
E. Neurologic deficits
327.
Substances that prevent complete phagocytosis of Mycobacterium
tuberculosis by macrophages and induce delayed type hypersensitivity are all
of the following, EXCEPT:
A. Cord factor
B. * Interleukin
C. Lipoarabinomanan (LAM)
D. Heart-shock protein
E. Activated complement
328.
The cause of measles development is which of the following:
A. RNA virus of the retrovirus family
B. * RNA virus of the paramyxovirus family
C. DNA virus of the picornavirus family
D. DNA virus of the adenovirus family
E. RNA virus of the togavirus family
329.
The cause of poliomyelitis development is which of the following:
A. RNA virus of the retrovirus family
B. RNA virus of the paramyxovirus family
C. * DNA virus of the picornavirus family
D. DNA virus of the adenovirus family
E. RNA virus of the togavirus family
330.
The causes of death in meningococcemia include all of the following,
EXCEPT:
A. Acute adrenal insufficiency
B. Bacterial shock
C. Acute tubular necrosis
D. Acute heart failure
E. * Cerebral cachexia
331.
The cavity in cavitary fibrocaseous tuberculosis is characterized by all of
the following features, EXCEPT:
A. Localized in the apex of the lung
B. Lined by yellow-grey caseous material
C. Walled by fibrous tissue
D. Drained by bronchus
E. * Filled with suppurative exudate
332.
The cells playing the most important role in chronic tuberculous
inflammation are which of the following:
A. * Macrophages
B. Leukocytes
C. Eosinophils
D. Erythrocytes
E. Plasma cells
333.
The complications of scarlet fever include all of the following, EXCEPT:
A. Poststreptococcal glomeru-lonephritis
B. Retropharyngeal abscess
C. Purulent mastoiditis
D. * Acute poststreptococcal hepatitis
E. Phlegmon of the neck
334.
The disease causing predominant derangement of the upper airways is
which of the following:
A. Bacterial pneumonia
B. * Diphtheria
C. Meningococcal nasopharyngitis
D. Cholera
E. Tuberculosis
335.
The disease characterized by the loss of motor neurons and flaccid
paralysis with muscle wasting and hyporeflexia is which of the following:
A. Intracerebral hemorrhage
B. Graves disease
C. * Poliomyelitis
D. Meningoencephalitis
E. Diabetes mellitus
336.
The electron micrograph of epithelial cells infected by influenza virus is
characterized by all of the following, EXCEPT:
A. Presense of influenza virus particles
B. Hyperchromatic and enlarged nuclei
C. * Displasia of epithelial cells
D. Dilation of cysterns of endoplasmic reticulum
E. Large vacuoles
337.
The granuloma in tuberculosis is composed predominantly of which of
the following cells:
A. Fibroblasts
B. * Epithelioid cells
C. Eosinophils
D. Plasma cells
E. Neutrophils
338.
The infectious agent causing meningococcal infection is which of the
following:
A. Streptococcus pyogenes
B. Staphylococcus aureus
C. Corynebacterium
D. Viruses
E. * Dyplococcus
339.
The intestine may be affected in which of the following forms of
tuberculosis:
A. Secondary tuberculosis
B. Cavitary fibrocaseous tuberculosis
C. Dormant tuberculosis
D. * Primary tuberculosis complex
E. Miliary tuberculosis
340.
The measles rash results from which of the following:
A. Nonimmune inflammation
B. Delayed — type hypersensitivity
C. Systemic immune complex reaction
D. * T-cell — mediated immunity
E. Antibody-mediated cellular dysfunction
341.
The meningococcal infection includes all of the following forms,
EXCEPT:
A. Nasopharyngitis
B. Pyogenic meningitis
C. Meningococcemia
D. * Pyogenic encephalitis
E. Chronic meningitis
342.
The miliary lung tuberculosis is characterized by which type of
inflammation:
A. * Granulomatous
B. Serous
C. Fibrinous
D. Suppurative
E. Hemorrhagic
343.
The most common cause of death in patients with diphtheria is which of
the following:
A. * Acute heart failure
B. Chronic heart failure
C. Chronic lung failure
D. Acute tubular necrosis
E. Intracerebral hemorrhage
344.
The most common sites of skeletal tuberculosis involvement are all of
the following, EXCEPT:
A. * Skull bones
B. Thoracic vertebrae
C. Lumbar vertebrae
D. Knees
E. Hips
345.
The pathologic change seen in the spinal cord in poliomyelitis is which
of the following:
A. * Neuronophagia of the anterior horns motor neurons
B. Large hematoma
C. Hemosiderosis of the anterior horns motor neurons
D. Lipofuscinosis of the posterior horns motor neurons
E. Amyloidosis of the posterior horns motor neurons
346.
The pathologic changes seen in the spinal cord in poliomyelitis are all of
the following, EXCEPT:
A. Mononuclear cell perivascular cuffs
B. Neuronophagia of the anterior horns motor neurons
C. Inflammation extension into the posterior horns
D. * Abscess formation
E. Cavitation
347.
The possible causes of chronicity of inflammation in tuberculosis are all
of the following, EXCEPT:
A.
B.
C.
D.
E.
Persistence of certain microorganisms
Prolonged exposure to toxic agents
Incomplete phagocytosis
* Complete phagocytosis
Resistance of etiologic agent
348.
The severe form of measles pneumonia is characterized by which of the
following:
A. Foci of hemorrhagic inflammation
B. Catarrhal bronchitis
C. * Destructive panbronchitis
D. Caseous necrosis
E. Granulomatous inflammation
349.
The severe form of measles pneumonia is characterized by all of the
following, EXCEPT:
A. Destructive panbronchitis
B. Interstitial mononuclear infiltration
C. Foci of suppurative inflammation
D. Measles giant cells in the alveoli
E. * Catarrhal bronchitis
350.
The spine tuberculosis breaking through intervertebral discs and
extending into the soft tissue with cold abscess forming is also referred to as:
A. Paget's disease
B. * Pott's disease
C. Gohn complex
D. Reinfection focus
E. Dormant disease
351.
The stain used to identify the causative organism in pyogenic meningitis
is which of the following:
A. Hematoxylin and eosin
B. PAS-reaction
C. * Gram
D. Congo red
E. Weigert's
352.
The term "Koplik spots" referes to which of the following:
A. Ulcerated gastric mucosal lesions
B. Foci of granulomatous inflammation of the oral cavity
C. Foci of purulent inflammation in the skin
D. Ulcerated esophageal mucosal lesions
E. * Ulcerated mucosal lesions in the oral cavity
353.
The type of immune response initiated in tuberculosis is which of the
following:
A. Direct cell cytotoxicity reaction mediated by CDD T-cells
B. * Delayed-type hypersensitivity reaction mediated by CDD T- cells
C. Complement-dependent reaction
D. Antibody-mediated cellular dysfunction
E. Antibody-dependent cell mediated cytotoxicity
354.
Tuberculoma is macroscopically characterized by all of the following
features, EXCEPT:
A. Intraparenchymal single mass
B. Greyish-white appearance
C. Well-circumscribed mass
D. Several centimeters in diameter
E. * Several millimeters in diameter
355.
Tuberculous salpingitis can be found in which of the following forms of
tuberculosis:
A. Secondary tuberculosis
B. Cavitary fibrocaseous tuberculosis
C. Dormant lesions
D. * Miliary tuberculosis
E. Primary tuberculosis
356.
The common morphologic features seen in the skin vessels in epidemic
typhus fever are all of the following, EXCEPT:
A. Endothelial cell proliferation
B. Thrombi
C. Hemorrhages
D. * Caseous necrosis
E. Vasculitis
357.
Typical tuberculous granuloma is characterized by all of the following,
EXCEPT:
A. * Plasma cells
B. Area of central necrosis
C. Epithelioid cells
D. Langhans-type giant cells
E. Lymphocytes
358.
Ulcerated mucosal lesions in the oral cavity near the opening of Stensen
ducts are referred to as:
A. * Koplik spots
B. Luschka spots
C. Burkitt spots
D. Paget spots
E. Vegetations
359.
Viruses can penetrate into the cell cytoplasm through all of the following
mechanisms, EXCEPT:
A. Translocation of the virus across the plasma membrane
B. Fusion of the viral envelope with the cell membrane
C. Receptor-mediated endocytosis of the virus
D. Fusion with endosomal membranes
E. * Virus apoptosis
360.
According to the topography of the process in the stomach distinguish
chronic gastritis:
A. pyloric;
B. small gastric curvature;
C. large gastric curvature
D. piloro-antral
E. * pangastritis
361.
According to the topography of the process in the stomach distinguish
chronic gastritis:
A. gastritis large curvature;
B. small gastric curvature;
C. pyloric
D. cardial
E. * fundal part;
362.
According to the topography of the process in the stomach with chronic
gastritis is:
A. small gastric curvature;
B. gastritis large curvature;
C. pyloric.
D. cardial
E. * antral;
363.
Acute appendicitis characterized by next morphological form as:
A. alterative;
B. dystrophic;
C. productive;
D. atrophic
E. * destructive
364.
Acute appendicitis characterized by next morphological form as:
A. deep
B. frontal;
C. complex;
D. false
E. * superficial.
365.
Acute appendicitis characterized by next morphological form as:
A. dystrophic;
B. complex;
C. combined
D. complex
E. * simple;
366.
According to clinic-morphological classification Acute virus hepatitis
can be:
A. fibrinous and necrotic
B. alterative and fibrinous
C. interstitial and fibrinous
D. complex
E. * cyclic and necrotic
367.
Appendicitis is caused by:
A. exogenous infection;
B. chemical factors;
C. physical factors.
D. radiation
E. * activated enterogenic autoinfection;
368.
They are distinguish aggressive chronic hepatitis, in which prevails:
A. fibrosis hepatocytes
B. hyalinosis hepatocytes
C. sclerosis hepatocytes
D. total fatty dystrophy
E. * dystrophy and necrosis of hepatocytes
369.
They are distinguish chronic cholestatic hepatitis, characterized by:
A. hyalinosis hepatocytes;
B. sclerosis hepatocytes;
C. fibrosis hepatocytes
D. "Muscat liver"
E. * cholestasis, cholangitis and cholangiolitis
370.
They are distinguish chronic cholestatic hepatitis, characterized by:
A. hyalinosis hepatocytes
B. sclerosis hepatocytes
C. fibrosis hepatocytes
D. disintegration of erythrocytes in vessels segments
E. * cholestasis, cholangitis and holanhiolitom
371.
They are distinguish chronic persistent hepatitis, in which prevails:
A. sclerosis hepatocytes
B. hyalinosis hepatocytes
C. fibrosis hepatocytes
D. necrosis of hepatocytes
E. * cell infiltration of portal zones and intralobulus stroma
372.
What artificial particles are differed at cirrhosis:
A. largest
B. placed
C. hyperchromic
D. presence of fat droplets
E. * broken angioarchitectonics
373.
As a result of chronic alcoholic hepatitis, disorders of nutrition and
metabolism occurs:
A. necrotic cirrhosis
B. atrophic cirrhosis
C. biliary cirrhosis
D. posttoxic cirrhosis
E. * portal cirrhosis
374.
Autoimmune gastritis is characterized by the presence of antibodies to
parietal cells and is damaged:
A. antral stomach
B. pyloric stomach
C. piloro-antral stomach
D. gastric body
E. fundal portion
375.
Autoimmunization at viral hepatitis associated with:
A. specific liver tromboplast
B. liver specific albumin
C. specific liver trombogen
D. basis of virus protein
E. * liver specific lipoprotein
376.
Bottom of acute ulcer is gray-black color due to impurities:
A. hemosiderin;
B. hemomelanin;
C. hematoidin
D. hemoporfiryn
E. * hematin hydrochloride;
377.
The destructive appendicitis includes:
A. catarrhal inflammation;
B. surface inflammation;
C. serous inflammation
D. fibrinous inflammation
E. * abscess;
378.
Chronic hepatitis is characterized by:
A. destruction of stroma, parenchyma cell infiltration, sclerosis
B. destruction, stromal cell infiltration, altered regeneration
C. cell infiltration of parenchyma & stroma, sclerosis
D. collapse and necrosis of hepatocytes
E. * destruction of parenchyma, stromal cell infiltration, sclerosis and
altered regeneration
379.
Combined ulcer complications are characterized:
A. sequential multiple complications
B. presence of only three complications in patients
C. presence of only three more complications, which change each other
D. change rapidly progressive stages of disease
E. * combination of several variants of complications
380.
According to particular morphological changes in gastric mucosa there is
next form of acute gastritis:
A. cirrhotic
B. fibrotic
C. adenomatous
D. hyperplastic
E. * fibrinous
381.
Have many time does the toxic liver dystrophy continue?
A. 1 year
B. 2 months
C. 5 months
D. about 1 week
E. * about three weeks
382.
During the first days at the toxic liver dystrophy characterized by fatty
dystrophy of hepatocytes of ...
A. periphery
B. around vessels
C. around ducts
D. total dystrophy
E. * in center lobular area
383.
During the first days in toxic liver dystrophy characterized by fatty
dystrophy of hepatocytes in the center lobular area, then it quickly change into:
A. sclerosis
B. hyalinosis
C. fibrosis
D. cirrhosis
E. * necrosis and autolysis collapse
384.
Fecal mass in patients with bleeding from peptic ulcer is characterized
by specific color and consistency. They have name ...
A. coffee grounds
B. raspberry jelly;
C. metrorrhahia
D. epistaxis
E. * melena
385.
Some chronic gastritis characterized by the presence of antibodies to
gastric parietal cells in blood. This is ...
A. reflux gastritis
B. focal acute gastritis
C. Helicobacter-associated chronic gastritis
D. acute diffuse gastritis
E. * autoimmune chronic gastritis
386.
Formation of false particles in liver cirrhosis leads to:
A. necrosis of hepatocytes
B. sclerosis hepatocytes
C. communities hemosyderosis
D. ischemic foci
E. * disturbed circulation in the liver
387.
Gross mucous resembles Crohn's disease:
A. island;
B. track;
C. conical cavity
D. Multiple cavity like plates
E. * pavement;
388.
In the course of toxic liver dystrophy distinguish stage:
A. Yellow hypertrophy
B. Yellow hypothrophy
C. Yellow gonadal
D. amiloidosis
E. * yellow atrophy
389.
In the course of toxic liver dystrophy distinguish stage:
A. Yellow hypertrophy
B. Yellow hypothrophy
C. Yellow gonadal
D. yellow necrosis
E. * yellow atrophy
390.
In the course of toxic liver dystrophy distinguish stage:
A.
B.
C.
D.
E.
391.
392.
393.
394.
395.
396.
397.
red aplasia
red anaplasia
red agenesia
red infarct
* red atrophy
In the course of toxic liver dystrophy distinguish stage:
A. cicatrization
B. restitution
C. substitution
D. vascularization
E. * restoration
In the red stage dystrophy in toxic liver dystrophy becomes:
A. gray
B. muscat
C. fat
D. green
E. * yellow with a red spots
In the red stage dystrophy in toxic liver dystrophy often develops:
A. renal failure
B. heart failure
C. cholangiocellular failure
D. DIC syndrome
E. * liver failure
In ulcer-destructive ulcer complications include:
A. stenosis;
B. tetania;
C. chlorhidropenia
D. malignization
E. * bleeding;
In viral hepatitis liver cells damaged by the mechanism:
A. apoptosis
B. response alternatives
C. anaphylactic reaction
D. hypoxia
E. * slow-type hypersensitivity
Inflammation, which extended into all stomach layers is called:
A. Massive gastritis
B. diffuse gastritis
C. widespread gastritis
D. total gastritis
E. * pangastritis
Liver in toxic dystrophy becomes less feeble, yellow comes in stages:
A. red atrophy
B. Sclerosis
C. fibrosis
D. necrotic foci
E. * yellow atrophy
398.
Overgrowth of connective tissue on the course portal tracts in the form
of sprouts, the penetration of particles in the liver, dividing them into small
artificial characteristic:
A. necrotic cirrhosis
B. toxic cirrhosis
C. biliary cirrhosis
D. focal hepatic
E. * portal cirrhosis
399.
Precancerous stomach are:
A. sharp erosion surface
B. ulcer
C. acute catarrhal gastritis
D. Chronic hypertrophic gastritis
E. * chronic atrophic gastritis
400.
Precancerous stomach are:
A. sharp erosion surface
B. ulcer
C. acute catarrhal gastritis
D. carcinomatosis
E. * chronic gastric ulcer
401.
Precancerous stomach are:
A. sharp erosion surface
B. ulcer
C. acute catarrhal gastritis
D. intestinal metaplasia
E. * severe dysplasia of gastric mucosal epithelium.
402.
Primary biliary cirrhosis is the result:
A. calculous cholecystitis
B. abscessed cholecystitis
C. purulent hepatitis hranulematoznoho
D. toxic liver dystrophy
E. * aseptic destructive (necrotizing), cholangitis and cholangiolitis
403.
Primary hepatitis occurs following:
A. bacteria
B. mushrooms
C. simpler
D. Streptococci
E. * Hepatotropic viruses
404.
Primary hepatitis occurs following:
A. bacteria
B. mushrooms
C. simpler
D. staphylococci
E. * Alcohol
405.
Primary hepatitis occurs following:
A. bacteria
B. mushrooms
406.
407.
408.
409.
410.
411.
412.
413.
C. simpler
D. Streptococci
E. * drugs
Primary hepatitis occurs following:
A. bacteria
B. mushrooms
C. simpler
D. staphylococci
E. * stagnation of bile
Secondary biliary cirrhosis caused by:
A. hepatitis
B. alcoholic hepatitis
C. active hepatitis
D. excess copper in the blood
E. * cholestasis due to extrahepatic biliary tract obstruction (stone, tumor)
Secondary biliary cirrhosis caused by:
A. hepatitis
B. alcoholic hepatitis
C. active hepatitis
D. chronic lead intoxication
E. * biliary tract infection (cholangiolitic cirrhosis)
The basis of cirrhosis is:
A. dystrophy portal tracts
B. necrosis of bile duct epithelium
C. necrosis and regeneration of pathological bile duct epithelium
D. hepatic vascular hyalinosis
E. * dystrophy and necrosis of hepatocytes
The death of hepatocytes in cirrhosis leads to:
A. Pathologic restitution portal tracts
B. incomplete substitution of hepatocytes
C. hyperplasia and hypertrophy of hepatic bile ducts
D. liver infarctions
E. * enhanced regeneration of stored parenchyma
The liver in primary biliary cirrhosis:
A. not increased red
B. increased brown-red
C. increased yellow
D. gray-red, reduced
E. * increased in terms of gray-green
The liver in secondary biliary cirrhosis:
A. brown due to hemosiderin
B. red due to diapedesis of erythrocytes
C. yellow color due to fat accumulation in hepatocytes
D. gray with a red spots
E. * green due to bile soaking
The signs of aggravation of peptic ulcer is the appearance:
A. hyalinosic ulcer
B.
C.
D.
E.
sclerosis during ulcer;
amyloid cells in the edges of ulcers
metaplasia mucosa at the edges of ulcers
* fibrynoid changes of vascular walls ulcer;
414.
Violation of circulation in liver cirrhosis is increasing hypoxia, which
leads to:
A. necrobiotic processes in hepatocytes
B. portal hypertension
C. local hypertrophy
D. venous plethora
E. * dystrophy and death of hepatocytes, regenerate the nodes, enhanced
overgrowth of connective tissue
415.
Viral hepatitis - a disease caused by:
A. hepatogenic viruses
B. tixotropic viruses
C. hepatocellular viruses
D. AIDS virus
E. * hepatotropic viruses
416.
When ulcer healing source of stomach forms in its place:
A. surface erosion
B. severe erosion, resulting substitution
C. partial epithelization, as a consequence of substitution
D. focal metaplasia
E. * rough scar, which often leads to stenosis of pylorus
417.
A study area with fibrotic inflamed pulp, revealed that cellular
infiltration consisting of:
A. erythrocytes, lymphocytes, monocytes
B. eosinophil, mast cells, neutrophils
C. lymphocytes, mast cells, macrophages
D. * lymphocytes, macrophages, plasma cells
E. lymphocyte, eosinophil, plasma cell
418.
Among age groups are usually observed the development of cement
caries?
A. in boys
B. girls
C. in pregnant women
D. * in older people
E. equally in the groups listed
419.
Among the types of dental stone are:
A. fisuric and contact
B. acute and chronic
C. local and distributed
D. surface and deep
E. * supragingival and subgingival
420.
At what degree of fluorosis dentine begins to collapse?
A. 1st
B. 2nd
C. * 3rd
D. 4th
E. 5th
421.
At what stage of caries begins to form a cavity in the tooth?
A. surface caries
B. stage spot
C. stage progression
D. * average caries
E. deep caries
422.
Caries development stage begins ...
A. stage of enamel demineralization
B. surface caries
C. chemical changes
D. * stage spot
E. stage alteration
423.
Cement caries usually occurs in ...
A. boys
B. girls
C. * seniors
D. pregnant women
E. equally in all groups listed
424.
Chalk pigmentation spots associated with the accumulation of tyrosine,
which becomes:
A. triiodthyronine
B. thyreotropin
C. Adenosine-3-phosphate
D. * melanin
E. hemosiderin
425.
Chronic pulpitis has the following form ...
A. serous
B. fibrinous
C. abscessed
D. * hranulyuyuchyy
E. Progressive
426.
Developing fluorosis in areas where fluoride content in water and food
than
A. 0. 7 mg / l
B. 1 mg / l
C. 1. 2 mg / l
D. * 2 mg / l
E. 4 mg / l
427.
Development of a typical medium caries is within ...
A. 2 / 3 of the surface area of enameling
B. enamel and cement
C. * dentin
D. dental pulp
E. half crown
428.
Development of caries in occlusal surface explains ...
A. Bacterial theory
B. acid theory
C. metabolitna theory
D. * proteolytic theory
E. hormonal theory
429.
Disease process in which the demineralization and destruction of hard
tooth tissues with formation of a defect - is:
A. * caries
B. hyperftorosis
C. tooth erosion
D. pulpitis
E. periodontitis
430.
Dispersal of cement when it caries entitled:
A. hipertsementoz
B. cementoma
C. * cementolisis
D. cementoblastoma
E. all of these terms are synonymous
431.
Favorable background for dental carious lesions are ...
A. hypersecretion of saliva
B. use hard toothbrushes
C. regular use of chewing gum
D. * sexual maturation
E. nothing from the above
432.
Favorable background of wedge defects of the tooth is ...
A. salivary gland disease
B. Staff bone disease
C. oral disease
D. * Gastrointestinal Tract
E. nothing among the above
433.
Fluorosis - is ...
A. * endemic
B. occupational illness
C. iatrogenic disease
D. metabolic diseases
E. hormonal diseases
434.
Fluorosis 3 degrees is called ...
A. poor lesion
B. * moderately expressed affection
C. heavy damage
D. stage complications
E. in the development of fluorosis no stadiynosti
435.
Fluorosis a first degree is called ...
A. poor lesion
B. * very weak damage
C. moderately expressed affection
436.
437.
438.
439.
440.
441.
442.
443.
D. heavy damage
E. stage complications
Fluorosis in its development is ...
A. 5 degrees
B. * 4 degrees
C. 3 degrees
D. 2 degrees
E. No degrees of separation
Fluorosis two first degree is called ...
A. * poor lesion
B. very weak damage
C. moderately expressed affection
D. heavy damage
E. stage complications
Formation of irregular carious dentin during cavity - a manifestation:
A. metaplasia
B. compensatory hypertrophy
C. physiological regeneration
D. * reparative regeneration
E. malignization
Frequently localization of caries?
A. Bicuspid
B. * Molar upper jaw
C. Cutters
D. Fangs
E. Mandibular molar
Frequently localization parodontosis:
A. In the incisors
B. In the canines
C. In the bicuspid
D. In the upper jaw molars
E. * In the incisors and canines
How is the secondary dentine of the primary?
A. * degree of mineralization and structural structure
B. location
C. absence of cellular elements
D. no different
E. no such structure in the teeth
How many stages of development of caries you know?
A. 2 stage
B. 3 stages
C. * 4 stage
D. 5 stage
E. its development has not stages
In the majority of tooth decay begins ...
A. bicuspid
B. * molars of the upper jaw
444.
445.
446.
447.
448.
449.
450.
451.
C. cutter
D. iklah
E. mandibular molars
In what is developing within typical average caries?
A. 2 / 3 of the surface area of enameling
B. pathological process extends to the enamel and cement
C. * process passes to dentine
D. is in the process of destruction of the tooth pulp
E. destroyed half crown
In which areas are placed interstitial dentykle?
A. pulp
B. on the border between dentin and enamel of the tooth
C. * middle weight dentin
D. near the tooth root
E. among sponge trabecula jaw
In which parts of a tooth caries development is the fastest?
A. * in dentin
B. in pulp
C. in cement
D. in enamel
E. no data
Most tooth decay affects:
A. fangs
B. Cutters
C. * first big teeth
D. 2nd tricuspid teeth
E. small teeth
Name the morphological features of dentin regeneration?
A. * randomly placed dentyn tubules
B. high content of fluorine and calcium in its structure
C. lack of it prisms
D. high content of loose connective tissue in its structures
E. broken relationship between cells and intercellular substance
Name the factors of fluorosis ...
A. genetic metabolic
B. Professional harmful effects on the body
C. violation of mineral metabolism
D. insufficient flow of fluoride in the body
E. * increased arrivals of fluoride in the body
Name the place where are most often dentykles situate.
A. molars of deciduous teeth
B. provisional bicuspid teeth
C. canines of the upper jaw
D. Permanent bicuspid teeth
E. * molars of permanent teeth
Uncariose damages of hard tooth tissues include ...
A. periodontitis
B.
C.
D.
E.
pulpitis
gingivitis
* enamel hypoplasia
hypertsementosis
452.
Nomenclature name soft tissue of the tooth A. stroma
B. * pulp
C. dentine
D. spongy bone tissue
E. keloid
453.
Often causes the development of flux:
A. * Acute apical periodontitis
B. Gangrenous pulpitis
C. Fibrous pulpitis
D. Catarrhal gingivitis
E. Periodontal disease
454.
One of the factors forming the transparent dentin is an excessive delay in
dentin channels
A. fluoride salts
B. acids
C. * calcium salts
D. phosphorus salts
E. microorganisms
455.
One of the main etiofactor of caries development is ...
A. * bacteria
B. Viruses
C. simple
D. chemical damage to teeth
E. physical damage to teeth
456.
One of the most common causes of periodontitis are:
A. * gingivitis
B. fluorosis
C. acute pulpitis
D. chronic pulpitis
E. caries
457.
One option caries, which is characterized by local dissolution of enamel
is ...
A. circular caries
B. early caries
C. lateral tooth decay
D. retrograde caries
E. * stationary caries
458.
Periodontal disease - a chronic periodontal disease:
A. Inflammatory nature
B. Alternatives character
C. Tumor disease
D. * Primary-dystrophic diseases
459.
460.
461.
462.
463.
464.
465.
466.
E. Secondary-dystrophic diseases
Periodontitis often affects ...
A. upper incisors
B. lower incisors
C. bicuspid
D. * lower molars
E. upper molars
Predecessor plaque on the teeth ...
A. * Dental plaque
B. pelikule
C. cuticle
D. tartar
E. bacteria
Primary cause of tooth erosion are ...
A. mechanical damage to the enamel chewing
B. thermal damage to the enamel
C. changes the chemical composition of saliva
D. professional Damage
E. * disease is kryptogenuc
Reactive changes include pulp
A. * atrophy, dystrophy, necrosis
B. necrosis, caries, periodontitis
C. dentykli, atrophy, pulpitis
D. atrophy, dystrophy, caries
E. pulpitis, periodontitis, caries
Resorption of bone tissue disease caused by cells in periodontitis:
A. osteoblasts
B. osteocyte
C. * osteoclast
D. plasma
E. macrophages
Synonymous with the diagnosis "subenamelum caries" are:
A. circular caries
B. * early caries
C. lateral tooth decay
D. stationary caries
E. retrograde caries
Tell the tooth surface which has the greatest resistance against caries
A. contact distal
B. proximal contact
C. buccal
D. * lingual
E. chewing
The concept of early caries means ...
A. caries development in milk teeth
B. caries development shortly after cuttings permanent teeth
C. shortly after the development of caries teeth cuttings
467.
468.
469.
470.
471.
472.
473.
474.
D. development of caries in children in July 1910 age
E. * development of caries directly under a layer of enamel
The concept of retrograde cavities means ...
A. development of caries in the back teeth
B. development of caries from the back surface of the tooth contact
C. * development of caries with dental pulp
D. development of dental caries cement
E. development of caries from the enamel layer
The development of caries on the smooth surface of the tooth explains ...
A. Bacterial theory
B. * acid theory
C. proteolytic theory
D. metabolitic theory
E. hormonal theory
The development of fluorosis caused by excessive amount in the body ...
A. calcium
B. phosphorus
C. * fluorine
D. boron
E. iron
The development of fluorosis share on ...
A. 2 degrees
B. 3 degrees
C. * 4 degrees
D. 5 degrees
E. No degrees of separation
The earliest stage of development of caries are:
A. surface caries
B. * Stage Spot
C. stage progression
D. secondary caries
E. deep caries
The first stage of caries - a ...
A. surface caries
B. prekariyes
C. initial caries
D. fisurnyy caries
E. * Stage Spot
The most common cause of periodontitis are:
A. caries
B. * gingivitis
C. acute pulpitis
D. chronic pulpitis
E. fluorosis
The presence of excessive number of cavities in parotynum ...
A. serves as a catalyst
B. accelerated destruction of hyperactivity in microbial hialuronidazy
C. accelerates fracture between teeth changes
D. * slows the destruction of tooth
E. not affect the course of caries
475.
The reason most often speaks of flux ...
A. * acute apical periodontitis
B. gangrenous pulpitis
C. fibrotic pulpitis
D. catarrhal gingivitis
E. parodontosis
476.
Through many stages of development is caries?
A. 2
B. 3
C. * 4
D. 5
E. its development has stages
477.
Transparent dentine formed as a result of excessive delay in dentynnyh
channels
A. microorganisms
B. acids
C. * calcium salts
D. fluoride salts
E. phosphorus salts
478.
Trophic damaged dentin and organic matter on soil development cause
disease alimentary canal
A. fluorosis
B. * wedge-shaped defects of hard tissue
C. erosion of teeth
D. acid hard tissue necrosis
E. pulpitis
479.
What characterized retrograde cavities?
A. distribution of root caries with cement on the neck and enamel crown
B. * carious damage passes from the enamel dentine
C. carious tooth root damage only
D. inflammation occurs in the root canal
E. only damaged tooth pulp
480.
What characterized the stationary tooth decay?
A. disease progresses rapidly to stay in a stationary office
B. caries developed as iatrogenic disease etiology
C. * carious damage is progressing
D. damage only large molar teeth
E. no such cavities
481.
What does dentycle formed in tooth?
A. pulp
B. primary dentin
C. * secondary dentin
D. peryodontium
E. rudimentary elements of permanent teeth
482.
483.
484.
485.
486.
487.
488.
489.
What is the criterion of secondary dentin differences from the original?
A. * degree of mineralization and structural building
B. location
C. presence or absence of cellular elements
D. no different
E. no such structure in the teeth
What is the etiological factor of caries?
A. Viruses
B. simple
C. * bacteria
D. chemical damage to teeth
E. physical damage to teeth
What is the first manifestation of caries?
A. appearance of black spots on tooth enamel
B. appearance of brown stains on tooth enamel
C. * appearance of white opaque spots on tooth enamel
D. appearance of radial cracks oriented
E. appearance of small black enamel erosion
What is the initial phase of wedge defects in tooth?
A. traumatic damage to the chewing
B. * trophic disorders of organic matter in enamel and dentin
C. Acidic chemical influences
D. alkaline chemical attack
E. significant temperature differences
What is the precursor to plaque?
A. pelikula
B. * Dental plaque
C. cuticle
D. tartar
E. bacteria
What regenerative dentine morphologically different from the typical?
A. prevailing lack of prisms
B. high levels of fluoride and calcium in the structures
C. * chaotic arrangement of dentine tubules
D. high content of loose connective tissue structures
E. abnormal ratio between cells and intercellular substance
What special stationary development of caries?
A. develops after medical interventions on teeth
B. develops when patients stay in hospital inpatient
C. * localized only in enamel and progresses
D. developing in the rudiments of permanent teeth
E. no such option caries
When excess in the body develops a chemical element fluorosis?
A. phosphorus
B. boron
C. iron
D. * fluorine
490.
491.
492.
493.
494.
495.
496.
497.
E. calcium
Which teeth are most often found dentykli?
A. molars of deciduous teeth
B. provisional bicuspid teeth
C. canines of the upper jaw
D. * molars of permanent teeth
E. Permanent bicuspid teeth
Why developing fluorosis?
A. a genetic metabolic
B. through professional detrimental effects on the body
C. for violations of mineral metabolism
D. insufficient flow through the body fluorine
E. * through increased revenues of fluoride in the body
3 rd degree of fluorosis was named ...
A. very weak damage
B. poor lesion
C. * moderately expressed affection
D. heavy damage
E. stage complications
4-th degree of fluorosis was named ...
A. poor lesion
B. very weak damage
C. moderately expressed affection
D. * heavy damage
E. stage complications
According to etiology, acid teeth hard tissue necrosis is ...
A. infections
B. * occupational diseases
C. hereditary diseases
D. polyetiologic disease
E. idiopathic disease
Acid necrosis hard tissue of teeth in its etiology - is:
A. * occupational illness
B. infectious diseases
C. hereditary pathology
D. unexplained etiology
E. vascular disease
Acid tooth hard tissue necrosis affects categories of people ...
A. chewing tobacco abusers
B. abusers products that canned acids
C. with low pH of saliva
D. * advancement in contact with mineral acids
E. no dependence
Acute apical periodontitis is inflammation of the form:
A. Granulematous
B. * Suppurative
C. Fibrinous
D. Alternatives
E. Productive derivative
498.
Against the background of digestive diseases frequently developing
trophic damage of organic matter and dentin entitled
A. fluorosis
B. * cuneiform hard tissue defects
C. erosion of teeth
D. acid hard tissue necrosis
E. pulpitis
499.
At what stage is formed carious cavities?
A. Stage Spot
B. surface caries
C. * secondary caries
D. stage progression
E. deep caries
500.
A follicle cyst develops from:
A. * enamel organ of tooth which was not cut
B. Granulation tissue
C. Fibrotic tissue
D. Inflammatory infitration
E. From the wall of chronic abscess
501.
A follicle cyst more frequently is combined:
A. with the first premolar
B. * with the second premolar
C. with the first molar
D. with the second molar
E. with the lower incisiva-tooth
502.
A follicle cyst more frequently is combined:
A. with the first premolar
B. * with the third molar
C. with the first molar
D. with the second molar
E. with the lower incisiva-tooth
503.
A follicle cyst more frequently is combined:
A. with the first premolar
B. * with the lower canina-tooth
C. with the first molar
D. with the second molar
E. with the lower incisiva-tooth
504.
A malignant lymphadenoma with typical localization in jaws bones is:
A. a Abrikosov's tumor
B. * a Bercit's tumor
C. a Willms' tumor
D. a Sezary tumor
E. a Khodjkin's tumor
505.
According to the clinic-morphological features the ameloblastomas are:
A. * benign tumor
B.
C.
D.
E.
malignanted benign tumor
malignant tumor
a tumor with local destructive growth
a tumular process
506.
Accrding to etiology giant cementoma is a:
A. result of trauma
B. result of specific inflammatory process
C. * inherited disease
D. complication of visceral mycosis
E. manifestation of chronic radiation illness
507.
Adenomatic tumor arises up more often:
A. intra-uterine
B. at infants and children of the first year-old
C. * in the second decade-period of life
D. in the people of mature age
E. in the old people
508.
At a parathyreoid osteodystrophy in the maxillufacial bones to the
skeleton is observed:
A. * lacunar resorbtion and formation of fibrotic tissue
B. specific inflammatory process
C. heterospecific inflammatory process
D. sequestra formation
E. hyperostosis
509.
At a paratireoid osteodystrophy in the bones of maxillufacial skeleton is
observed:
A. * lacunar resorbtion and formation of fibrotic tissue
B. specific inflammatory process
C. unspecific (heterospecific) inflammatory process
D. sequestr formation
E. hyperostosis
510.
At acuteinig of inflammation in the wall of radicular cyst develops:
A. Abscess
B. Granulosum
C. Necrosis
D. * Acanthosis of covering epithelium
E. dysplasia of epithelium
511.
At histological research of dentinoma it is possible to discover:
A. trabeculs of inactive odontogenic epithelium
B. immature connecting tissue
C. the islets of dysplastic dentine
D. * all structures that are marked
E. nothing from enumerated
512.
At inflammation the radicular cyst cavity can be filling:
A. * By granulation tissue
B. By fibrotic tissue
C. by fibrinosic exsudate
D. By necrosis
E. By a festering exsudate
513.
At the congenital form of uncompleted osteogenesis in bones of
maxillufacial skeleton is observed:
A. * normal amount of osteocytes and deficiti of basic matter in bones
trabecules
B. the deficit of osteocytes is expressed
C. specific inflammatory process
D. aseptic necrosis of bone tissue
E. substituting of bone tissue by a cartilage
514.
At the histological research of jaw periosteum thay found inflammatory
hyperemia, swollen, moderate infiltration by neutrophilic leucocytes.
Inflammation developed after a trauma. What process is it?
A. * Serosic periostitis
B. Festering periostitis
C. Fibrotic periostitis
D. osteomielitis
E. Secondary amyloidosis
515.
Character of inflammation acute periostitis is:
A. fibrotic
B. * serosal, festering
C. alterativ, productive
D. destructive, hyperplastic
E. primary, recidive
516.
For a adenomatoid tumor typically is:
A. * a odontogenic epithelium forms structures which remind the channels
B. atypizm and polymorphism of odontogenic epithelium are expressed
C. epidermoid metaplasia of odontogenic epithelium
D. metastasis in the regional lymphatic nodes
E. all that are enumerated
517.
For an ameloblastoma the typically is:
A. metastasis by a haematogenic way
B. metastasis by a lymphogenic way
C. * numerous niduses of bone destruction
D. quick growth
E. more often is localized in a maxilla
518.
For what form of ameloblastoma the typically is epidermoid cell
metaplasia with keratin formation?
A. follicle
B. plexiform
C. * akantomatic
D. basalcell
E. granularcell
519.
Histological form of ameloblastoma, which meets more often than other:
A. akantomatosal
B. * follicle
C. basalcell
D. roundcell
520.
521.
522.
523.
524.
525.
526.
527.
E. granularcell
It is possible to see in the internal wall of follicle cyst:
A. atypic cells
B. osteoblasts
C. * mucus productings cells
D. Ferrous cells
E. All that are enumerated cells
Osteomielitis is the:
A. * Marrow inflamation of jaws bones
B. Festering inflammation of mucus membrane
C. Odontogenic antritis
D. Odontogenic sepsis
E. Secondary amyloidosis
Osteomielitis more frequent develops:
A. in a maxilla
B. * in a mandibula
C. frequency of defeat is identical
D. at the same time in the maxilla and mandibula
E. in a maxilla osteomielit arises up never
Periostitis is :
A. infectiously-allergic process which develops in a bone
B. festering-necrotizing process which develops in a bone
C. * an inflammatory process in a periosteum
D. inflammatory process which will strike tissues of parodont and spreads
on bones structures adjoining to him
E. a tumular process is in bone
Premordial cyst develops more often in …
A. In the area of the first molar
B. in the area of the second molar
C. * in the area of the third molar
D. in the area of the first premolar
E. in the area of the second premolar
The ameloblastic fibroma is characterized by:
A. contains the islets of proliferativ odontogenic epithelium
B. contains the fluffy tissue which reminds the tissue of dental papilla
C. typical localization is the area of premolar
D. developsmore more frequently in in child's and young age
E. * all that are enumerated
The benign odontogenic tumor of ectodermal origin is:
A. * ameloblastoma
B. cementoma
C. odontogenic carcinoma
D. osteoblastoclastoma
E. dentinoma
The benign odontogenic tumors of mesenchyma origin are:
A. ameloblastoma, adenomatic tumor
B. * dentinoma, myxoma, cementoma
C. odontogenic carcinoma, osteoblastoclastoma
D. ameloblastic fibroma, odontoameloblastoma
E. all that are enumerated
528.
The benign unodontogenic tumor of jaw is:
A. ameloblastoma
B. odontoameloblastoma
C. * osteoblastoclastoma
D. osteosarcoma
E. all are enumerated
529.
The capsule of sequestral cavity is formed by:
A. the unchanged bone tissue
B. necrotic bone tissue
C. * fibred connecting tissue
D. granular tissue
E. cartilaginous tissue
530.
The cells which form the parenchima of osteoblastoclastomas are
belonging to:
A. * osteogenic
B. odontogenic ectodermic origin
C. odontogenic mesenchimal origin
D. melaninproductors
E. ephithelial
531.
The clinic-anatomical forms of ameloblastoma is:
A. odontogenic and osteogenic
B. ectodermal and mesenchimal
C. smallnidus and diffuse
D. acute and recidivic
E. * cystophorous and solid
532.
The inflammatory diseases of jaws are:
A. * periostitis, osteomielitis
B. osteomielitis, osteoma
C. cyst,osteoblastoclastoma
D. periostitis, paratireoid osteodystrophy
E. all that are enumerated disease
533.
The internal surface of radicular cyst wall is covered:
A. By a ferrous epithelium
B. By a transitional epithelium
C. * By a multi-layered flat uncarotinized epithelium
D. By a multi-layered flat carotinized epithelium
E. By fibrotic tissue
534.
The internal surface of wall of premordial cyst is covered:
A. By mature connecting tissue
B. * By a multi-layered flat epithelium
C. By a ferrous epithelium
D. By a transitional epithelium
E. By fluffy connecting tissue
535.
The odontogenic epithelium of ameloblastoma contains the numerous
acidophilic granules. This is a:
A. malignant ameloblastoma
B. plexiform form of ameloblastoma
C. akantomatic form of ameloblastoma
D. basalcell form of ameloblastoma
E. * granularcell form of ameloblastoma
536.
The odontogenic fibroma is characterized by:
A. contains the islets of inactive odontogenic epithelium
B. contains the mature connecting tissue
C. more frequently meets in senior age-dependent groups
D. * all that are enumerated is right
E. all that are enumerated is incorrectly
537.
The pathognomic sign of cemenomas is:
A. a presence of young or mature fibrotic tissue
B. * a formation of cementoid matter with the different degree of
mineralization
C. a presence of trabeculs of inactive odontogenic epithelium
D. a presence of trabeculs of odontogenic epithelium with the signs of
polymorphism and atypizm
E. it contains the islets of dysplastic dentine
538.
The primary intracyst carcinoma can develop from:
A. the epithelium of mucus membrane of mouth cavity
B. * the epithelium of disontogenetic odontogenic cysts
C. the odontogenic mesenchyma
D. the bone tissue
E. can develop from all enumerated structures
539.
The primary intracyst carcinoma develops from:
A. the epithelium of mucus membrane of mouth cavity
B. * the epithelium of Malasse's islets
C. the odontogenic mesenchyma
D. the bone tissue
E. can develop from all enumerated structures
540.
The radicular cyst develops as a ressult of:
A. acute abscess
B. chronic abscess
C. acute periodontitis
D. * Chronic periodontitis
E. Follicle cyst
541.
The radicular cyst develops:
A. * from a complex granulomas
B. From a follicle cyst
C. From acute periodontitis
D. From an acute abscess
E. From a chronic abscess
542.
The radicular cyst meets:
A. In 20% from all cases
B.
C.
D.
E.
In 40% from all cases
In 50% from all cases
In 60% from all cases
* In 80% from all cases
543.
The typically difference of odontogenic myxoma from the myxomas of
other localization is next:
A. the base of tumor is formed by mucous mass
B. contains the a lot of acid mucopolisaccharides
C. * contains the trabeculs of inactive odontogenic epithelium
D. a tumor does not metastases
E. a tumor is predispositioned to the relapses (recedives) after the surgical
delete
544.
The wall of sequestral cavity consists of:
A. unchanged bone tissue
B. granulation tissue
C. connective tissue capsule and ephithelial layer
D. * connective tissue capsule and pyogenic membrane
E. all enumerated variants are possible
545.
What changes are present in bone tissue at acute osteomyelitis:
A. excrescence of granulation tissue
B. sclerosis
C. * resorbtion of bones trabecules
D. regeneration of bones trabecules
E. tissue atipism
546.
What form of ameloblastoma according to a histological structure
reminds an enamel organ?
A. * follicle
B. plexiform
C. akantomatosal
D. basalcell
E. granularcell
547.
What form of ameloblastoma according to a histological structure
reminds a basalcell cancer?
A. follicle
B. plexiform
C. akantomatic
D. * basalcell
E. granularcell
548.
What histological types of cementom do you known?
A. * benign cementoblastoma, cementitious fibroma, giant cementoma
B. benign cementoblastoma, dentinoma, ameloblastoma
C. benign cementoblastoma, cementitious fibroma, odontoameloblastoma
D. cementitious fibroma, giganticcells tumor
E. all that are enumerated
549.
What name of the tumor, which is introduced by the trabecular net of
odontogenic epithelium with whimsical branching?
A. a follicle form of ameloblastoma
B.
C.
D.
E.
* a plexiform form of ameloblastoma
an akantomatic form of ameloblastoma
a basalcell form of ameloblastoma
a granularcell form of ameloblastoma
550.
What pathomorphological change characterizes odontogenic
osteomyelitis most full?
A. inflammation of peridont
B. inflammation and niduses of peridont destruction
C. inflammation and destruction of peridont, festering-inflammatory
process in a periosteum
D. * festering infiltration of marrow, thrombosis of vessels, festering
melting of trombs, area of hemorrhage and osteonecrosis
E. festering-inflammatory process in a jaw and surrounding tissues
551.
What structure of teeth has the ectodermal origin?
A. dentine
B. odontoblasts
C. pulp
D. * enamel
E. cement
552.
What tumor does not belong to the odontogenic tumor s of the mixed
origin?
A. ameloblastic fibroma
B. odontoameloblastoma
C. ameloblastic fibroodontoma
D. * cementitious fibroma
E. odontogenic fibroma
553.
What tumors are belonging to odontogenic carcinomas?
A. osteoblastoclastoma and osteosarcoma
B. basalcell and acidophilic forms of ameloblastomas
C. * malignant ameloblastoma and primary intracyst carcinoma
D. ameloblastic fibroma and odontoameloblastoma
E. all that are transferred
554.
What tumors does not belong to the odontogenic tumors of mesenchymal
origin?
A. dentinoma
B. odontogenic myxoma
C. * odontoameloblastoma
D. benign cementoma
E. giant cementoma
555.
A typical diagnostic criteria of dry syndrome is autoimmune sialoadenit
and:
A. adenocarcinoma of salivary gland
B. chronic osteomyelitis of the jaw
C. obesity at the top type
D. all referred
E. * polyarthritis
556.
At which disease occurs sialoadenit with salivary gland destruction of
lymphocytes and macrophages?
A. Down syndrome
B. Patau syndrome
C. Shereshevsky-Turner syndrome
D. Syndrome Stein-Leventhal
E. * Sjogren syndrome
557.
At which disease often develops autoimmune sialoadenit:
A. Systemic lupus erythematosus
B. Systemic sclerosis
C. AIDS
D. Viscidosis
E. * Sjogren syndrome dry
558.
Causative agent of mumps are:
A. retrovirus
B. adenoidal pharyngeal conjunctival virus
C. respiratory syncytial virus
D. Mycoplasma
E. * miksovirus
559.
Chronic inflammation sialoadenit in nature tends to be:
A. alternatives
B. catarrhal
C. abscess
D. gangrenous
E. * productive
560.
Combined secondary lesions of salivary glands and tearfully known as:
A. Alport syndrome
B. Patau syndrome
C. Cider Zhilbera
D. Syndrome kartagener
E. * Syndrome of Mykulich
561.
Diseases of salivary gland origin are:
A. Genetically-determined
B. Intoksykatsiynoho origin
C. Inflammatory origin
D. Chronic
E. * Congenital
562.
Erysipelas caused lips:
A. Staphylococcus
B. Ps.aeruginosa
C. Papilloma-virus
D. With Epstein-Bar
E. * Streptococcus
563.
Frequently localization of adenolymphoma:
A. small salivary glands of oral cavity
B. sublingual gland
C. submandibular gland
564.
565.
566.
567.
568.
569.
570.
571.
D. tumor occurs about equally in all glands
E. * parotid
Frequently localization carcinoma of salivary glands:
A. sublingual gland
B. submandibular gland
C. parotid
D. tumor occurs about equally in all glands
E. * small salivary glands hard and soft palate
Frequently monomorphic adenoma localization:
A. small salivary glands of oral cavity
B. sublingual gland
C. submandibular gland
D. tumor occurs about equally in all glands
E. * parotid
Frequently pleomorfnoyi adenoma localization:
A. small salivary glands of oral cavity
B. sublingual gland
C. submandibular gland
D. tumor occurs about equally in all glands
E. * parotid
Frequently sialoadenitu primary reason:
A. Epstein-Bar virus
B. Streptococcus
C. Staphylococcus
D. Fungi of Candida
E. * Mumps
Frequently sialoadenitu primary reason:
A. Epstein-Bar virus
B. Streptococcus
C. Staphylococcus
D. Fungi of Candida
E. * Cytomegalovirus
Granulomatous heylitis is the first manifestation of the syndrome:
A. Uoterhauzena-Frederiksena
B. Kon
C. Pituitary Kushinh
D. Simonds
E. * Melkerson-Rosenthal
Hlositis - is:
A. inflammation of the lips
B. inflammation of the mucous membrane of mouth
C. salivary gland inflammation
D. inflammation of the jaw bone
E. * tongue inflammation
In the parotid glands in mumps develops
A. Unilateral purulent parotitis
B. two-sided purulent parotitis
C. cirrhosis glands
D. sialolitiaz
E. * duplex interstitial parotitis
572.
Most dangerous complication of furuncle lips:
A. Stroke
B. Lymphadenitis
C. Limphangoitis
D. Dermatitis
E. * Angular vein thrombophlebitis
573.
Name acute infectious viral disease, which is typical for inflammation in
the stroma of parotid salivary glands.
A. measles
B. rubella
C. paragripom
D. Filatov's disease
E. * parotitis
574.
Name innate disease of salivary glands.
A. sialoadenit
B. sialolitiaz
C. onkocitoma
D. all of these
E. * ektopia
575.
Pleomorfna adenoma of salivary glands are often found in:
A. children of the first year of life
B. pubertal age
C. in men under 30
D. in the elderly, regardless of the article
E. * in women after 40 years
576.
What are the main morphological structures are signs of pregnancy?
A. glandular hyperplasia edometriya
B. placental polyp
C. * chorionic villi and decidual tissue
D. Iron-cystic hyperplasia
E. Endotservikoz
577.
Termination of pregnancy and the allocation of the uterus before the
fetus from 14 weeks of conception denote as:
A. Miscarriage
B. Late abortion
C. *Early abortion
D. Premature birth
E. Criminal abortion
578.
Termination of pregnancy and the allocation of the fetus from the
uterus of 14 weeks to 20 weeks from the moment of conception denote as:
A. Miscarriage
B. *Late abortion
C. Early abortion
D. Premature birth
E. Criminal abortion
579.
Allocation of the fetus from the uterus of 28 to 37 weeks is called:
A. Miscarriage
B. Late abortion
C. Early abortion
D. *Premature birth
E. Criminal abortion
580.
Abortion was carried out outside the boundaries of the medical
institution is called:
A. Septic abortion
B. *Criminal abortion
C. Artificial board
D. Cystic mole
E. Vesiculare cystic mole
581.
It is called an abortion outside a medical institution:
A. Septic abortion
B. *Criminal abortion
C. Artificial board
D. Puzirny mole
E. Vesiculare cystic mole
582.
Villi cystic placenta reborn, their number increased dramatically,
becoming a conglomerate of the vine with:
A. *Cyst change of placenta
B. Premature birth
C. Late abortion
D. Miscarriage
E. Trubnaya pregnancy
583.
What is the name of the process, when the placenta villi cystic reborn,
their number increased dramatically, becoming a conglomerate of vines:
A. *Vesiculare mole
B. Premature birth
C. Late abortion
D. Miscarriage
E. Tubular Pregnancy
584.
Name the process in endometrium which appears diffuse leukocyte
infiltration.
A. Placental polyp
B. Horionepithelioma
C. *Endometritis
D. Septic endometritis
E. Glandular hyperplasia of endometrium
585.
Embriopathias A. *Pathology embryonic period, with 16 days of pregnancy
B. Pathology embryonic period, with 20 days of pregnancy
C. Pathology embryonic period, with 40 days of pregnancy
D. Pathology embryonic period, with 52 days of pregnancy
E. Pathology embryonic period, with 58 days of pregnancy
586.
Which includes embriopatias A. *Pathology embryonic period, with 16 days of pregnancy
B. Pathology embryonic period, with 20 days of pregnancy
C. Pathology embryonic period, with 40 days of pregnancy
D. Pathology embryonic period, with 52 days of pregnancy
E. Pathology embryonic period, with 58 days of pregnancy
587.
What is embriopathias A. *Pathology embryonic period, with 16 days of pregnancy
B. Pathology embryonic period, with 20 days of pregnancy
C. Pathology embryonic period, with 40 days of pregnancy
D. Pathology embryonic period, with 52 days of pregnancy
E. Pathology embryonic period, with 58 days of pregnancy
588.
The most difficult gaps arise in the development of the child injury in
the early neural tube bookmark:
A. *3-4 weeks of intrauterine development
B. 5-6 week of intrauterine development
C. 7-8 week of intrauterine development
D. 9-10 weeks of intrauterine development
E. 11-12 weeks of intrauterine development
589.
What are the disadvantages of the most difficult child development
occur when damage to the neural tube early favorite:
A. 3-4 weeks of intrauterine development
B. 5-6 week of intrauterine development
C. 7-8 week of intrauterine development
D. 9-10 weeks of intrauterine development
E. 11-12 weeks of intrauterine development
590.
When the most difficult developmental damage in children with neural
tube early favorite:
A. *3-4 weeks of intrauterine development
B. 5-6 week of intrauterine development
C. 7-8 week of intrauterine development
D. 9-10 weeks of intrauterine development
E. 11-12 weeks of intrauterine development
591.
Microcephaly, primary marrow hypoplasia relate to:
A. *Embriopathias
B. Fetopathias
C. Pathology consequences
D. Secular change
E. Perinatal Pathology
592.
To which group pathology include microcephaly-(hypoplasia main
brain):
A. *Embriopathias
B. Fetopathias
C. Pathology consequences
D. Secular change
E. Perinatal Pathology
593.
Indicate which group pathology include microcephaly-(hypoplasia
main brain):
A. *Embriopathias
B. Fetopathias
C. Pathology consequences
D. Secular change
E. Perinatal Pathology
594.
Born hydrocephalus, the excess accumulation of liquor in the cerebral
ventricle or subarachnoid space, refers to:
A. *Embriopathias
B. Fetopathias
C. Pathology consequences
D. Secular change
E. Perinatal Pathology
595.
In which group of diseases include hydrocephalus nee-(excessive
accumulation of liquor in the cerebral ventricle or subarachnoid space):
A. *Embriopathias
B. Fetopathias
C. Pathology consequences
D. Secular change
E. Perinatal Pathology
596.
Specify which group of diseases include hydrocephalus nee-(excessive
accumulation of liquor in the cerebral ventricle or subarachnoid space):
A. *Embriopathias
B. Fetopathias
C. Pathology consequences
D. Secular change
E. Perinatal Pathology
597.
Spinal cord herniation associated with splitting dorsal vertebrae
divisions, are:
A. *Embriopathias
B. Fetopathias
C. Pathology consequences
D. Secular change
E. Perinatal Pathology
598.
To which group of diseases included spinal cord herniation associated
with splitting dorsal vertebrae divisions:
A. *Embriopathias
B. Fetopathias
C. Pathology consequences
D. Secular change
E. Perinatal Pathology
599.
Specify which group of diseases included spinal cord herniation
associated with splitting dorsal vertebrae divisions:
A. Embriopathias
B. Fetopattias
C. Pathology consequences
D. Secular change
E. Perinatal Pathology
600.
Defect mezhzheludochnoy partition belongs to:
A. Fetopathias
B. Pathology consequences
C. *Embriopathias
D. Birth trauma
E. Perinatal Pathology
601.
To which group of diseases include defect ventricular septa:
A. Fetopathias
B. Pathology consequences
C. *Embriopathias
D. Birth trauma
E. Perinatal Pathology
602.
Specify which group of diseases include defect ventricular septa:
A. Fetopathias
B. Pathology consequences
C. *Embriopathias
D. Birth trauma
E. Perinatal Pathology
603.
Complete transposition pulmonary artery and aorta refers to:
A. Fetopathias
B. Pathology consequences
C. Birth trauma
D. *Embriopathias
E. Perinatal Pathology
604.
To which group of diseases include full transposition pulmonary artery
and aorta:
A. Fetopathias
B. Pathology consequences
C. Birth trauma
D. *Embriopathias
E. Perinatal Pathology
605.
Specify which group of diseases include full transposition pulmonary
artery and aorta:
A. Fetopathias
B. Pathology consequences
C. Birth trauma
D. *Embriopathias
E. Perinatal Pathology
606.
Stenosis and atresia occurs when the pulmonary artery walls of blood
replacement barrel to the right, often with a defect ventricular septa, and is a
manifestation of:
A. Fetopathias
B. Pathology consequences
C. Birth trauma
D. *Embriopathias
E. Perinatal Pathology
607.
Triad Fallouh (defect ventricular septa, pulmonary artery stenosis, right
ventricular hypertrophy) is to:
A. Fetopathias
B. Pathology consequences
C. Birth trauma
D. Perinatal Pathology
E. *Embriopathias
608.
To which the pathology refers tetrad Fallouh (defect ventricular septa,
narrowing of the pulmonary artery, dekstrapozition of aorta, right ventricular
hypertrophy):
A. Fetopathias
B. *Embriopathias
C. Pathology consequences
D. Birth trauma
E. Perinatal Pathology
609.
Polycystic of liver - multiple cysts of various sizes, found polycystic of
the kidneys and pancreas, refers to:
A. *Embriopathias
B. Fetopathias
C. Pathology consequences
D. Birth trauma
E. Perinatal Pathology
610.
To which the pathology of the liver include polycystic - (multiple
cysts of various sizes, found out polycystic kidney and pancreas):
A. *Embriopathias
B. Fetopathias
C. Pathology consequences
D. Birth trauma
E. Perinatal Pathology
611.
Ageneziya of kidney congenital absence of one or both kidneys is
related to
A. *Embriopathias
B. Fetopathias
C. Pathology consequences
D. Birth trauma
E. Perinatal Pathology
612.
What pathology refers ageneziya kidney - the absence of one or both
kidneys:
A. *Embriopathias
B. Fetopathias
C. Pathology consequences
D. Birth trauma
E. Perinatal Pathology
613.
Renal hypoplasia - reducing weight and volume of the kidney may be
one-and two-way:
A. Fetopathias
B.
C.
D.
E.
*Embriopathias
Pathology consequences
Birth trauma
Perinatal Pathology
614.
Which renal pathology include renal hypoplasia - reducing weight and
volume of the kidney may be one-and two-way:
A. Fetopathias
B. *Embriopathias
C. Pathology consequences
D. Birth trauma
E. Perinatal Pathology
615.
Displasia renal hypoplasia, with the simultaneous presence of
embryonic kidney tissue, refers to:
A. Fetopathias
B. Pathology consequences
C. Birth trauma
D. Perinatal Pathology
E. *Embriopathias
616.
Congenital emphysema - a dramatic swelling are the top percentage of
the left lung with hypoplasia cartilage, elastic and muscular bronchial tissue
related to:
A. *Fetopathias
B. Pathology consequences
C. Birth trauma
D. Perinatal Pathology
E. Embriopathias
617.
Dislocations and dysplasia of the hip joint is related to:
A. Pathology consequences
B. Birth trauma
C. Perinatal Pathology
D. Embriopathias
E. *Fetopathias
618.
Polidaktiliya - Increase the number of fingers is related to:
A. *Fetopathias
B. Pathology consequences
C. Birth trauma
D. Perinatal Pathology
E. Embriopathias
619.
What pathology refers polydactylia - Increase the number of fingers:
A. *Fetopathias
B. Pathology consequences
C. Birth trauma
D. Perinatal Pathology
E. Embriopathias
620.
Perinatal period starts from:
A. *196 days fetal life and lasts for up to 1 week of life outside the mother's
body
B. 197 days fetal life and lasts for up to 1 week of life outside the mother's
body
C. 198 days fetal life and lasts for up to 1 week of life outside the mother's
body
D. 199 days fetal life and lasts for up to 1 week of life outside the mother's
body
E. 200 days fetal life and lasts for up to 1 week of life outside the mother's
body
621.
When starting the perinatal period:
A. *196 days fetal life and lasts for up to 1 week of life outside the mother's
body
B. 197 days dnya fetal life and lasts for up to 1 week of life outside the
mother's body
C. 198 days fetal life and lasts for up to 1 week of life outside the mother's
body
D. 199 days fetal life and lasts for up to 1 week of life outside the mother's
body
E. 200 days fetal life and lasts for up to 1 week of life outside the mother's
body
622.
Infant mortality and childbirth are called:
A. Perinatal mortality
B. Intranatal
C. *Antenatal
D. Neonatal
E. -623.
Hyaline membrane lung disease refers to:
A. Antenatal pathology
B. Intranatal pathology
C. Postnatal pathology
D. *Pneumopathy
E. -624.
What pathology include hyaline membrane lung disease:
A. Antenatal pathology
B. Intranatal pathology
C. Postnatal pathology
D. *Pneumopathy
E. -625.
Birth injury (damage to tissues and organs of the fetus mechanical
forces) occurs:
A. Before birth
B. *At the time of childbirth
C. After childbirth
D. As a consequence of childbirth
E. -626.
To which the pathology include birth injury (damage to tissues and
organs of the fetus mechanical forces):
A. Before birth
B.
C.
D.
E.
*At the time of childbirth
After childbirth
As a consequence of childbirth
-627.
Indicate how pathology relates to birth injury (damage to tissues and
organs of the fetus mechanical forces):
A. Before birth
B. *At the time of childbirth
C. After childbirth
D. As a consequence of childbirth
E. -628.
Causes of fetal birth trauma, set in a position of the fetus during labor is
all listed, except:
A. High moving vessels
B. Lack of blood prothrombin
C. The shortage of vitamins C and P
D. Softness bone
E. *The shortage of vitamin C
629.
What pathology include birth injury (damage to tissues and organs of
the fetus mechanical forces):
A. Before birth
B. *At the time of childbirth
C. After childbirth
D. As a consequence of childbirth
E. –
630.
631.
632.
Extrapulmonary complication of croupous pneumonia can be:
A. abscess of the lung
B. empyema of pleura
C. gangrene of the lung
D. bronchitis
E. meningitis
ANSWER: E
Extrapulmonary complication of croupous pneumonia can be:
A. abscess of the lung
B. empyema of pleura
C. gangrene of the lung
D. bronchitis
E. meningitis
ANSWER: E
Extrapulmonary complication of croupous pneumonia can be:
A. peritonitis
B. abscess of the lung
C. empyema of pleura
D. gangrene of the lung
E. carnification
ANSWER: A
633.
At dissection examination in intimae of aorta of the abdominal
department the flat yellow bars, which do not rise above its surface were found.
The presence of leiomyocytes is histologically marked, Macrophages with
foamy cytoplasm. At colouring of sudan III in these cells are marked yellow
granules. Specify the credible stage of atherosclerosis.
A. Lipoidosis
B. Prelipi
C. Liposclerosis
D. Atheromatosis
E. .Atherocalcinosis
ANSWER: A
634.
On a section fibrinous inflammation of serous and mucus membranes,
swollen cerebrum and lungs were found out in a patient which died with
uremia. In kidney arteries are numeral atherosclerotic plaques which close 2/3
of lumen. In kidneys there are plural band scars, polycystosis. Specify the form
of nephrosclerosis .
A. Primary wrinkled kidney
B. Secondary wrinkled kidney
C. Atherosclerotic wrinkled kidney
D. Amyloidal wrinkled kidney
E. Born polycystosis of kidneys
ANSWER: C
635.
The woman had suffered from a rheumatic heart disease for a long
period of time. She was hospitalized with complaints of short breath, edema on
legs, ascites and enlarged liver. She died of chronic cardiac insufficiency. In
section one could observe mitral stenosis. Which of the morphogenetic factors
is the most probable?
A. Sclerosis and thickening of cusps
B. Shortening of tendinous fibres
C. Sclerosis and shortening of cusps
D. Inosculation of cusps
E. Small thrombi on the surface of valves
ANSWER: D
636.
A child died after adenoviral infection and low lobular bilateral
pneumonia. On a section the sites of suppuration and necrosis were found out.
Round the sites of necrosis there is serous-hemorrhagic inflammation. Specify
the type of pneumonia after the credible etiology.
A. Viral
B. Staphylococcus
C. Streptococcus
D. Mycoplasmic
ANSWER: B
637.
Gastrobiopsy has revealed chronic atrophic gastritis with considerable
decrease of epithelium cells number. It is also known that the patient has a high
level of gastrinaemia and low level of hydrochloric acid in gastric juices and
signs of anaemia. Name the most probable disease.
A. Chronic fundic gastritis
B. Chronic stomach ulcer
C. Reflux gastritis
D. Stomach haemorrhage
E. Giant hypertrophic gastritis
ANSWER: A
638.
The person affected by peptic ulcer suffers from frequent vomiting of
undigested food and periodic convulsion. An X-ray examination of stomach
revealed pyloric stenosis. What complication has developed in the patient’s
body?
A. Perforated ulcer
B. Penetrating ulcer
C. Acute ulcer
D. Malignization
E. Chlorohydropenic tetany
ANSWER: E
639.
The person affected by stomach ulcer began to vomit with “coffee
grounds” character. What complication has developed?
A. Haemorrhage from esophagus
B. Stomach haemorrhage
C. Malignization of ulcer
D. Perforation of ulcer
E. Penetration of ulcer
ANSWER: B
640.
Cervical node examinations revealed the effaced picture, diffused spread
of lymph cells, the presence of eosinophils, giant binuclei cells, nidi of necrosis
and sclerosis. What type of lymphogranulomatosis is represented?
A. With lymph tissue domination
B. With lymph tissue oppression
C. Mixed cellular
D. Mixed
E. Nodular sclerosis
ANSWER: C
641.
During the necropsy of man, 48 years in intimae of aorta of the
abdominal department the flat yellow bars, which do not rise above its surface
were found. The presence of leiomyocytes is histologically marked,
Macrophages with foamy cytoplasm. At colouring of sudan III in these cells
are marked yellow granules. Specify the credible stage of atherosclerosis.
A. Prelipid
B. Lipoidosis
C. Liposclerosis
D. Atheromatosis
E. .Atherocalcinosis
ANSWER: B
642.
During histological and histochemical research of coronal vessels of
heart of man, who died suddenly, amorphous masses, which consisted of
crystals of cholesterol, fragments of elastic and collagenic fibres are
determined. The masses are marked off from lumen of arteries by hyalinised
the connective tissue. Specify the credible stage of atherosclerosis.
A. Prelipid
B. Lipoidosis
C. Liposclerosis
D. Atheromatosis
E. Atherocalcinosis
ANSWER: D
643.
During necropsy of old man, who died of acute myocardial infarction ,
an aorta in an abdominal department bursts saccatee. Its wall is refined, with
yellow and white bars, areas of stony closeness. Specify the name of the
pathologically changed aorta.
A. Displasia
B. Atrophy from pressure
C. Atrophy disfunctional
D. Hypertrophy working
E. Aneurysm
ANSWER: E
644.
An old man entered clinic with the complaints about sharp pains in an
abdominal region. Death came in a few minutes. On a section in the abdominal
department of aorta the saccate thrusting was found out. A wall is refined with
the presence of defect with unequal edges; the surrounding tissues are saturated
with blood. Complication of what form of arteriosclerosis took place in this
case?
A. Ischemic diseases of heart
B. Abdominal ischemic disease
C. Atherosclerosis of aortas
D. Syphilitic mesaortitis
E. System vasculitis
ANSWER: B
645.
A man, who died from the cardio-vascular insufficiency, at necropsy in
the coronal arteries of heart numerous atherosclerotic plaques were found out
which close 2/3 of lumen of vessel. At histological research of myocardium
diffuse surplus development of the connective tissue and proliferation of
fibroblasts are marked in interstitium, dystrophy and hypertrophy of
cardiomyocytes. Specify a credible disease.
A. Acute ischemic heart diseases (myocardial infarction )
B. Acute ischemic heart diseases (stage of ischemic dystrophy)
C. Chronic ischemic heart diseases (diffuse cardiosclerosis)
D. Chronic ischemic heart diseases (postattack cardiosclerosis)
E. Chronic ischemic heart diseases (chronic aneurysm of heart)
ANSWER: C
646.
A patient died of uremia. On a section fibrinous inflammation of serous
and mucus membranes, swollen cerebrum and lungs were found out. In kidney
arteries are numeral atherosclerotic plaques which close 2/3 of lumen. In
kidneys there are plural band scars, polycystosis. Specify the form of
nephrosclerosis .
A. Primary wrinkled kidney
B. Secondary wrinkled kidney
C. Amyloidal wrinkled kidney
D. Atherosclerotic wrinkled kidney
E. Born polycystosis of kidneys
ANSWER: D
647.
A man, 40 years, died suddenly after a considerable psyhoemotional
overstrain. Histologically and histochemically in the myocardium of the left
ventricle of heart paretic expansion of capillaries, focus eozinofilia of
cardiomyocytes with the loss of them banding, diminishing of number of
granules of glycogen were found. Intimae of coronal arteries is waval,
endothelicytes take a place as a tile. Specify the credible pathology.
A. Acute myocardial infarction , stage of ischemic dystrophy
B. Acute myocardial infarction , stage of necrosis
C. Acute myocardial infarction , stage of scarring
D. Repeated myocardial infarction
E. Relapsable myocardial infarctio
ANSWER: A
648.
A female patient has suffered from rheumatism complicated by mitral
defect since childhood. In the recent years, signs of cardiovascular collapse and
cough with rusty expectoration have frequently occurred. Name the
complication that has developed in the lungs.
A. Brown induration of lungs
B. Emphysema
C. Atelectasis
D. Pneumosclerosis
E. Bronchiectasis
ANSWER: A
649.
A female patient had suffered from a rheumatic heart disease for a long
period of time. She was hospitalized with complaints of short breath, edema on
legs, ascites and enlarged liver. She died of chronic cardiac insufficiency. In
section one could observe mitral stenosis. Which of the morphogenetic factors
is the most probable?
A. Sclerosis and thickening of cusps
B. Shortening of tendinous fibres
C. Sclerosis and shortening of cusps
D. Inosculation of cusps
E. Small thrombi on the surface of valves
ANSWER: D
650.
At the incision of a patient who had suffered from rheumatoid arthritis
one found enlargement of kidney that was of yellow-white colour with waxy
hue and quite dense. Stained with Congo-rot, it showed deposition of pink
masses in capillary loops of glomerules, on the walls of arterioles, arteries, in
basal membrane of tubules, in the stroma. What complication of rheumatoid
arthritis developed in the patient’s body?
A. Acute necrotic nephrosis
B. Secondary renal amyloidosis
C. Fibroblastic glomerulonephritis
D. Rapidly progressing glomerulonephritis
E. Post-infectious glomerulonephritis
ANSWER: B
651.
At histologic examination of the patient’s heart auricle for mitral stenosis
one has found Aschoff-Talalayev granulomas. What genesis of heart disease
suggests itself?
A. Rheumatic
B. Septic
C. Congenital
D. Syphilitic
E. Atherosclerotic
ANSWER: A
652.
An old man with the stomach cancer had pneumonia from which he died.
On section: the basal parts of lungs from the two lobes are of grey color.
Histologically: exudate from mucus, neutrophiles and filaments of fibrin, is
distributed in alveolar ways unevenly: in one there is much, in other- few. The
sites of emphysema, atelectasis are determined. Bronchitis is expressed.
Specify the type of pneumonia.
A. Lobar
B. Bronchopneumonia
C. Interstitial
D. Friedlander’s
ANSWER: B
653.
A child after adenoviral infection had lowlobular bilateral pneumonia
from which he died. On a section the sites of suppuration and necrosis were
found out. Round the sites of necrosis there is serous-hemorrhagic
inflammation. Specify the type of pneumonia after the credible etiology.
A. Viral
B. Staphylococcus
C. Streptococcus
D. Mycoplasmic
ANSWER: B
654.
Lobar pneumonia was diagnosed in a clinic. In a week purulent
meningitis developed from which the man died. It is discovered on section that
the lower part of the right lung developed into an airless fleshy tissue.
Histologically – the masses of fibrin in alveolar ways developed granulation
tissue. Specify the complication of lobar pneumonia.
A. Gangrene
B. Carnification
C. Pneumocirrhosis
D. Black induration
ANSWER: B
655.
During a birth, in the premature removing of the placenta layer by layer,
pneumonia was marked from which the new-born child died. It is discovered
on a section that lungs are enlarged in volume and dense. Histologically in
bronchiole and alveolar ways meconium, and clots of fat, bronchitis and
purulent alveolitis were discovered. Specify the credible form of pneumonia.
A. Lobar
B. Intermediate
C. Aspirate
D. Lipid
ANSWER: C
656.
Histologic examination of gastrobiopsy has revealed chronic atrophic
gastritis with considerable decrease of epithelium cells number. It is also
known that the patient has a high level of gastrinaemia and low level of
hydrochloric acid in gastric juices and signs of anaemia. Name the most
probable disease.
A. Chronic fundic gastritis
B. Chronic stomach ulcer
C. Reflux gastritis
D. Stomach haemorrhage
E. Giant hypertrophic gastritis
ANSWER: A
657.
A person affected by peptic ulcer suffers from frequent vomiting of
undigested food and periodic convulsion. An X-ray examination of stomach
revealed pyloric stenosis. What complication has developed in the patient’s
body?
A. Perforated ulcer
B. Penetrating ulcer
C. Acute ulcer
D. Malignization
E. Chlorohydropenic tetany
ANSWER: E
658.
A person affected by stomach ulcer began to vomit with “coffee
grounds” character. What complication has developed?
A. Haemorrhage from esophagus
B. Stomach haemorrhage
C. Malignization of ulcer
D. Perforation of ulcer
E. Penetration of ulcer
ANSWER: B
659.
A person affected by stomach ulcer suddenly felt pain in epigastrium due
to the development of a collapse. At examination one found that the abdominal
wall was very tense and painful. A laparotomy revealed a through defect with
dense edges and gastric contents flowing out of it in the area of the pyloric
part, the mesentery was hyperaemic and of dirty-grey colour with grey
deposition. Set the diagnosis taking into consideration the clinical picture and
the operational findings.
A. Penetrating stomach ulcer
B. Perforated stomach ulcer
C. Acute stomach ulcer
D. Chronic stomach ulcer
E. Malignant stomach ulcer
ANSWER: B
660.
At an operation on chronic appendicitis was discovered excrescence of
mucus-like tissue on the peritoneum and cystic dilatation of the appendix. The
latter is filled with mucus and mucous masses in the form of layers. What is the
diagnosis?
A. Phlegmonous appendicitis
B. Apostematous appendicitis
C. Primary gangrenous appendicitis
D. Mucocele
E. False appendicitis
ANSWER: D
661.
Microscopic examination of an enlarged appendix revealed diffuse
suppurative inflammation of the organ and the mesentery as well as ulcerative
defects of the mucous tunic. Identify the form of appendicitis.
A. Acute simple appendicitis
B. Acute superficial appendicitis
C. Apostematous appendicitis
D. Ulcerophlegmonous appendicitis
E. Secondary gangrenous appendicitis
ANSWER: D
662.
A patient died of acute loss of blood. At autopsy a large quantity of
blood and blood clots was found in the stomach and the intestines. In the
stomach wall on the lesser curvature there was a deep defect with dense edges
and a patulous vessel at the bottom. Set the diagnosis.
A. Acute stomach ulcer
B. Stomach cancer
C. Exacerbation of chronic stomach ulcer
D. Perforation of stomach ulcer
E. Penetration of stomach ulcer
ANSWER: C
663.
Macroscopic examination of an enlarged appendix has shown that the
serous tunic is of dirty-grey colour and hyperaemic. The appendix is thickened
and its mesentery is swollen and hyperaemic. Microscopically, one can see that
the inflammatory infiltration involves all layers of the appendix wall. Identify
the morphologic form of acute appendicitis.
A. Simple
B. Superficial
C. Phlegmonous
D. Apostematous
E. Gangrenous
ANSWER: C
664.
Macroscopic examination of an enlarged appendix has shown that it is
thickened and its serous tunic and mesentery is hyperaemic and of dirty-grey
colour. Microscopically one can observe diffuse leucocytic infiltration and
microabscesses. Identify the morphologic form of acute appendicitis.
A. Simple
B. Superficial
C. Phlegmonous
D. Apostematous
E. Gangrenous
ANSWER: D
665.
A laparoscopy revealed signs of peritonitis and thrombosis of mesentery
vessels of the appendix. The appendix was thickened and the serous tunic was
of dirty-grey colour. Microscopically one found haemorrhages, vessel
thrombosis and appendix wall necrosis. Identify the morphologic form of acute
appendicitis.
A. Simple
B. Phlegmonous
C. Ulcerophlegmonous
D. Apostematous
E. Gangrenous
ANSWER: E
666.
For a woman, 92, which died at the phenomena of cardiovascular
insufficiency, at life the phenomena of forgetfulness were marked, it is set on a
section, that bend of cerebrum narrow, sulci are deep. In the vessels of the base
of brain numerous atherosclerotic plaqueswhich close 2/3 orifice. Specify the
credible displays of pathology of cerebrum.
A. Sharp ischemia
B. It was swollen
C. Swelling
D. Heart arrest
E. Atrophy
ANSWER: E
667.
For a woman years, which long time suffered from the atherosclerotic
defeat of vessels of cerebrum, on a section, diminishing of sizes of
hemispheres, smoothing of crinkles and deepening of fissure are found. What
is the reason of atrophy changes in main of dying woman?
A. From insufficient blood supply
B. From high pressure
C. From actions of physical and chemical factors
D. From decline of function
E. From violation of innervation
ANSWER: A
668.
A 37-year-old woman was operated on for ulcerophlegmonous
appendicitis. Three days later she suffered from hectic fever, pain in the area of
the right hypochondrium, icteritiousness of skin and visible mucous tunics; in
her blood the increased levels of conjugated and unconjugated bilirubin were
found. What complication of acute destructive appendicitis developed in the
given case?
A. Minor pelvis abscesses
B. Subphrenic space abscesses
C. Pylephlebitic abscesses
D. General fibrinopurulent peritonitis
E. Viral hepatitis
ANSWER: C
669.
A 6-year-old child was taken to the resuscitation department with signs
of hepatorenal syndrome which developed after eating mushrooms. The next
day she died. In section the liver was diminished, the capsule – wrinkled, the
parenchyma – of cherry-brown colour, with numerous haemorrhages.
Histologically one could observe deranged liver particle structure, massive
centrolobular hepatocytes necrosis, central veins of particles were preserved
but overfilled with blood. What disease are such changes typical for?
A. Parenchymatous hepatitis
B. Fatty liver
C. Acute active hepatitis
D. Massive hepatic necrosis
E. Persistent hepatitis
ANSWER: D
670.
For the sake of diagnosis of a liver pathology, patient c. underwent
puncture biopsy of the organ. Histologically, hydropic degeneration and
necrosis of hepatocytes and a large quantity of Councilman’s corpuscles were
seen. Electronomicroscopically were hyaloid hepatocytes and sandy nuclei.
What diagnosis is the most probable?
A. Progressive massive necrosis
B. Viral hepatitis
C. Acute alcoholic hepatitis
D. Persistent alcoholic hepatitis
E. Hepatic injury
ANSWER: B
671.
At the cholecystectomy because of chronic calculous cholecystitis, was
discovered that the gallbladder was completely filled with faceted stones. What
is the type of jaundice in this case?
A. Mechanical
B. Obstructive
C. Haemolytic
D. Parenchymatous
E. No jaundice
ANSWER: E
672.
Mushroom poisoning of a patient resulted in jaundice with signs of
hepatic failure. One observed progressing liver diminution. Identify the most
probable pathology.
A. Nutmeg cirrhosis
B. Biliary cirrhosis
C. Obstructive jaundice
D. Postnecrotic cirrhosis
E. Massive hepatic necrosis
ANSWER: E
673.
A patient has undergone 8-year treatment for liver pathology which
developed after viral hepatitis type A (Botkin’s disease). Histological
examination of the biopsy material revealed dilated and sclerosed periportal
areas, false particles and regeneration nodes and fatty degeneration of
hepatocytes. What is the most probable pathology?
A. Portal cirrhosis
B. Postnecrotic cirrhosis
C. Biliary cirrhosis
D. Liver adenoma
E. Hepatocellular cancer
ANSWER: A
674.
The autopsy of a woman who had suffered from calculous cholangitis for
a long time revealed an enlarged, dense, fine-grained and green liver with
dilated and bile-filled ducts on section. What is the most probable cirrhosis
form?
A. Primary biliary cirrhosis
B. Secondary biliary cirrhosis
C. Portal cirrhosis
D. Postnecrotic cirrhosis
E. Mixed cirrhosis
ANSWER: B
675.
Three months after blood transfusion jaundice was detected in the
patient’s body. The liver was enlarged. Puncture biopsy revealed ballooning
degeneration of hepatocytes and Councilman’s corpuscles. What pathologic
process is implied?
A. Viral hepatitis type A
B. Viral hepatitis type B
C. Acute massive hepatic necrosis
D. Chronic aggressive hepatitis
E. Viral hepatitis type E
ANSWER: B
676.
Palpation of a patient, affected by chronic nonspesific pheumonia for the
last five months, showed an enlarged liver. Puncture biopsy revealed
hyperaemia of central veins and sinusoids, degeneration of centrolobular
hepatocytes and hypertrophy of peripheral ones, copper cells proliferation,
continuous basic membrane in sinusoids. Identify the presumable pathologic
process in the liver.
A. Nutmeg liver
B. Nutmeg cirrhosis
C. Portal cirrhosis
D. Mixed cirrhosis
E. Hepatitis
ANSWER: A
677.
A patient died of hepatorenal insufficiency. On section one found portal
liver cirrhosis. Histologically, fatty degeneration of hepatocytes and the
presence of Mellori’s hyaline was seen. What cirrhosis developed in the
patient’s body, according to the etiologic principle?
A. Viral
B. Alcoholic
C. Autoimmune
D. Hemochromatic
E. Obstructive
ANSWER: B
678.
Patient, 55 years old, has suffered from diabetes mellitus for a long time.
On the left foot a skin became black, tissues are dense. What is your diagnosis?
A. Wet gangrene
B. Soft gangrene
C. Gas gangrene
D. Trophical ulcer
E. Bedsore
ANSWER: B
679.
The woman, 42 years old, has obesity on an overhead type, arterial
hypertension, hyperglikemia, glucosuria lasted. Death was caused by
hemorrhage in a cerebrum. Morphological research showed basophil adenoma
of the hypophysis, hyperplasy of bark of adrenal gland. What is your
diagnosis?
A. Greyvs illnes
B. Itsenko-Kushing disease
C. Acromegal
D. Diabetes mellitu
E. Hypophysial naniz
ANSWER: B
680.
At histological research of biopsy of kidney patient (70 years old) with
the phenomena of uremia, which long time was ill wth fibrocavernous
tuberculosis, found out the deposit of homogeneous eosinophilic masses at a
glomerule mesangium and in the walls of vessels which give the positive
reaction of metachromasia at colouring iodine-grune. Specify the most reliable
type of dystrophy.
A. Nephropathic inherited amyloidosis
B. Gerontic amyloidosis
C. Genetic amyloidosis
D. Primary amyloidosis
E. Secondary amyloidosi
ANSWER: E
681.
A patient directed to the hospital with the signs of sharp kidney colic.
The cystoskopy showed that from right ureter the urine is not selected.
Obturation of ureter by the stone could be the result of:
A. Polycystic renal disease
B. Oliguria and anuria
C. Primary wrinkling of kidney
D. Secondary wrinkling of kidney
E. To hydronephrosis
ANSWER: E
682.
A woman which during 10 years treats the glomerulonephritis has
appeared the signs of chronic kidney deficiency. Specify the most credible
reason of it.
A. Secondary wrinkling of kidney
B. Primary wrinkling of kidney
C. Atherosclerotic wrinkled kidney
D. Pyelonephritic wrinkled kidney
E. Amyloid wrinkled kidney
ANSWER: A
683.
A boy (10 years old) who two weeks ago carried quinsy has a sharp
rising of temperature, pain in loin, urine became turbid. Laboratory found out
the proteinuria and gematuria. Name the most reliable reason of disease.
A. Acute glomerulonephritis
B. Acute pyelonephritis
C. Nephrotic syndrome
D. Chronic glomerulonephritis
E. Necrotic nephrosis
ANSWER: A
684.
At histological research of kidneys found out next changes: sclerosis and
lymphoplasmatic infiltration of calyxes and pelvises, chronic intermediate
inflammation and sclerosis of kidney tissue,canals are sharply stretched,
lumens of them are filled wih coloid-liked maintenance (“thyroid kidney”).
Name the pathology of kidneys.
A. Acute glomerulonephritis
B. Chronic pyelonephritis
C. Acute pyelonephritis
D. Subacute glomerulonephritis
E. Amyloidosis
ANSWER: B
685.
A patient delivered to hospital in a state of shock after car accident, an
anury developed. What morphological changes of kidneys are predefine such
complication?
A. The anemia of kidney cortex
B. Necrosis of the canals epithelium
C. The oedema of intersticium
D. The hyaloid-spot dystrophy of canals epithelium
E. Stroma hemorrhage
ANSWER: B
686.
A child (8 years old) delivered to hospital with the phenomena of
moderate catarrhal quinsy and urticaria. She had a scarlatina month ago.
Treatment did not give any effect. Death came from chronic kidney deficiency.
Histological in kidney tissue there is the formation of “half-moons” from the
proliferative cells of nephrotelium and podocytes of glomerules capsules .
What most credible pathology of kidney brought on uremia?
A. Acute glomerulonephritis
B. Subacute glomerulonephritis
C. Mesangial glomerulonephritis
D. Fibroplastic glomerulonephritis
E. Nephropathy with minimum changes
ANSWER: B
687.
A child 10 years old has a nephrotic syndrome and face edema after
carried quinsy two weeks ago. In the analysis of urine is haematuria,
leukocyturia, cylindruria, unsharp proteinuria. On the nephrobiopsy there are
the plethora of malpighian corpuscle, lymphoplasmocytic infiltration of
mesangium. Serous exudate is in the lumen of capsule. Your diagnose?
A. Acute poststreptococcus glomerulonephritis
B. Acute fast-progressive glomerulonephritis
C. Chronic glomerulonephritis
D. Acute pyelonephritis
E. Acute tubulo-interstitial nephrit
ANSWER: A
688.
On the autopsy of the deceased 48 years old, which long time was ill
with cystitis and diskinesia of ureters, was found out the morphological signs
of uremia. Histological was found out a “thyroid kidney”, foci of intermediate
inflammation. What from is most reliable diagnose?
A. Acute pyelonephritis
B. Chronic pyelonephritis
C. Atherosclerotic wrinkled kidney
D. Primary wrinkled kidney
E. Amyloid wrinkled kidney
ANSWER: B
689.
A woman which during 10 years treats the glomerulonephritis has
appeared the signs of nephrotic syndrome (face and feet edemata, arterial high
blood pressure). Specify the credible pathology.
A. Primary nephrotic syndrome
B. Secondary nephrotic syndrome
C. Alport's syndrome
D. Pyelonephritic wrinkled kidney
E. Amyloid wrinkled kidney
ANSWER: B
690.
A young woman, 36 years old, died from intoxication and respiratory
insufficiency. On the section of the dead woman: tissue of lungs in all
departments is multi-coloured with plural hemorrhages, focuses of
emphysema. Histological in lungs found out hemorrhagic bronchopneumonia
with abscess formation, in the cytoplasm of epithelium of bronchial tubes there
are eosinophilic and basophilic includings. What disease did the sick woman
have?
A. Parainfluenza
B. Influenza
C. Adenoviral infection
D. Staphylococcal bronchopneumonia
E. Respiratory syncytial infection
ANSWER: B
691.
At patient who suffered from tubercular hyperpigmentation of skin,
adynamy, low blood pressure appeared. What disease developed in him?
A. Toxic nigredo
B. Illness of Icenco-Kyshin
C. Illness of Simmon
D. Pellagra
E. Illness of Addison
ANSWER: E
692.
At histological biopsy research of kidney of patient, 70 years old, with
the phenomenon of uremia, who was ill with fibrous-cavernouse tuberculosis
for a long time, the deposits of homogeneous eosinophilic masses are exposed
in mesangium of glomerule and in the walls of vessels which give the positive
reaction of metachromasia at colouring of iodine -green. Specify the most
possible type of dystrophy.
A. Secondary amyloidosi
B. Senile amyloidosi
C. Genetic amyloidosi
D. Primary amyloidosi
E. Nephropathy inherited
ANSWER: A
693.
In a child with sharply enlarged quinsy lymphatic nodes : paratracheal,
bifurcational, neck. At microscopic research of neck lymphatic knot the
following things are exposed: presence of focuses of necrosis bordered by
epithelioid cells, lymphocytes and giant cells of Pirogov-Langhans. Specify the
most possible pathology.
A. Tuberculosis
B. Sarcoidosis
C. Scarlatina
D. Diphtheria
E. Rhinoscleroma
ANSWER: A
694.
The following changes are exposed in the remote enlarged lymphatic site
of histological: round focuses of necrosis are billow from epithelioid cells
which are among single giant multinuclear cells, on periphery is a roller from
lymphocytes. What is the disease?
A. Syphili
B. Tuberculosis
C. Leprosies
D. Scleroma
E. Lymphogranulematosis
ANSWER: B
695.
A young woman has been treated for sterility for a long tme. There is a
positive test of Mantu at an inspection. Biological test on tuberculosis positive
is (infection of guinea-pig by material of aspiration of uterus cavity content).
What is your diagnosis?
A. Tuberculosis of kidneys
B. Tuberculosis of adrenals
C. Tuberculosis of Fallopian tubes
D. Tuberculosis lymphadenitis
E. Tuberculosis leptomeningitis
ANSWER: C
696.
On the section of dying K., 40 years the supervalvular aortoclasia is
found with tamponade of pericardium. At microscopic research of ascending
department of aorta in more thick walls it is found out infiltration by
lymphocytes, plasmocytes, epithelioid cells, areas of necrosis in a middle shell,
adventitia, endothelial cells. Such changes in an aorta more frequent can
develop at:
A. Syphilitic aortitis
B. Septic aortitis
C. Rheumatic aortitis
D. Atherosclerosis of aorta
E. Hypertensive illness
ANSWER: C
697.
The cervical node examinations revealed the effaced picture, diffused
spread of lymph cells, the presence of eosinophils, giant binuclei cells, nidi of
necrosis and sclerosis. What type of lymphogranulomatosis is represented?
A. With lymph tissue domination
B. With lymph tissue oppression
C. Mixed cellular
D. Mixed
E. Nodular sclerosis
ANSWER: C
698.
At the dissection doctor hold necropsy of old man, who died of acute
myocardial infarction , an aorta in an abdominal department bursts saccatee. Its
wall is refined, with yellow and white bars, areas of stony closeness. Specify
the name of the pathologically changed aorta.
A. Displasia
B. Atrophy from pressure
C. Atrophy disfunctional
D. Hypertrophy working
E. Aneurysm
ANSWER: E
699.
A patient died from the chronic cardiac insufficiency. The syndrome of
lameness was marked during life. On a section atrophy and dryness of skin and
atrophy of muscles of shin, gangrene of the first finger of foot were found out.
What were the credible changes of vessels?
A. Thromboflebitis
B. Vasculitis
C. –
D. Capillaritis
E. Atherosclerosis
ANSWER: E
700.
At histologic examination Aschoff-Talalayev granulomas were founded
in the patient’s heart auricle with mitral stenosis. What genesis of heart disease
suggests itself?
A. Rheumatic
B. Septic
C. Congenital
D. Syphilitic
E. Atherosclerotic
ANSWER: A
701.
A patient with the symptoms of the dyspnoea and cough uremia. At
bronchoscopy mucus bronchial tubes are sanguineous, filling out with shallow
hemorrhages. In the lumen of bronchial tubes there is much mucus. Specify the
form of bronchitis.
A. Primary acute
B. Secondary acute
C. Primary chronic
D. Secondary chronic
E. Deformate.
ANSWER: B
702.
The doctor diagnosed lobar pneumonia in a clinic. In a week purulent
meningitis developed from which the man died. It is discovered on section that
the lower part of the right lung developed into an airless fleshy tissue.
Histologically – the masses of fibrin in alveolar ways developed granulation
tissue. Specify the complication of lobar pneumonia.
A. Gangrene
B. Carnification
C. Pneumocirrhosis
D. Black induration
ANSWER: B
703.
The patient with jaundice was operated with diagnosis "chronic
calculous cholecystitis". What is the type of jaundice in this case?
A. Mechanical
B. Suprahepatic
C. Haemolytic
D. Parenchymatous
E. No jaundice
ANSWER: A
704.
The woman has been treated for sterility for a long time. There is a
positive test of Mantu at an inspection. Biological test on tuberculosis positive
is (infection of guinea-pig by material of aspiration of uterus cavity content).
What is your diagnosis?
A. Tuberculosis of kidneys
B. Tuberculosis of adrenals
C. Tuberculosis of Fallopian tubes
D. Tuberculosis lymphadenitis
E. Tuberculosis leptomeningitis
ANSWER: C
705.
On the dissection the supervalvular aortoclasia is found with tamponade
of pericardium. At microscopic research of ascending department of aorta in
more thick walls it is found out infiltration by lymphocytes, plasmocytes,
epithelioid cells, areas of necrosis in a middle shell, adventitia, endothelial
cells. Such changes in an aorta more frequent can develop at:
A. Syphilitic aortitis
B. Septic aortitis
C. Rheumatic aortitis
D. Atherosclerosis of aorta
E. Hypertensive illness
ANSWER: C
706.
The patient with alcoholism suddenly had a cough with the secretion of
purulent phlegm, blood cough and fever and died from hepatargia. Dissection
showed part of the right lung is dense, heavy and grey. Histologically alveolar
ways are filled with fibrin and leucocytes, venous stagnation. What is stage of
lobar pneumonia
A. Rush.
B. Red hepatization.
C. Grey hepatization.
D. Resorbtion.
E. Carnification.
ANSWER: C
707.
An old man had a moist cough and fever after supercooling. He died on
the fourth day from pulmonary-cardiac insufficiency. On section: the lower
parts of lungs are dense with the deposits of fibrin on a pleura, by consistency
resembles a liver, on cut is grey brown. Specify the credible disease.
A. Lobar pneumonia.
B. Pneumonia of Friedlander.
C. Influenza bronchopneumonia.
D. Streptococcus bronchopneumonia.
E. Intermediate pneumonia.
ANSWER: A
708.
A man, with alcoholism, suddenly had moist cough with the secretion of
ferruginous sputum, fever developed. Lobar pneumonia was diagnosed in a
clinic. He died of the progressing pulmonary-cardiac insufficiency. It was
discovered on section that the lung was of black brown color with the
widespread of necrosis. Grey brown liquid flowed down from thesurface.
Specify the complication of lobar pneumonia.
A. Gangrene.
B. Carnification.
C. Pneumocirrhosis.
D. Black induration.
E. Brown induration.
ANSWER: A
709.
At pathoanatomical research of lungs of a child who died of measles
encephalitis, the presence of panbronchitis was revealed. An inflammatory
process passed to surrounding tissue and spread on adjoining intraalveolar
partitions, that predetermines their bulge. In alveolar ways there is exudate
which consists of alveolar macrophages and single neutrophiles.Specify the
form of pneumonia.
A. Lobar.
B. Bronchopneumonia.
C. Peribronchial.
D. Interlobular.
E. Interalveolar.
ANSWER: C
710.
At pathoanatomical research of the lungs of a dying person of purulent
mediastenitis is revealed that inflammation has flegmonous character in
mediastinal pleura with "stratification" lungs on a fate. Specify the form of
pneumonia.
A. Lobar.
B. Bronchopneumonia.
C. Peribronchial.
D. Interlobular.
E. Interalveolar.
ANSWER: D
711.
In the patient on a section the thrombosis of left middle cerebral arteria is
found. The site of grey softening of cerebrum is found in the area of ??left
hemisphere;. It is needed to define a pathological process which developed for
a patient.
A. Ischemic attack.
B. Hemorrhagic attack.
C. Swelling.
D. Abscess.
E. Atrophy.
ANSWER: A
712.
At sick, 29 years old, the particle of thyroid gland is remote.Histological
research showed considerable limphoid infiltration with the formation of
limphoid follicles, destruction of parenchymatose structures, excrescence of
connecting fabric.What pathology of thyroid gland takes place?
A. Colloid goitre.
B. Endemic goitre.
C. Parenchymatose goitre.
D. Hasimoto goitre.
E. Toxic goitre.
ANSWER: D
713.
Patient, 40 years old, has multiplied extremities: feet, hands, bones of
facial skull, lips. Found out the adenoma of hypophysis.What is the pathology?
A. Basedov's illness.
B. Acromegaly.
C. Diabetes mellitus.
D. Diabetes insipidus.
E. Addisons illness.
ANSWER: B
714.
At insufficiency of circulation of blood, especially on a background the
stagnant phenomena in lungs at patients can arise up:
A. stagnant pneumonia
B. hypostatic pneumonia
C. paravertebral pneumonia
D. atelectatic pneumonia
E. croupous pneumonia
ANSWER: A
715.
At the protracted bed mode at heavy and hyposthenic patients in lungs
can arise up:
A. stagnant pneumonia
B. aspiration pneumonia
C. atelectatic pneumonia
D. croupous pneumonia
E. hypostatic pneumonia
ANSWER: E
716.
Blood vessels affected in atherosclerosis are which of the following:
A. Aorta only
B. Large arteries and large veins
C. Large veins
D. Aorta and arterioli
E. Aorta and large arteries
ANSWER: E
717.
Blood vessels affected in systemic hypertension are which of the
following:
A. Aorta and large arteries only
B. Aorta only
C. Large arteries and large veins
D. Venules
E. Arterioli
ANSWER: E
718.
The cells accumulating cholesterol or cholesterol esters in atherosclerotic
plaque are which of the following:
A. Neutrophils.
B. Lymphocytes and macrophages.
C. Mesenchymal cells
D. Giant cells.
E. Macrophages and smooth muscle cells
ANSWER: E
719.
Material accumulated in macrophages and smooth muscle cells in
atherosclerotic plaque is which of the following:
A. Glycogen
B. Lipohyaline
C. Triglycerides
D. Lipoproteids
E. Cholesterol or cholesterol esters
ANSWER: E
720.
Material deposited in arteriolar walls in systemic hypertension is which
of the following:
A. Amyloid
B. Glycogen
C. Cholesterol or cholesterol esters
D. Lipoprotein
E. Hyaline
ANSWER: E
721.
Atherosclerosis commonly affects the arteries supplying all of the
following organs, except:
A. Lower extremities
B. Heart
C. Brain
D. Kidney
E. Upper extremities
ANSWER: E
722.
The consequences of atherosclerosis are all of the following, except:
A. Myocardial infarction
B. Cerebral infarct
C. Gangrene of lower extremities
D. Abdominal aortic aneurism
E. Cardiomyopathy
ANSWER: E
723.
The complications of atheromatous plague are all of the following,
except:
A. Thrombus formation
B. Plaque rupture
C. Calcification
D. Hemorrhage
E. Plague malformation
ANSWER: E
724.
All of the following components can be found in the necrotic center of
atheromatous plaque, except:
A. Cell debris
B. Cholesterol crystals
C. "Foam" cells
D. Calcium crystals
E. Glycogen granules
ANSWER: E
725.
Cholesterol-laden macrophages and smooth muscle cells in atheromatous
plaque are also referred to as:
A. Giant cells
B. "Popcorn" -cells
C. Epithelioid cells
D. Vacuolated cells
E. "Foam" cells
ANSWER: E
726.
The fibrous cap of atheromatous plaque consists of all of the following
components, except:
A. Smooth muscle cells
B. Macrophages
C. Collagen and elastan
D. Proteoglycans
E. Glycogen
ANSWER: E
727.
The lowered risk of ischemic heart disease in atherosclerosis depends on
which of the following:
A. High serum level of low-density lipoproteins
B. High serum level of very low density lipoproteins
C. High serum-level of intermediate-density lipoproteins
D. High serum level of triglycerides
E. High serum level of high-density lipoproteins
ANSWER: E
728.
The well developed atheromatous plaque results from all of the
following processes, except:
A. Endothelial injury with adhesion of monocytes and platelets
B. Migration of monocytes from the lumen and smooth muscle cells from
the media into the intima
C. Lipid absorption by macrophages and smooth muscle cells
D. Smooth muscle cell proliferation, collagen deposition
E. Action of membrane-bound tissue factor
ANSWER: E
729.
The chronic endothelial injury in atherosclerosis may be caused by all of
the following factors, except:
A. Hyperlipidemia
B. Hypertension
C. Smoking
D. Immune reactions
E. Genetic defects
ANSWER: E
730.
The chronic endothelial injury in atherosclerosis includes all pathologic
processes, except:
A. Increased endothelial permeability
B. Adhesion of blood monocytes to the endothelium
C. Adhesion of platelets to the intima
D. Insudation of lipoproteins into the vessel wall
E. Regeneration of endothelial cells
ANSWER: E
731.
The clinical syndrome of malignant hypertension includes all ofthe
following symptoms, except:
A. Retinal hemorrhages
B. Diastolic pressure >120 mmHg
C. Renal failure
D. Papilledema
E. Systolic pressure >120 mmHg
ANSWER: E
732.
Secondary hypertension may be caused by all of the following diseases,
except:
A. Acute glomerulonephritis
B. Gushing's syndrome
C. Pheochromocytoma
D. Coarctation of aorta
E. Lobar pneumonia
ANSWER: E
733.
The humoral constriction factors that lead to the peripheral resistence in
hypertension are all of the following, except:
A. Angiotensin II
B. Catecholamines
C. Thromboxanes
D. Leukotriens
E. Prostaglandines
ANSWER: E
734.
The renin-angiotensin system consists of all of the following
components, except:
A. Renin
B. Angiotensin I
C. Plasma angiotensinogen
D. Angiotensin II
E. Tissue angiotensinogen
ANSWER: E
735.
The antihypertensive substances produced in the kidney are all of the
following, except:
A. Prostaglandines
B. Urinary kallikrein-kinin system
C. Platelet-activating factor
D. Nitric oxide
E. Thromboxane
ANSWER: E
736.
All of the following factors may contribute to vasoconstriction, except:
A. Behavioral factors
B. Neurogenic factors
C. Increased release of vasoconstric-tive agents
D. Increased sensitivity of vascular smooth muscles to constricting agent
E. Acute coagulation factor
ANSWER: E
737.
The special stain used to identify "foam cells" in atherosclerotic plaque
is which of the following:
A. Picrofuchsin
B. Hematoxylin and eosin
C. Congo red stain
D. PAS-reaction
E. SudanIII
ANSWER: E
738.
The malignant hypertension can be identified microscopically by all of
the following features, except:
A. Onion-skin concentric laminated thickening of the arteriolar walls
B. Progressive narrowing of the arteriolar lumina
C. Fibrinoid necrosis of the arteriolar walls
D. Perivascular fibrosis
E. Cholesterol-laden "foam cells" in the arteriolar walls
ANSWER: E
739.
On gross examination, the kidneys in systemic chronic hypertension are
characterized by all of the following features, except:
A. Reduced in size and weight
B. Fine granularity on cortical surface
C. Grain leather-like cortical surface
D. Narrowed cortical surface on section
E. Hemorrhages on cortical surface
ANSWER: E
740.
The typical pathologic proces in arterioles and small arteries in systemic
hypertension is referred to as:
A. Hyaline atrophy
B. Hyaline hyperplasia
C. Fibroelastic hyalinosis
D. Hyaline atherosclerosis
E. Hyaline arteriolosclerosis
ANSWER: E
741.
All of the following are stages of the atherosclerotic plaque
development, except:
A. Fibrous plaque
B. Fibrofatty plaque
C. Atheromatous plaque
D. Fibrolipid plaque
E. Fibrinous plaque
ANSWER: E
742.
The major nonmodifiable risk factors for atherosclerosis are all of the
following, except:
A. Increasing age
B. Male gender
C. Family history
D. Genetic abnormalities
E. Increased water resorption
ANSWER: E
743.
The potentially controllable risk factors for atherosclerosis are all of the
following, except:
A. Hyperlipidemia
B. Hypertension
C. Cigarette smoking
D. Diabetes mellitus
E. Gene tic abnormalities
ANSWER: E
744.
The gene defects in enzymes involved in aldosterone metabolism in
systemic hypertension lead to all of the following effects, except:
A. Adaptive increase in aldosterone secretion
B. Increased salt resorption
C. Increased water resorption
D. Plasma volume expansion
E. Increased lipoprotein resorption
ANSWER: E
745.
The pathologic process in arterioles in malignant hypertension is referred
to as:
A. Endothrombovasculitis
B. Thromboangitis obliterans
C. Necrotizing atherosclerosis
D. Productive arteriolitis
E. Necrotizing arteriolitis
ANSWER: E
746.
Coronary artery suppying blood to the posterior portion of the
interventricular septum is which of the following:
A. Left anterior descending artery
B. Left circumflex coronary artery
C. Proximal marginal coronary artery
D. Right coronary artery
ANSWER: D
747.
Coronary artery supplying blood to the anterior part of the interventricular septum is which of the following:
A. Left main coronary artery
B. Right coronary artery
C. Left circumflex coronary artery
D. Proximal marginal coronary artery
E. Left anterior descending artery
ANSWER: E
748.
Changes in the aging heart are all of the following, except:
A. Epicardial coronary artery tortuosity
B. Decreased subepicardial fat
C. Brown atrophy of myocardium
D. Increased number of inflammatory cells in myocardium
ANSWER: D
749.
Symptoms of cardiac dysfunction include all of the following, except:
A. Failure of the pump itself
B. Obstruction of blood flow
C. Regurgitant blood flow
D. Disorders of cardiac conduction
E. Disorders of blood coagulation
ANSWER: E
750.
Microscopically, most cases of acute myocarditis are characterized by all
of th following, except:
A. Nonspecific inflammatory infiltrate
B. Variable amount of interstitial edema
C. Variable degree of myofiber degeneration
D. Variable degree of myofiber necrosis
E. Marked fibrosis of interstitium
ANSWER: E
751.
Nonbacterial thrombotic endocarditis is characterized by all of the
following, except:
A. Mitral and/or aortic valve involvement
B. Small (1 to 5 mm), sterile vegetations
C. Vegetations along the line of valve closure
D. Systemic emboli development
E. Tricuspid valve involvement
ANSWER: E
752.
Acute rheumatic carditis is characterized by all of the following, except:
A. Unremarkable gross appearance of the heart
B. The Aschoff bodies in myocardium
C. Diffuse nonspecific myocarditis
D. The Aschoff bodies in joints
E. "Tigered effect" gross appearance
ANSWER: E
753.
The correct order of histologic phases in the development of Aschoff
bodies is which of the following:
A. Granulomatous — late — exudative
B. Acute — chronic — healed
C. Nonspecific — specific
D. Early — pathognomonic — intermediate
E. Exudative - proliferative - fibrous
ANSWER: E
754.
Aschoff giant cells are large mesenchymal cells and are believed to be
derived from which of the following:
A. Fibroblasts
B. Lymphocytes
C. Eosinophils
D. Erythrocytes
E. Macrophages
ANSWER: E
755.
The valvular effects of chronic rheumatic heart disease are all of the
following, except:
A. Thickened and deformed valve leaflets
B. Valvular calcification
C. Thickened and shortened chordae tendineae
D. "Fish-mouth" appearance of mitral valve
E. Valvular abscess formation
ANSWER: E
756.
Left-sided heart failure is most often caused by all of the following,
except:
A. Ischemic heart disease
B. Hypertension
C. Aortic and mitral valvular diseases
D. Nonischemic heart disease
E. Brown atrophy of myocardium
ANSWER: E
757.
Resulting from left-sided heart failure are all of these pathologic
changes, except:
A. Brown induration of the lung
B. Hypoxic encephalopathy
C. Pulmonary edema
D. Prerenal azotemia
E. Secondary amyloidosis
ANSWER: E
758.
The nonrheumatic degenerative calcific aortic stenosis is characterized
by all of the following, except:
A. Heaped-up calcified masses within aortic cusps
B. Distorted cuspal architecture
C. Not involved free cuspal edges
D. Primary involvement of the valvular fibrosa
E. Lipid deposition and cellular proliferation
ANSWER: E
759.
The mitral annular calcification is characterized by all of the following,
except:
A. Irregular, stony hard nodule appearance
B. Possible thromboembolic complications
C. Infective endocarditis
D. Absence of inflammatory changes
E. Severely impaired valvular function
ANSWER: E
760.
The disease that is characterized by the systolic murmur without signs of
mitral valve failure is called as:
A. Rheumatic heart disease
B. Mitral annular calcification
C. Degenerative calcific aortic valve stenosis
D. Congenitally bicuspid aortic valve
E. Mitral valve prolapse
ANSWER: E
761.
Etiologically and pathogenetically, rheumatic fever and rheumatic heart
disease are characterized by all of the following, except:
A. Initial attack of illness some weeks after streptococcal infection
B. Elevated serum titers of antibodies to streptolysin and hyaluronidase
C. Sterile tissue lesions not resulting from direct bacterial invasion
D. Recurrent acute illness following the streptococcal infection
E. Decreased serum protein levels
ANSWER: E
762.
Heart lesions in acute rheumatic fever are called as which of the
following:
A. Tuberculoma
B. Foreign bodies
C. Gumma
D. Mitral stenosis
E. Aschoff bodies
ANSWER: E
763.
The Aschoff bodies consist of all of the following, except:
A. Lymphocytes (primarily T cells)
B. Anitschkow cells
C. Fibrinoid degeneration
D. Plasma cells
E. "Foam cells"
ANSWER: E
764.
The Anitschkow cells are characterized by all of the following, except:
A. Abundant amorphophilic cytoplasm
B. Central round-to-ovoid nucleus
C. Central disposed chromatin (caterpillar cells)
D. Monocytic cytogenesis
E. B-lymphocytic cytogenesis
ANSWER: E
765.
Infective endocarditis is characterized by all of the following, except:
A. Microbic invasion into the heart valves
B. Destructive vegetation formation
C. Mural thrombosis
D. Destruction of the underlying cardiac tissues
E. Amyloid mass deposition
ANSWER: E
766.
Systemic emboli in infective endocarditis may occur because of all of the
following, except:
A. Involvement of aortic and mitral valves
B. Severe valvular damage
C. Numerous amount of vegetations
D. Friable nature of vegetations
E. Chronic heart insufficiency
ANSWER: E
767.
Systemic embolism in infective endocarditis may cause infarcts in all of
the following organs, except:
A. Brain
B. Kidneys
C. Heart
D. Spleen
E. Lungs
ANSWER: E
768.
What vessels are struck by atherosclerosis?
A. Aorta
B. V. cava inferior
C. Veins
D. Arteries
E. Capillaries
ANSWER: E
769.
Microscopycally the vegetations of subacute infective endocarditis are
characterized by all of the following, except:
A. Fibrosis
B. Microbic body calcifications
C. Chronic inflammatory infiltrate
D. Granulation tissue at vegetation's bases
E. Granuloma formation
ANSWER: E
770.
Non embolic complications of infective endocarditis include all of the
following, except;
A. Valvular insufficiency or stenosis with cardiac failure
B. Myocardial ring abscess
C. Suppurative pericarditis
D. Endocarditis of artificial valves
E. Acute myocardial infarction
ANSWER: E
771.
The cardinal anatomic changes of the mitral (or tricuspid) valve in
chronic rheumatic heart disease are all of the following, except:
A. Leaflet thickening
B. Commisural fusion
C. Shortening and thickening of the tendinous cords
D. Fusion of the tendinous cords
E. Leaflet softening
ANSWER: E
772.
Microscopically, chronic rheumatic heart disease is characterized by all
of the following, except:
A. Diffuse fibrosis of leaflets
B. Neovascularization of leaflets
C. Destruction of original leaflet architecture
D. Aschoff bodies in perivascular connective tissue
E. Acute purulent inflammation
ANSWER: E
773.
Fibrosis bridging across the valvular commissures and calcification in
chronic rheumatic heart disease cause which of the following:
A. "Tigered effect"
B. Fibrofatty atheromas
C. Starry sky appearance
D. Granulomatosis
E. Fish mouth or buttonhole stenosis
ANSWER: E
774.
The morphological and clinical effects of pure (isolated) right-sided
heart failure are all of following, except:
A. Congestive hepatomegaly
B. Ascites
C. Pleural and pericardial effusions
D. Anasarca
E. Cerebral hematoma
ANSWER: E
775.
The clinical consequences of valvular dysfunction depend on all of the
following, except:
A. Valve involved
B. Degree of impairment
C. Rate of dysfunction development
D. Rate and quality of compensatory mechanisms
E. Sex of the patient
ANSWER: E
776.
What is the name of the soft tooth tissue?
A. stroma
B. dentine
C. spongy bone tissue
D. keloid
E. pulp
ANSWER: E
777.
What is a physiological anticaries factor?
A. cleaning of teeth a twice a day
B. the using of ftoric food
C. to make use of ftoric tooth-pastes
D. to chew the cuds regullary wich do not contain sugar
E. formation of transparent (sclerotic) and reparative dentine
ANSWER: E
778.
What is a favourable background for caries development?
A. hypersecretion of saliva
B. the use of hard teeth brushes
C. to chew the cuds regullary
D. nothing from mentioned above
E. sexual ripening
ANSWER: E
779.
What surfaces of teeth are destroyed by a caries rarely than other?
A. cheek's
B. contact distal
C. contact proximal
D. occlusal
E. lingual
ANSWER: E
780.
What surface of tooth is destroyed by wedge-shaped defects?
A. occlusal surface
B. contact proximal surface
C. contact distal surface
D. lingual surface of crown
E. vestibular surface of tooth collum
ANSWER: E
781.
In what essence of fluorosis development?
A. excessive mineralization of hard tissues of tooth
B. excessive mineralization of soft tissues of tooth
C. violation of tooth-alveolar connections
D. discoloration teeth without the change their durability properties
E. combined defeat of internalss with violation of forming and calcification
of tooth enamel
ANSWER: E
782.
How is the 4th degree of development of fluorosis named?
A. weak defeat
B. very weak defeat
C. the defeat is moderato expressed
D. stage of complications
E. heavy defeat
ANSWER: E
783.
What age-old categories of people more frequent are ill by teeth
erosion?
A. people of old years
B. youths and girls of young age
C. it is the disease of child's age
D. there is no age-old dependence
E. people of mature age
ANSWER: E
784.
What is structural different between highly developed denticles from
under-developed one?
A. they have a good mineralization
B. they have a good developed interdenticular connections
C. they are complete integrated in the system of tooth vital functions
D. by chemical composition
E. under-developed denticles have no dentinal canals
ANSWER: E
785.
How is the name of 1th degree of fluorosis development?
A. weak defeat
B. the defeat is moderato expressed
C. heavy defeat
D. stage of complications
E. very weak defeat
ANSWER: E
786.
What does differ the secondary dentine from primary?
A. the place of location
B. the cellular elements absence
C. no differs
D. there is no such structure in teeth
E. the mineralization degree and structure
ANSWER: E
787.
What delays the spreading of microorganisms and products of their vital
functions in transparent and reparative dentine?
A. antibacterials secretion of dentincells
B. the enhanceable osmotic pressure in a dentine
C. the mionectic osmotic pressure in a dentine
788.
789.
790.
791.
792.
793.
D. the improved trophism of such dentine
E. the chaotic placing of dentinal tubulis and their narrowing
ANSWER: E
Where does a superficial teeth caries develop?
A. a pathological process develops only on-the-spot enamel
B. a pathological process engulfs an enamel and part of dentine
C. a pathological process destroys dentinal-enamel connection
D. a pathological process is characterized by dismineralization and
demineralizing of enamel in a subsuperficial layer
E. a pathological process keeps indoors outside of an enamel
ANSWER: E
What from the indicated variants is an offtype caries?
A. middle caries
B. caries of the front teeth
C. caries of incisival teeth
D. caries of third molars
E. circular caries
ANSWER: E
What part of teeth is destroyed by wedge-shaped defects?
A. crown of the tooth
B. root of tooth
C. masticatory surface of tooth
D. crown with a neck
E. neck of tooth
ANSWER: E
How many degrees does development fluorosis have?
A. 3
B. 2
C. 5
D. there is no division on a degree
E. 4
ANSWER: E
How are the 2-nd degree of fluorosis development characterized?
A. chalky spots occupy more than the half of surface of teeth and meet
between itself
B. separated and plural erosions of enamels are different form and
colourless or pigmented. Teeth are fragile and collapse easily
C. chalky spots are small and expressed weakly. They occupied 1/3 labial
and tongue surface of teeth
D. dicking and fall out of teeth
E. chalky spots occupy near the half of area the crowns of the teeth, some
pigmented spots situated in enamel
ANSWER: E
What categories of people are ill acid necrosis of hard tissues of tooth?
A. the people, which abuse to mastications of tobacco
B. the people, which abuse products which canned by acids
C. the people, which have the mionectic рН of saliva
D. there are no such diseases of teeth
E. professional disease of teeth at people, which contact with inorganic
acids
ANSWER: E
794.
What is the name of inflammatory process which is localized directly at
connective tissue shell of tooth?
A. paradontitis
B. pulpitis
C. gingivitis
D. paradontosis
E. pericementitis
ANSWER: E
795.
What name of 3th degree of fluorosis development?
A. weak defeat
B. very weak
C. heavy defeat
D. stage of complications
E. moderato expressed defeat
ANSWER: E
796.
What structures form the denticles?
A. pulp
B. a primary dentine
C. periodont
D. the embryonic elements of the second teeth
E. the secondary dentine
ANSWER: E
797.
What age-old categories, as a rule, are ill by caries of cement?
A. the youths
B. the girls
C. the pregnant women
D. identically in the all groups
E. the old people
ANSWER: E
798.
What is the way of microorganisms diffusion in the enamel of tooth?
A. dentinal-enamel connections
B. dentinal tubulis
C. enamel prisms
D. intraprisms channels
E. the soft intraprisms matter and cracks
ANSWER: E
799.
What is the retrograde caries?
A. the caries of the back teeth
B. caries which begins from the back contact surface of tooth
C. caries which begins from cement of tooth
D. caries which begins under the enamel layer
E. caries which begins from a pulp of tooth
ANSWER: E
800.
What is the initial phase of development of wedge-shaped defects of
tooth?
A. traumatic damages at mastication
B. acid chemical influencing
C. alkaline chemical influencing
D. considerable temperature overfalls
E. trophic violations in the organic matter of enamel and dentine
ANSWER: E
801.
How is a 1 degree of development of fluorosis characterized?
A. chalky spots occupy near the half of area the crowns of the teeth, there
are separate pigmented spots in an enamel
B. chalky spots occupy more than the half of surface of teeth and meet
between itself
C. separate and plural erosions of the enamels have the different forms.
They are colourless or pigmented. The teeth are fragile and collapse easily
D. dicking and fall out of teeth
E. chalky spots are small and poorly expressed. They occupy 1/3 labial and
tongue surface of teeth
ANSWER: E
802.
How are 3 degrees of development of fluorosis characterized?
A. chalky spots occupy near the half of area the crowns of the teeth,
separate spots pigmented and situated in an enamel
B. separated and plural erosions of enamels are the different form. They are
colourless or pigmented. Teeth are fragile and collapse easily
C. chalky spots are small and weakly expressed. They occupy 1/3 labial
and tongue surface of teeth
D. dicking and fall out of teeth
E. chalky spots occupy more than the half of surface of teeth and meet
between yourself
ANSWER: E
803.
What is acid necrosis of hard tooth tissues characterized?
A. necrosis and tearing away of hard tissues of tooth with pulpitis
B. the rapid necrosis development and sequestration of teeth crowns
C. the quick progress of dentine necrosis with the saving of enamel of tooth
D. the quick progress of necrosis of dentine and enamel of tooth
E. plural demineralizing of dental crowns and their destroyed
ANSWER: E
804.
What teeth more frequent are destroyed by periodontitis?
A. superior incisives
B. inferior incisives
C. premolars
D. superior molars
E. inferior molars
ANSWER: E
805.
What essence of fluorosis development?
A. surplus mineralization of hard tooth tissues
B. surplus mineralization of soft tooth tissues
806.
807.
808.
809.
810.
811.
C. violation of tooth-alveolar connections
D. discoloration of teeth without the change them strength properties
E. combined defeat of internalss with violation of processes of forming and
calcification of teeth enamel
ANSWER: E
What groups of people more frequent are ill by caries?
A. men
B. women
C. people of mature age
D. there is no age-old and sexual gradation
E. the children and teenagers
ANSWER: E
How many stages of development of caries do you know?
A. 2 stages
B. 3 stages
C. 5 stages
D. its development have no phases
E. 4 stages
ANSWER: E
In what limit does a typical middle caries develop?
A. on 2/3 areas of enamel surface
B. a pathological process spreads on an enamel and cement
C. the destruction is localized in pulp of tooth
D. there is destruction of half of tooth crown
E. a process passes into dentin
ANSWER: E
What feature is of stationary caries?
A. it develops after medical interventions on teeth
B. it develops when the patient are in hospital
C. it develops in the rudiments of the second teeth
D. there is no such variant of caries
E. it is localized only in an enamel and is not made progress
ANSWER: E
How quickly do the wedge-shaped defects of teeth develop?
A. very quickly (sometimes instantly)
B. from a few days to a few weeks
C. from a few weeks to a few months
D. after the multiple repeated influencing
E. some years
ANSWER: E
What name of the 2th degree of fluorosis development?
A. very weak defeat
B. the defeat is moderato expressed
C. heavy defeat
D. stage of complications
E. weak defeat
ANSWER: E
812.
813.
814.
815.
816.
817.
How the 4 degrees of fluorosis development are characterized?
A. chalky spots occupy near the half of area the crowns of the teeth, some
separated spots are pigmented and situated in an enamel
B. chalky spots occupy more than the half of surface of teeth and meet
between itself
C. chalky spots occupy near the 1/3 of the labial| and tongue surface of the
teeth
D. dicking and falling out of teeth
E. separated and plural erosions of enamels are the different shape, they are
colourless or pigmented. Teeth are fragile and destroy easily
ANSWER: E
What is the denticles of tooth?
A. the granular formings are in the cavity of tooth
B. the granular stratifications are in the area of dental alveolus
C. salts crystals are in the area of neck of tooth
D. the structures which are like to the tooth dentine and are disposed on the
external surface of enamel or cement of tooth
E. the round or oval formations which are built by amorphous salts or are
like to the structure of recovery dentine and are disposed in pulp or dentine
of tooth
ANSWER: E
Name the types of chronic periodontitis?
A. serosal and festering
B. typical and atypical
C. protract and progressive
D. atrophy and resorbtic, hypertrophy and hyperplastic
E. granular, granulomatosal and fibrotic
ANSWER: E
How many degrees does fluorosis development subdivide?
A. 3
B. 2
C. 5
D. there is no degree division
E. 4
ANSWER: E
What is the etiologic factor of caries development?
A. viruses
B. the simplest
C. chemical damages of teeth
D. physical damages of teeth
E. bacteria
ANSWER: E
How is a caries classified after motion?
A. sharp and chronic
B. repeated and recedivic
C. superficial and deep
D. turning and irreversible
E. fast and slow
ANSWER: E
818.
What is the typical deep caries?
A. there is destroyed 1/3 teeth
B. there is destroyed the superior or lateral surface of crown enamel
C. a carious cavity spread into the root of tooth
D. all tooth is destroyed
E. the forming of large cavity in a dentine, It occupies in pulp
ANSWER: E
819.
What is the early caries?
A. the caries develops in baby teeth
B. the caries develops shortly after cuting of the second teeth
C. the caries develops shortly after cuting of baby teeth
D. develops at 7 - 10-years-old children
E. develops directly under the layer of enamel
ANSWER: E
820.
What is the favourable background of development of wedge-shaped
defects of tooth?
A. disease of bone sceleton
B. disease of mouth cavity
C. disease of salivary glands
D. nothing among the above-mentioned
E. disease of gastroenteric tract
ANSWER: E
821.
How name of the 1th degree of fluorosis development?
A. weak defeat
B. the defeat is moderato expressed
C. heavy defeat
D. stage of complications
E. very weak defeat
ANSWER: E
822.
What is the reason of development of tooth erosion?
A. the mechanical damage at mastication
B. the considerable thermal overfalls
C. the changes of chemical composition of saliva
D. the professional harmful
E. the reason is unknown
ANSWER: E
823.
What types of denticles do you know?
A. undifferentiated and amorphous denticles
B. simple and complex denticles
C. highly mineralized dense and lowly mineralized soft denticles
D. structured denticles and unstructured
E. highly developed and lowly developed denticles
ANSWER: E
824.
To which group of diseases does epulis belong?
A. granulomatous diseases of soft tissues of tooth
825.
826.
827.
828.
829.
830.
B. inflammatory processes of gums
C. tumular processes of muscular tissue of cheeks or tongue
D. the granulomatous formings on bone tissue of jaw
E. tumular processes of gums
ANSWER: E
Characterize the 1st degree of fluorosis development?
A. chalky spots occupy near the half of area the crowns of the teeth, there
are separate pigmented spots in an enamel
B. chalky spots occupy more than the half of surface of teeth and meet
between itself
C. separate and plural erosions of the enamels have the different forms.
They are colourless or pigmented. The teeth are fragile and collapse easily
D. dicking and fall out of teeth
E. chalky spots are small and poorly expressed. They occupy 1/3 labial and
tongue surface of teeth
ANSWER: E
The caries of which parts of tooth develops more quickly?
A. pulp
B. cement
C. enamel
D. develops with the same speed
E. dentine
ANSWER: E
What is the first display of caries development?
A. appearance of black spot on the enamel of tooth
B. appearance of brown spot on the enamel of tooth
C. appearance of the radially oriented cracks
D. appearance of a little black erosion on an enamel
E. appearance of white opaque spot on the enamel of tooth
ANSWER: E
What teeth are defeated by a caries more frequently than other?
A. lower premolar
B. superior premolar
C. lower first molar
D. there is no set conformity to the law
E. superior first molar
ANSWER: E
What teeth are more frequently defeated by wedge-shaped defects?
A. medial incisiva-teeth
B. lateral incisiva-teeth
C. molar
D. all teeth identically often
E. canina-teeth and premolars
ANSWER: E
Why does a fluorosis develop?
A. because to the genetic metabolic disturbances
B. because to professional harmful influences into an organism
831.
832.
833.
834.
835.
C. because to violation of mineral exchange
D. because to the insufficient entering of fluorine in organism
E. because to the enhanceable entering of fluorine in organism
ANSWER: E
How is the name of 3d degree of fluorosis development?
A. weak defeat
B. very weak
C. heavy defeat
D. stage of complications
E. moderato expressed defeat
ANSWER: E
Give the definition of teeth erosion?
A. diminishing of mineralization of teeth enamel with development of
unache defect
B. demineralizing of noncommunicative areas of enamel of one or two
molars
C. forming of pigmented defects on the labial surface of incisiv-teeth,
which can precede to development of caries
D. the progressiv forming of pigmented cracks on the masticatory surface
of molars
E. the progressiv like cup form defects with diminishing of enamel and
dentine which localized on the vestibular surface of teeth and have ache
character
ANSWER: E
Where are the interstitial denticles localized?
A. in pulp
B. on a border between a dentine and enamel of tooth
C. near the tooth root
D. between the jaw spongy tissue
E. in the deep of dentine
ANSWER: E
What is the basic display of paradontosis development?
A. an inflammatory process in gums with a pain symptom
B. an inflammatory process in gums without a pain symptom
C. an inflammatory process in tissues of paradontium
D. inflammatory process in a paradontium with increasing of vascularizing
and hyperplasia of surround tissues
E. retraction of gums with baring of tooth neck
ANSWER: E
What name of the 2th degree of fluorosis development?
A. very weak defeat
B. the defeat is moderato expressed
C. heavy defeat
D. stage of complications
E. weak defeat
ANSWER: E
836.
At a parathyreoid osteodystrophy in the maxillufacial bones to the
skeleton is observed:
A. specific inflammatory process
B. heterospecific inflammatory process
C. sequestra formation
D. hyperostosis
E. lacunar resorbtion and formation of fibrotic tissue
ANSWER: E
837.
The benign unodontogenic tumor of jaw is:
A. ameloblastoma
B. odontoameloblastoma
C. osteosarcoma
D. all are enumerated
E. osteoblastoclastoma
ANSWER: E
838.
The cells which form the parenchima of osteoblastoclastomas are
belonging to:
A. odontogenic ectodermic origin
B. odontogenic mesenchimal origin
C. melaninproductors
D. ephithelial
E. osteogenic
ANSWER: E
839.
A malignant lymphadenoma with typical localization in jaws bones is:
A. a Abrikosov's tumor
B. a Willms' tumor
C. a Sezary tumor
D. a Khodjkin's tumor
E. a Bercit's tumor
ANSWER: E
840.
What structure of teeth has the ectodermal origin?
A. dentine
B. odontoblasts
C. pulp
D. cement
E. enamel
ANSWER: E
841.
The benign odontogenic tumor of ectodermal origin is:
A. cementoma
B. odontogenic carcinoma
C. osteoblastoclastoma
D. dentinoma
E. ameloblastoma
ANSWER: E
842.
According to the clinic-morphological features the ameloblastomas are:
A. benign tumor
B. malignanted benign tumor
C. malignant tumor
D. a tumular process
E. a tumor with local destructive growth
ANSWER: E
843.
For an ameloblastoma the typically is:
A. metastasis by a haematogenic way
B. metastasis by a lymphogenic way
C. quick growth
D. more often is localized in a maxilla
E. numerous niduses of bone destruction
ANSWER: E
844.
Histological form of ameloblastoma, which meets more often than other:
A. akantomatosal
B. basalcell
C. roundcell
D. granularcell
E. follicle
ANSWER: E
845.
The clinic-anatomical forms of ameloblastoma is:
A. odontogenic and osteogenic
B. ectodermal and mesenchimal
C. smallnidus and diffuse
D. acute and recidivic
E. cystophorous and solid
ANSWER: E
846.
What form of ameloblastoma according to a histological structure
reminds an enamel organ?
A. plexiform
B. akantomatosal
C. basalcell
D. granularcell
E. follicle
ANSWER: E
847.
What name of the tumor, which is introduced by the trabecular net of
odontogenic epithelium with whimsical branching?
A. a follicle form of ameloblastoma
B. an akantomatic form of ameloblastoma
C. a basalcell form of ameloblastoma
D. a granularcell form of ameloblastoma
E. a plexiform form of ameloblastoma
ANSWER: E
848.
For what form of ameloblastoma the typically is epidermoid cell
metaplasia with keratin formation?
A. follicle
B. plexiform
C. basalcell
D. granularcell
E. akantomatic
ANSWER: E
849.
What form of ameloblastoma according to a histological structure
reminds a basalcell cancer?
A. follicle
B. plexiform
C. akantomatic
D. granularcell
E. basalcell
ANSWER: E
850.
The odontogenic epithelium of ameloblastoma contains the numerous
acidophilic granules. This is a:
A. malignant ameloblastoma
B. plexiform form of ameloblastoma
C. akantomatic form of ameloblastoma
D. basalcell form of ameloblastoma
E. granularcell form of ameloblastoma
ANSWER: E
851.
For a adenomatoid tumor typically is:
A. atypizm and polymorphism of odontogenic epithelium are expressed
B. epidermoid metaplasia of odontogenic epithelium
C. metastasis in the regional lymphatic nodes
D. all that are enumerated
E. a odontogenic epithelium forms structures which remind the channels
ANSWER: E
852.
Adenomatic tumor arises up more often:
A. intra-uterine
B. at infants and children of the first year-old
C. in the people of mature age
D. in the old people
E. in the second decade-period of life
ANSWER: E
853.
What tumors are belonging to odontogenic carcinomas?
A. basalcell and acidophilic forms of ameloblastomas
B. malignant ameloblastoma and primary intracyst carcinoma
ANSWER: B
854.
What tumors are belonging to odontogenic carcinomas?
A. osteoblastoclastoma and osteosarcoma
B. basalcell and acidophilic forms of ameloblastomas
C. ameloblastic fibroma and odontoameloblastoma
D. all that are transferred
E. malignant ameloblastoma and primary intracyst carcinoma
ANSWER: E
855.
The primary intracyst carcinoma develops from:
A. the epithelium of mucus membrane of mouth cavity
B. the odontogenic mesenchyma
C. the bone tissue
D. can develop from all enumerated structures
E. the epithelium of Malasse's islets
ANSWER: E
856.
The primary intracyst carcinoma can develop from:
A. the epithelium of mucus membrane of mouth cavity
B. the odontogenic mesenchyma
C. the bone tissue
D. can develop from all enumerated structures
E. the epithelium of disontogenetic odontogenic cysts
ANSWER: E
857.
The benign odontogenic tumors of mesenchyma origin are:
A. ameloblastoma, adenomatic tumor
B. odontogenic carcinoma, osteoblastoclastoma
C. ameloblastic fibroma, odontoameloblastoma
D. all that are enumerated
E. dentinoma, myxoma, cementoma
ANSWER: E
858.
What tumors does not belong to the odontogenic tumors of mesenchymal
origin?
A. dentinoma
B. odontogenic myxoma
C. benign cementoma
D. giant cementoma
E. odontoameloblastoma
ANSWER: E
859.
The typically difference of odontogenic myxoma from the myxomas of
other localization is next:
A. the base of tumor is formed by mucous mass
B. contains the a lot of acid mucopolisaccharides
C. a tumor does not metastases
D. a tumor is predispositioned to the relapses (recedives) after the surgical
delete
E. contains the trabeculs of inactive odontogenic epithelium
ANSWER: E
860.
At histological research of dentinoma it is possible to discover:
A. trabeculs of inactive odontogenic epithelium
B. immature connecting tissue
C. the islets of dysplastic dentine
D. nothing from enumerated
E. all structures that are marked
ANSWER: E
861.
The pathognomic sign of cemenomas is:
A. a presence of young or mature fibrotic tissue
B. a presence of trabeculs of inactive odontogenic epithelium
C. a presence of trabeculs of odontogenic epithelium with the signs of
polymorphism and atypizm
D. it contains the islets of dysplastic dentine
E. a formation of cementoid matter with the different degree of
mineralization
ANSWER: E
862.
What histological types of cementom do you know?
A. benign cementoblastoma, dentinoma, ameloblastoma
B. benign cementoblastoma, cementitious fibroma, odontoameloblastoma
C. cementitious fibroma, giganticcells tumor|
D. all that are enumerated
E. benign cementoblastoma, cementitious fibroma, giant cementoma
ANSWER: E
863.
Accrding to etiology giant cementoma is a:
A. result of trauma
B. result of specific inflammatory process
C. complication of visceral mycosis
D. manifestation of chronic radiation illness
E. inherited disease
ANSWER: E
864.
What tumor does not belong to the odontogenic tumor s of the mixed
origin?
A. ameloblastic fibroma
B. odontoameloblastoma
C. ameloblastic fibroodontoma
D. odontogenic fibroma
E. cementitious fibroma
ANSWER: E
865.
The ameloblastic fibroma is characterized by:
A. contains the islets of proliferativ odontogenic epithelium
B. contains the fluffy tissue which reminds the tissue of dental papilla
C. typical localization is the area of premolar
D. developsmore more frequently in in child's and young age
E. all that are enumerated
ANSWER: E
866.
The odontogenic fibroma is characterized by:
A. contains the islets of inactive odontogenic epithelium
B. contains the mature connecting tissue
C. more frequently meets in senior age-dependent groups
D. all that are enumerated is incorrectly
E. all that are enumerated is right
ANSWER: E
867.
The inflammatory diseases of jaws are:
A. osteomielitis, osteoma
B. cyst,osteoblastoclastoma
C. periostitis, paratireoid osteodystrophy
D. all that are enumerated disease
E. periostitis, osteomielitis
ANSWER: E
868.
Periostitis is :
869.
870.
is:
A. infectiously-allergic process which develops in a bone
B. festering-necrotizing process which develops in a bone
C. inflammatory process which will strike tissues of parodont and spreads
on bones structures adjoining to him
D. a tumular process is in bone
E. an inflammatory process in a periosteum
ANSWER: E
Character of inflammation acute periostitis is:
A. fibrotic
B. alterativ, productive
C. destructive, hyperplastic
D. primary, recidive
E. serosal, festering
ANSWER: E
For acute serosal periostitis the followings changes are typically, except
A. hyperemia
B. inflammatory edema
C. moderate neutrophilic infiltration of periosteum
D. posttraumatic
E. resorbtion of bone
ANSWER: E
871.
"Serosal periostitis" is the synonym of diagnosis:
A. productive periostitis
B. alterative periostitis
C. festering periostitis
D. fibrotic periostitis
E. simple periostitis
ANSWER: E
872.
Removing a layer by the layer of periosteum with the accumulation of
pus between him and bone at festering periostitis is this:
A. acute osteomielitis
B. chronic osteomyelitis
C. cyst
D. granulim
E. subperiosteal abscess
ANSWER: E
873.
What term is the synonym of diagnosis "osteomielitis of jaw"?
A. phlegmona of bone
B. gaversitis
C. osteitis
D. panostitis
E. osteoblastoclastoma
ANSWER: E
874.
Osteomielitis more frequent develops:
A. in a maxilla
B. frequency of defeat is identical
C. at the same time in the maxilla and mandibula
D. in a maxilla osteomielit arises up never
E. in a mandibula
ANSWER: E
875.
What pathomorphological change characterizes odontogenic
osteomyelitis most full?
A. inflammation of peridont
B. inflammation and niduses of peridont destruction
C. inflammation and destruction of peridont, festering-inflammatory
process in a periosteum
D. festering-inflammatory process in a jaw and surrounding tissues
E. festering infiltration of marrow, thrombosis of vessels, festering melting
of trombs, area of hemorrhage and osteonecrosis
ANSWER: E
876.
What changes are present in bone tissue at acute osteomyelitis:
A. excrescence of granulation tissue
B. sclerosis
C. regeneration of bones trabecules
D. tissue atipism
E. resorbtion of bones trabecules
ANSWER: E
877.
What from enumerated does not meet at acute osteomielitis?
A. necrosis of bone tissue
B. neutrophilic infiltration
C. phlegmonous character of inflammation
D. thrombosis of vessels
E. sclerosis of bone tissue
ANSWER: E
878.
The wall of sequestral cavity consists of:
A. unchanged bone tissue
B. granulation tissue
C. connective tissue capsule and ephithelial layer
D. all enumerated variants are possible
E. connective tissue capsule and pyogenic membrane
ANSWER: E
879.
The capsule of sequestral cavity is formed by:
A. the unchanged bone tissue
B. necrotic bone tissue
C. granular tissue
D. cartilaginous tissue
E. fibred connecting tissue
ANSWER: E
880.
At the congenital form of uncompleted osteogenesis in bones of
maxillufacial skeleton is observed:
A. the deficit of osteocytes is expressed
B. specific inflammatory process
C. aseptic necrosis of bone tissue
D. substituting of bone tissue by a cartilage
E. normal amount of osteocytes and deficiti of basic matter in bones
trabecules
ANSWER: E
881.
At a paratireoid osteodystrophy in the bones of maxillufacial skeleton is
observed:
A. specific inflammatory process
B. unspecific (heterospecific) inflammatory process
C. sequestr formation
D. hyperostosis
E. lacunar resorbtion and formation of fibrotic tissue
ANSWER: E
882.
At the histological research of jaw periosteum thay found inflammatory
hyperemia, swollen, moderate infiltration by neutrophilic leucocytes.
Inflammation developed after a trauma. What process is it?
A. Festering periostitis
B. Fibrotic periostitis
C. osteomielitis
D. Secondary amyloidosis
E. Serosic periostitis
ANSWER: E
883.
Osteomielitis is the:
A. Festering inflammation of mucus membrane
B. Odontogenic antritis (Himoritis)
C. Odontogenic sepsis
D. Secondary amyloidosis
E. Marrow inflamation of jaws bones
ANSWER: E
884.
Premordial cyst develops more often in …
A. In the area of the first molar
B. in the area of the second molar
C. in the area of the first premolar
D. in the area of the second premolar
E. in the area of the third molar
ANSWER: E
885.
The internal surface of wall of premordial cyst is covered:
A. By mature connecting tissue
B. By a ferrous epithelium
C. By a transitional epithelium
D. By fluffy connecting tissue
E. By a multi-layered flat epithelium
ANSWER: E
886.
A follicle cyst develops from:
A. Granulation tissue
B. Fibrotic tissue
C. Inflammatory infitration
D. From the wall of chronic abscess
887.
888.
889.
890.
891.
892.
893.
E. enamel organ of tooth which was not cut
ANSWER: E
A follicle cyst more frequently is combined:
A. with the first premolar
B. with the first molar
C. with the second molar
D. with the lower incisiva-tooth
E. with the second premolar
ANSWER: E
A follicle cyst more frequently is combined:
A. with the first premolar
B. with the first molar
C. with the second molar
D. with the lower incisiva-tooth
E. with the third molar
ANSWER: E
A follicle cyst more frequently is combined:
A. with the first premolar
B. with the first molar
C. with the second molar
D. with the lower incisiva-tooth
E. with the lower canina-tooth
ANSWER: E
It is possible to see in the internal wall of follicle cyst:
A. atypic cells
B. osteoblasts
C. Ferrous cells
D. All that are enumerated cells
E. mucus productings cells
ANSWER: E
The radicular cyst meets:
A. In 20% from all cases
B. In 40% from all cases
C. In 50% from all cases
D. In 60% from all cases
E. In 80% from all cases
ANSWER: E
The radicular cyst develops as a ressult of:
A. acute abscess
B. chronic abscess
C. acute periodontitis
D. Follicle cyst
E. Chronic periodontitis
ANSWER: E
The radicular cyst develops:
A. From a follicle cyst
B. From acute periodontitis
894.
895.
896.
897.
898.
899.
900.
C. From an acute abscess
D. From a chronic abscess
E. from a complex granulomas|
ANSWER: E
The internal surface of radicular cyst wall is covered:
A. By a ferrous epithelium
B. By a transitional epithelium
C. By a multi-layered flat carotinized epithelium
D. By fibrotic tissue
E. By a multi-layered flat uncarotinized epithelium
ANSWER: E
At acuteinig of inflammation in the wall of radicular cyst develops:
A. Abscess
B. Granulosum
C. Necrosis
D. dysplasia of epithelium
E. Acanthosis of covering epithelium
ANSWER: E
At inflammation the radicular cyst cavity can be filling:
A. By fibrotic tissue
B. by fibrinosic exsudate
C. By necrosis
D. By a festering exsudate
E. By granulation tissue
ANSWER: E
Before the innate diseases of salivary glands belongs:
A. Sialoadenit
B. Salivary stoun
C. Kheylit
D. Parotitis
E. Ageneziya
ANSWER: E
Parotitis – it:
A. Inflammation of salivary gland
B. Inflammation of sublingual salivary gland
C. Inflammation of palatal salivary gland
D. Inflammation of all of salivary glands
E. Inflammation of parotid salivary gland
ANSWER: E
Most frequent reason of primary sialoadenitis:
A. A virus is Epstain- Bar
B. Streptococcus
C. Staphylococcus
D. sort of Candida
E. Virus of epidemic parotitis
ANSWER: E
That is reason of sialodenit of uninfectious origin:
A. Tumours of cavity of mouth
B. sort of Candida
C. Poisoning of mushrooms
D. Alcoholic intoxication
E. Poisoning salts of heavy metal
ANSWER: E
901.
Most frequent reason of primary sialodenitis:
A. A virus is Epshtain -Bar
B. Streptococcus
C. Staphylococcus
D. sort of Candida
E. Citomegalovirus
ANSWER: E
902.
The second defeat of salivary and tear glands is combined it is known as:
A. Syndrome of Al'porta
B. Syndrome of Patau
C. By cider of Zhil'bera
D. Syndrome of Kartagenera
E. Syndrome of Mikulich
ANSWER: E
903.
Name the innate disease of salivary glands.
A. sialodenit
B. sialolithiasis
C. oncocitoma
D. all are marked
E. aectopy
ANSWER: E
904.
Sialoadenit - is:
A. ephithelial tumour of salivary gland
B. inflammation of sublingual salivary gland
C. inflammation of sublingual salivary gland
D. inflammation of parotid salivary gland
E. inflammation be what salivary glands
ANSWER: E
905.
Parotitis – it:
A. inflammation be what salivary gland
B. inflammation of sublingual salivary gland
C. inflammation of sublingual salivary gland
D. ephithelial tumour be what salivary gland
E. inflammation of parotid salivary gland
ANSWER: E
906.
What from the transferred morphological changes do not meet at
epidemic parotitis?
A. it was swollen the interstitium of parotid glands
B. lymphoid infiltrations round channels and аcinus
C. a plethora is expressed
D. secret is in channels
E. diffuse neutrophilic snfiltration
ANSWER: E
907.
Name complication of epidemic parotitis.
A. meningitis
B. meningiencephalitis
C. orchitis
D. pancreatitis
E. all
ANSWER: E
908.
At what infection of еpiteliocites channels of salivary glands does
acquire the type of "owl eye"?
A. epidemic parotitis
B. windy pox
C. orchitis
D. german measles
E. citomegalia
ANSWER: E
909.
What organs are more frequent all struck at the noncommunicative form
of cytomegaly?
A. liver, buds
B. buds, pancreas
C. lymph organs
D. submandibular salivary glands
E. parotid salivary glands
ANSWER: E
910.
What group does the exciter of citomegalic parotitis belong to?
A. gram-positive bacteria
B. gram-negative bacteria
C. the simplest
D. RNA-virus
E. DNA-virus
ANSWER: E
911.
Is there what way of penetration of infections in a salivary gland at
cialoadenitis?
A. stomatogenic
B. gematogenic
C. lymphogenic
D. pin
E. all
ANSWER: E
912.
What factor is instrumental in formation of salivary stone?
A. dyskinesia of channels
B. cialodochitis
C. increase of pH saliva
D. a hit of extraneous body is in a channel
E. all
ANSWER: E
913.
What compounds do form the bulk of salivary stone?
A. salts of urinary acid
B. cholesterol and salts of bilious acids
C. chloride and sulfate of sodium
D. mucopolysacharids
E. phosphate and carbonate of Ca
ANSWER: E
914.
What from transferred does not behave to complications of sialolithiasis?
A. cialodochitis
B. festering cialodenitis
C. sclerosis of salivary gland
D. sialocele
E. ectopia of salivary gland
ANSWER: E
915.
What from the diseases transferred below belongs to precancer:
A. inflammation of lips
B. Granulematois of cheylitis
C. Eksfoliativ of cheylitis
D. Glandular cheylit
E. cheylit Manganotti
ANSWER: E
916.
That from below transferred behaves to the tubular diseases of salivary
glands:
A. Sialoadenit
B. Ectopia of salivary glands
C. Stoun illness
D. Pleomorfic adenoma
E. Sialozis
ANSWER: E
917.
Most dangerous complication of furuncle of lip:
A. Stroke
B. Limfadenitis
C. Limfangoitis
D. Dermatitis
E. Thrombophlebitis of angular vine
ANSWER: E
918.
How is a sialocele, in education of which there is mucus named?
A. retential cyst
B. varicocele
C. mucoepidermoid tumour
D. оncocytoma
E. mucocele
ANSWER: E
919.
What tumour of salivary glands does meet more frequent all?
A. оncocytoma
B. аcynocells tumour
C. mucoepidermoid tumour
D. carcinoma
E. pleomorfic adenoma
ANSWER: E
920.
Tumours of which histogenic groups do meet in salivary glands most
frequent ?
A. mesenchimal
B. teratomas
C. tumours of меlaninforming cells
D. none of the transferred groups
E. Epithelial
ANSWER: E
921.
What from transferred does not behave to the ephithelial tumours of
salivary glands?
A. Aciniforming tumour
B. carcinoma
C. monoforming adenoma
D. oncocitoma
E. mucocele
ANSWER: E
922.
Pleomorfsc adenoma of salivary glands more frequent all meets in:
A. children of first-year of life
B. rof men under 20 years
C. for men under age 30
D. for old people
E. for women after 40 years
ANSWER: E
923.
Most frequent localization of plepomorphyc adenoma:
A. small salivary glands of oral cavity
B. sublingual gland
C. submandibular gland
D. a tumour meets approximately with identical frequency in all of glands
E. parotid glands
ANSWER: E
924.
What most widespread localization of tumours of salivary glands?
A. small salivary glands of oral cavity
B. sublingual gland
C. submandibular gland
D. tumours meets approximately with identical frequency in all of glands
E. parotid gland
ANSWER: E
925.
How macroscopically does the pleomorphyc adenoma of salivary gland
look on a cut?
A. reminds fish meat
B. hollow education, surrounded the thin-walled capsule, is muciferous
C. a greyish knot is with numerous hemorrhages and necrotic areas
D. all are possible variants are marked (depending on the stage of
development of tumour)
E. gum-blush, with shallow cists
ANSWER: E
926.
For the histological structure of pleomorpyc adenoma characteristic is:
A. there are ephithelial cages of different form
B. tumour cages form structures which remind channels
C. there are the fields of mucoid membrane
D. all is transferred
E. the local hyalinosis of stroma is possible
ANSWER: E
927.
What assertion in relation to the pleomorfic adenoma of salivary gland is
incorrect?
A. grows slowly
B. more frequent meets for women
C. tumour cages form structures which remind channels
D. a hyperkeratinization is possible in the ephithelial areas of tumour
E. more frequent all localized in submandibular glands
ANSWER: E
928.
Most frequent localization of monomorfic adenoma:
A. small salivary glands of oral cavity
B. sublingual gland
C. submandibular gland
D. a tumour meets approximately with identical frequency in all of glands
E. parotid gland
ANSWER: E
929.
Is there what main difference of monomorfic adenoma of salivary gland
from pleomorfic?
A. has an ephithelial origin
B. it is of high quality
C. slow growth
D. more frequent all localized in a parotid gland
E. same of histological structure within the limits of one tumor
ANSWER: E
930.
The synonym of diagnosis "оncocitoma" is:
A. pleoformic adenoma
B. adenolimfoma
C. mucoepidermoid tumor
D. acinocellular tumour
E. oxifilic adenoma
ANSWER: E
931.
Most frequent localization of adenolimphomas:
A. small salivary glands of oral cavity
B. sublingual gland
C. submandibular gland
D. a tumour meets approximately with identical frequency in all of glands
E. parotid gland
ANSWER: E
932.
The tumour of salivary gland is built from a prismatic epithelium from
sharply by a aeosinophylyc cytoplasm. The expressed lymphocitic infiltration
of stroma of tumour is marked. Specify the most credible diagnosis.
A. Oxifilic adenoma
B. Basalcells monomorfic adenoma
C. Small cells monomorfic adenoma
D. pleomorfic adenoma
E. Adenolymhosarcoma
ANSWER: E
933.
What from transferred is not characteristic for adenolymphoma?
A. built from a prismatic epithelium
B. the epithelium of tumour takes a place in two rows
C. the epithelium of tumour forms nipple structures
D. lymphocitic infiltration of stroma of tumour
E. cytoplasm of tumour cages sharply basophilic
ANSWER: E
934.
The tumour of salivary gland is presented combination of cages of
epidermoid of type and gravitated from mucoidprodaction cages, stroma is
expressed well. Specify the most credible diagnosis.
A. basalcalls моnomorphyc adenoma
B. smallcells monomorphyc adenoma
C. oncocitoma
D. adenolimphoma
E. mucoepidermoid tumor
ANSWER: E
935.
Most frequent localization of карциноми of salivary glands:
A. sublingual gland
B. submandidular gland
C. parotid gland
D. a tumour meets approximately with identical frequency in all of glands
E. small salivary glands of hard
ANSWER: E
936.
What malignant tumour of salivary glands does meet more frequent all?
A. oncocitoma
B. adenolymphoma
C. pleomorfic adenoma
D. mucoepidermoid tumour
E. Adenocist carcinoma
ANSWER: E
937.
What pathological processes are underlaid diseases of soft fabrics of oral
cavity?
A. dystrophic
B. disorders of circulation of blood
C. inflammatory
D. tumour
E. all
ANSWER: E
Glossitis – it:
A. inflammation of lips
B. inflammation of mucus shell of mouth
C. inflammation of salivary gland
D. inflammation of bone
E. inflammation of tongue
ANSWER: E
939.
What is stomatitis?
A. inflammation of lips
B. glossitis
C. inflammation of salivary gland
D. inflammation of bone fabric of jaw
E. inflammation of mucus shell of mouth
ANSWER: E
940.
Name a sharp infectious viral disease for which typical is an
inflammatory process in stroma of parotid salivary glands.
A. mesesels
B. german measles
C. parainfluenza
D. illness of Filatova
E. epidemic paropitis
ANSWER: E
941.
What assertion in relation to epidemic parotitis is incorrect?
A. an entrance gate is a mucus shell of mouth, nose
B. an edema and hyperemia registers in the interstitium of salivary glands
C. there is lymphoid infiltration of stroma of salivary glands
D. proof immunity a stay after illness
E. DNA-virus
ANSWER: E
942.
Describe an inflammatory process at epidemic parotitis.
A. gangrenous parotitis
B. sharp local festering parotitis
C. sharp diffuse festering parotitis
D. productive parotitis
E. interstitial parotitis
ANSWER: E
943.
Chronic sialoadenit after character inflammation most frequent is:
A. alterativ
B. catarrhal
C. phlegmonous
D. gangrenous
E. productive
ANSWER: E
944.
What disease is sialoadenitis at with destruction of salivary glands
lymphocites and macrophages?
A. syndrome of Daun
B. syndrome of Patau
938.
C. syndrome of Shereshevsky-Terner
D. syndrome of Shteyl-Levental
E. syndrome of Shegren
ANSWER: E
945.
Name complication of sialoadenit.
A. sclerosis of gland
B. atrophy of acinus
C. lipomatosis of stroma
D. xerostomia
E. all
ANSWER: E
946.
How is a disease at which concrements appear in the channels of salivary
gland named?
A. oncocitoma
B. parotitis
C. sialoadenitis
D. sialocirros
E. sialolitiasis
ANSWER: E
947.
The cell types in the corpus gastric glands are all of the following,
EXCEPT:
A. Mucous cells
B. Parietal cells
C. Chief cells
D. Endocrine cells
E. Hurtle cells
ANSWER: E
948.
The gastric mucosal protection from autodigestion is possible due to all
of the following factors, EXCEPT:
A. Mucus secretion
B. Bicarbonate secretion
C. Epithelial barrier
D. Mucosal blood flow
E. Gastrin receptor activation
ANSWER: E
949.
Chronic infection of the gastric mucosa is associated with which of the
following bactertia:
A. Escherichia coli
B. Campilobacter jejuni
C. Enterococcus falcium
D. Escherichia chaffeensis
E. Helicobacter pylori
ANSWER: E
950.
Chronic gastritis may be characterized by all of the following features,
EXCEPT:
A. Lymphocyte infiltration
B. Plasma cell infiltration
C. Intestinal metaplasia
D. Atrophy
E. Suppurative inflammation
ANSWER: E
951.
Acute gastritis is commonly caused by all of the following factors,
EXCEPT:
A. Excessive use of aspirin
B. Heavy smoking
C. Excessive alcohol consumption
D. Treatment with cancer chemotherapeutic drugs
E. Genetic abnormalities
ANSWER: E
952.
Acute gastritis is commonly associated with all of the following factors,
EXCEPT:
A. Uremia
B. Severe stress
C. Systemic infections
D. Ischemia and shock
E. Delayed gastric emptying
ANSWER: E
953.
Acute catarrhal gastritis is morphologically characterized by aU of the
following features, EXCEPT:
A. Moderate edema of lamina propria
B. Vascular congestion of lamina propria
C. Neutrophil infiltration
D. Abundant mucus amount on the gastric epithelium
E. Atrophy of the mucosa
ANSWER: E
954.
The major etiologic associations of chronic gastritis are all of the
following, EXCEPT:
A. Chronic Helicobacter pylori infection
B. Autommune factors
C. Toxic factors
D. Bile reflux
E. Ischemia and shock
ANSWER: E
955.
The major etiologic associations of chronic gastritis are all of the
following, EXCEPT:
A. Helicobacter pylori infection
B. Radiation injury
C. Granulomatous diseases
D. Autoimmune reactions
E. Salmonella infections
ANSWER: E
956.
Special forms of gastritis are all of the following, EXCEPT:
A. Eosinophilic gastritis
B. Allergic gastroenteropathy
C. Lymphocytic gastritis
D. Granulomatous gastritis
E. Interstitial gastritis
ANSWER: E
957.
The clinical syndrome associated with gastric, duodenal and jejunal
peptic gastrin-induced ulcers development is which of the following:
A. Horner syndrome
B. Nephrotic syndrome
C. DIG syndrome
D. Malabsorption syndrome
E. Zollinger- Ellison syndrome
ANSWER: E
958.
Morphologically, all of the following zones can be found in the active
ulcer base, EXCEPT:
A. Necrotic debris
B. Nonspecific acute inflammation
C. Granulation tissue
D. Fibrosis
E. Granulomatous tissue
ANSWER: E
959.
The appendix in acute appendicitis is grossly characterized by all of the
following features, EXCEPT:
A. Enlarged with thickened walls
B. Swollen
C. Hyperemic with engorged vessels
D. With fibrin covered serosa
E. Firm and indurated
ANSWER: E
960.
Catarrhal gastritis is grossly characterized by all of the following
features, EXCEPT:
A. Thickened gastric walls
B. Swollen gastric mucosa
C. Pinpoint hemorrhages on the mucosa
D. Abundant viscuos mucus on the mucosa
E. Multiple erosions on the mucosa
ANSWER: E
961.
All of the following complications may be found in chronic peptic ulcer,
EXCEPT:
A. Perforation
B. Bleeding
C. Malformation
D. Penetration
E. Caseation
ANSWER: E
962.
The causes of malabsorption syndrome are all of the following,
EXCEPT:
A. Defective intraluminal digestion
B. Primary mucosal cell abnormalities
C. Reduced surface of small intestine
D. Lymphatic obstruction
E. Small intestine diverticula
ANSWER: E
963.
Pathologic changes of the intestinal wall in Crohn's disease include all of
the following, EXCEPT:
A. Granuloma formation
B. Areas of chronic inflammation with vasculitis
C. Segmental fibrosis
D. Slit ulcers
E. Pseudomembranous colitis
ANSWER: E
964.
The predisposing conditions for ischemic bowel disease are all of the
following, EXCEPT:
A. Arteriolar thrombosis
B. Arteriolar embolism
C. Venous thrombosis
D. Portal hypertension syndrome
E. Helicobacter pylori infection
ANSWER: E
965.
Giant cerebriform enlargement of the gastric mucosa in Menetrier
disease is caused by which of the following pathologic processes:
A. Inflammation
B. Atrophy of the mucosa
C. Interstitial metaplasia
D. Fibrosis
E. Hyperplasia of the mucosal epithelial cells
ANSWER: E
966.
Complications of the duodenal peptic ulcer disease include all of the
following, EXCEPT:
A. Bleeding
B. Perforation
C. Penetration
D. Obstruction by edema or scarring
E. Malabsorption
ANSWER: E
967.
Cancerous ulcer margins are grossly characterized by all of the following
features, EXCEPT:
A. Greyish-white
B. Elevated
C. Firm
D. Irregular and thickened .
E. Relatively straight walls
ANSWER: E
968.
The most common localization of peptic ulcer is which of the following:
A. Greater curvature
B. Lesser curvature
C. Anterior wall of the gastric corpus
D. Posterior wall of the gastric corpus
E. Duodenal pyloric ring
ANSWER: E
969.
The classic peptic ulcer is grossly characterized by all of the following
features. EXCEPT:
A. Round-to-oval
B. Sharply defined
C. Punchet-out defect
D. Elevated margins
E. Superficial defect
ANSWER: E
970.
The inflammatory infiltrate in the remission stage of chronic gastritis
may consist of all of the following cells, EXCEPT:
A. Lymphocytes
B. Plasma cells
C. Fibroblasts
D. Macrophages
E. Neutrophils
ANSWER: E
971.
The granulation tissue in the active ulcer base is infiltrated
predominantly by all of the following cells, EXCEPT:
A. Neutrophils
B. Lymphocytes
C. Plasma cells
D. Macrophages
E. Giant cells
ANSWER: E
972.
The most common pathologic change of uninvolved gastrointestinal tract
in peptic ulcer patients is which of the following:
A. Hypertrophic gastropathy
B. Gastric dilatation
C. Menetrier disease
D. Chronic gastritis
ANSWER: D
973.
Bleeding from peptic ulcer is characterized by all of the following,
EXCEPT:
A. Occurs in 15-20% of patients
B. Is most frequent complication of peptic ulcer
C. May be life-threatening
D. Causes 25% of ulcer deaths
E. Causes chronic renal failure
ANSWER: E
974.
Gastrointestinal tract obstruction from edema or scarring in peptic ulcer
disease is characterized by all of the following, EXCEPT:
A. Occurs in 2% of patients
B. Often occurs in pyloric channel ulcer
C. May occur in duodenal ulcer
D. Causes incapacitating, crampy abdominal pain
E. May occur in the stomach corpus
ANSWER: E
975.
Multiple superficial lesions located in the gastric mucosa are called:
A. Peptic ulcers
B. Acute stress ulcers
C. Gastric ulcerations
D. Active ulcers
E. Erosions
ANSWER: E
976.
Point out the hepatic cells that participate in the storage and metabolism
of vitamin A:
A. Hepatocytes
B. Kupfer cells
C. Endothelial cells
D. Hepatic fibroblasts
E. Stellate cells (Ito cells)
ANSWER: E
977.
Microfilaments surrounding the canaliculi and propelling secreted biliary
fluid along the canaliculi are which of the following:
A. Fibronectin and laminin
B. Vinculin and actin
C. Talin and vinculin
D. Tensin and myosin
E. Actin and myosin
ANSWER: E
978.
A single large lipid droplet, macro vesicular steatosis, that displaces the
nucleus may be seen in all of the following, EXCEPT:
A. Hepatitis virus C
B. Alcohol abuse
C. Obesity
D. Diabetis mellitus
E. Hepatitis virus B
ANSWER: E
979.
Injury to the liver associated with an influx of acute or chronic
inflammatory cells is termed as which of the following:
A. Liver regeneration
B. Liver cirrhosis
C. Hepatoma
D. Liver degeneation
E. Hepatitis
ANSWER: E
980.
Yellow discoloration of the skin and sclerae, internal organs and mucus
membranes is referred to as:
A. Melanosis
B. Vitiligo
C. Nevi
D. Cyanosis
E. Jaundice
ANSWER: E
981.
Unconjugated bilirubin is characterized by all of the following,
EXCEPT:
A. Insoluble in water
B. Tightly complexed to serum albumin
C. Cannot be excreted in the urine
D. May diffuse into tissue and produce toxic injury
E. Increased only in impaired bile flow
ANSWER: E
982.
Conjugated bilirubin is characterized by all of the following, EXCEPT:
A. Water soluble
B. Nontoxic
C. Loosely bound to albumin
D. Can be excreted in the urine
E. May induce kernicterus
ANSWER: E
983.
Jaundice occurs when the equilibrium between bilirubin production and
clearance is disturbed by all of the following factors, EXCEPT:
A. Excessive production of bilirubin
B. Impaired bile flow
C. Reduced hepatocyte uptake
D. Impaired conjugation
E. Increased hepatocellular excretion
ANSWER: E
984.
All of the following syndromes are characterized by hereditary
hyperbilirubinemias, EXCEPT:
A. Crigler-Najjar syndrome type II
B. Gilbert syndrome
C. Dubin-Johnson syndrome
D. Rotor syndrome
E. Crigler-Najjar syndrome type III
ANSWER: E
985.
The morphologic alterations that cause liver failure are all of the
following, EXCEPT:
A. Massive hepatic necrosis
B. Acute inflammation
C. Cirrhosis
D. Chronic inflammation
E. Small fatty droplets in hepatocytes
ANSWER: E
986.
Liver chirrhosis is characterized by all of the following, EXCEPT:
A. Bridging fibrous septa
B. Parenchymal nodular architecture
C. Reorganized vascular architecture
D. Disrupted liver architecture
E. Hemosiderin granules in liver cells
ANSWER: E
987.
Collagen synthesis by hepatic stellate cells (Ito cells) and its deposition
may be caused by all of the following, EXCEPT:
A. Disruption of the extracellular matrix
B. Direct stimulation of stellate cells by toxins
C. Cytokine production by stimulated liver cells
D. Inflammatory cytokine production (TNF-a, TGF-b)
E. Apoptosis of Ito cells
ANSWER: E
988.
The possible machanisms of most cirrhotic deaths are all of the
following, EXCEPT:
A. Progressive liver failure
B. Complication related to portal hypertension
C. Development of hepatocellular carcinoma
D. Systemic infection
E. Pulmonary thromboembolism
ANSWER: E
989.
The major consequences in portal hypertension in cirrhosis are all of the
following, EXCEPT:
A. Formation of portosystemic venous shunts
B. Congestive splenomegaly
C. Hepatic encephalopathy
D. Ascites
E. Intracerebral hematoma
ANSWER: E
990.
The pathogenesis of ascites involves all of the following mechanisms,
EXCEPT:
A. Sinusoidal hypertension
B. Percolation of the hepatic lymph into the peritoneal cavity
C. Intestinal fluid leakage
D. Renal retention of sodium and water
E. Increased oncotic plasma pressure
ANSWER: E
991.
Principal sites of portosystemic shunts opening with the rise of portal
systemic pressure are all of the following, EXCEPT:
A. Veins within the rectum (hemorrhoids)
B. Veins of cardioesophageal junction
C. Retroperitoneal vein
D. Veins of falciform ligament of the liver
E. Hepatic veins
ANSWER: E
992.
Hepatitis A is characterized by all of the following, EXCEPT:
A. Benign, self-limited infectious course
B. Low death rate
C. Fecal-oral route of transmission
D. Protective immunity against reinfection
E. Fulminant course
ANSWER: E
993.
Hepatitis B may be characterized by all of the following, EXCEPT:
A. Prolonged persistence of HBV in patient's blood
B. Presence of HBV in all pathologic and physiologic body fluids
C. Vertical transmission
D. Contamination of blood and blood products
E. Small size virus with defective RNA
ANSWER: E
994.
Hepatitis B virus may produce all of the following morphologic variants
of liver disease, EXCEPT:
A. Acute hepatitis
B. Nonprogressive chronic hepatitis
C. Progressive chronic hepatitis
D. Fulminant hepatitis
E. Liver steatosis
ANSWER: E
995.
Hepatitis B virus may produce all of the following variants of liver
disease, EXCEPT:
A. Progressive chronic hepatitis
B. Asymptomatic carrier state
C. Indolent chronic hepatitis
D. Acute hepatitis
E. Congestive cirrhosis
ANSWER: E
996.
The carrier state for hepatitis virus B is defined by the presence of which
of the following:
A. HBeAg in the urine for 12 months
B. HBcAg in the cerebrospinal fluid for at least 6 months
C. HBcAg in the serum for 12 months
D. HBsAg in the urine for 8 months
E. HBsAg in the serum for 6 months
ANSWER: E
997.
The possible outcomes of hepatitis virus B acute infection include all of
the following, EXCEPT:
A. Subclinical disease
B. Acute hepatitis
C. "Healthy" carrier
D. Persistent infection
E. "Nutmeg" liver
ANSWER: E
998.
Hepatitis virus C is characterized by all of the following, EXCEPT:
A. Parenteral route of transmission
B. Possible sexual and vertical transmission
C. High rate of progression to chronic disease
D. Absence of effective immunity
E. Low rate of progression to cirrhosis
ANSWER: E
999.
Hepatitis virus D is characterized by all of the following, EXCEPT:
A. High risk for HDV infection to male homosexuals
B. Rare development of chronicity
C. Conversion of mild chronic HBV infection to fulminant disease
D. High risk for HDV infection to drug addicts and hemophiliacs
E. Common development of hepato-cellular carcinoma
ANSWER: E
1000.
Hepatitis virus E is characterized by all of the following, EXCEPT:
A. Enterically transmitted, water-borne infection
B. Common in young to middle aged adults
C. High mortality rate among pregnant women
D. Commonly self-limited disease
E. Inducing massive liver necrosis
ANSWER: E
1001.
Morphologically, acute viral B hepatitis is characterized by all of the
following, EXCEPT:
A. Diffuse swelling (ballooning degeneration) of hepatocytes
B. Inflammatory cells in the sinusoids and portal tracts
C. Hepatocellular apoptosis
D. Confluent and bridging necrosis of hepatocytes
E. Portal tract expansion with fibrous tissue
ANSWER: E
1002.
Morphologically, chronic indolent viral B hepatitis is characterized by
all of the following, EXCEPT:
A. Well preserved liver architecture
B. Periportal and bridging fibrosis
C. Limited to portal tract inflammation
D. Smoldering hepatocyte necrosis throughout the lobule
E. Severe cholestasis
ANSWER: E
1003.
Microscopically, fulminant hepatitis virus B is characterized by all of the
following, EXCEPT:
A. Complete destruction of hepatocytes in damaged lobules
B. Collapsed reticulin framework
C. Preserved portal tracts
D. Inflammatory cell reaction
E. Hepatocyte regenerative nodules
ANSWER: E
1004.
Macroscopically, the liver in chronic alcoholism is characterized by all
of the following, EXCEPT:
A. Enlargement (up to 4 to 6 kg)
B. Soft consistence
C. Yellow and greasy appearance
D. Smooth surface
E. "Nutmeg" liver
ANSWER: E
1005.
Alcoholic hepatitis is characterized by all of the following, EXCEPT:
A. Swelling, steatosis and necrosis of hepatocytes
B. Mallory bodies formation
C. Neutrophilic reaction
D. Sinusoidal and perivenular fibrosis
E. Councilman bodies formation
ANSWER: E
1006.
Most forms of chronic renal failure produce increased serum levels of all
of the following substances, EXCEPT:
A. Aldosterone
B. Phosphate
C. Parathormone
D. Renin
E. Calcium
ANSWER: E
1007.
Uremia is associated with all of the following abnormalities, EXCEPT:
A. Peripheral neuropathy
B. Gastritis
C. Pericarditis
D. Diffuse alveolar damage
E. Polycythemia
ANSWER: E
1008.
Glomerular injury caused by circulating complexes occurs in all of the
following disorders, EXCEPT:
A. Syphilis
B. Goodpasture's syndrome
C. Hepatitis B
D. Systemic lupus erythematosus
E. Lung cancer
ANSWER: E
1009.
Diabetes mellitus is associated with all of the following renal disorders,
EXCEPT:
A. Diffuse glomerulosclerosis
B. Nodular glomerulosclerosis
C. Benigh nephrosclerosis
D. Acute pyelonephritis
E. Urate nephropathy
ANSWER: E
1010.
All of the following conditions predispose to urolithiasis, EXCEPT:
A. Hyperparathyroidism
B. Gout
C. Proteus pyelonephritis
D. Enteric hyperoxaluria
E. Sickle cell nephropathy
ANSWER: E
1011.
The factor least likely to cause acute pyelonephritis is which of the
following:
A. Pregnancy
B. Nephrolithiasis
C. Catheterization of the bladder
D. Prostatic hypertrophy
E. Septicemia
ANSWER: E
1012.
Mesangial cells can be characterized by all of the following properties,
EXCEPT:
A. Ingestion of macromolecules
B. Connection with Lacis cells
C. Ability to contract
D. Production of basement membrane proteins
E. Production of renin
ANSWER: E
1013.
Immunologicaly mediated glomerulonephritis all of the following cells
contribute to the glomerular injury, EXCEPT:
A. Macrophages
B. Platelets
C. Neutrophils
D. Mesangial cells
E. Mast cells
ANSWER: E
1014.
Systemic lupus erythematosus gives rise to all of the following patterns
of glomerular injury, EXCEPT:
A. Focal proliferative glomerulonephritis
B. Diffuse membranous glomerulonephritis
C. Diffuse proliferative glomerulonephritis
D. Mesangial proliferative glomerulonephritis
E. Lipoid nephrosis
ANSWER: E
1015.
All of the following statements correctly describe chronic pyelonephritis,
EXCEPT:
A. It causes asymmetrically scarred kidneys
B. It is associated with vesicoureteral reflux in most cases
C. It may produce thyroidization of tubules
D. It is an important cause of secondary nephrosclerosis
E. It spares the calyces and pelvis
ANSWER: E
1016.
All of the following statements correctly describe analgesic abuse
nephropathy, EXCEPT:
A. It is characterized by tubulo-interstitial component
B. It is often caused by phenacetin
C. It causes inability to concentrate urine
D. It often improves with drug with drawal
E. It predisposes to the development of renal cell carcinoma
ANSWER: E
1017.
Renal diseases producing systemic hypertension include all of the
following, EXCEPT:
A. Acute glomerulonephritis
B. Chronic glomerulonephritis
C. Chronic pyelonephritis
D. Renal vasculitis
E. Renal amyloidosis
ANSWER: E
1018.
Histologic features of malignant nephrosclerosis include all of the
following, EXCEPT:
A. Fibrinoid necrosis of arterioles
B. Medial thickening of arterioles
C. Renal artery thrombosis
D. Focal renal parenchymal infarction
E. Fibromuscular dysplasia of the renal artery
ANSWER: E
1019.
All of the following statements correctly describe renal artery stenosis,
EXCEPT:
A. It is an uncommon form of hypertension
B. It is the most common curable form of hypertension
C. It is usually caused by atherosclerotic plaque
D. It produces high renin levels in the venous blood of the ischemic kidney
E. It is treated by hemodialysis
ANSWER: E
1020.
Obstetrically related renal disease includes all of the following disorders,
EXCEPT:
A. Diffuse cortical renal necrosis
B. Acute ischemic tubular necrosis
C. Acute glomerulonephritis
D. Hydronephrosis
E. Nephrocalcinosis
ANSWER: E
1021.
Hematuria is a characteristic clinical feature of all of the following
diseases, EXCEPT:
A. Glomerulonephritis
B. Nephrolithiasis
C. Renal cell carcinoma
D. Bladder papilloma
E. Malakoplakia
ANSWER: E
1022.
Ultrastructural changes in children primary nephrotie syndrome involve
which of the following glomerular elements:
A. Endothelium
B. Mesangium
C. Bloodvessels
D. Basement membrane
E. Podocytes
ANSWER: E
1023.
Which statement correctly characterizes membranous glomerulopathy?
A. It is the most common cause of nephrotie syndrome in children
B. Patients usually present with acute renal failure
C. It is characterized by diffuse proliferative glomerulonephritis
D. It is characterized by mesangial interposition phenomenon
E. Electron microscopy demonstrates numerous subepithelial immunetype
deposits
ANSWER: E
1024.
Subendothelial granular electron-dense deposits can be found in which
of the following diseases:
A. Rapidly progressive glomerulonephritis
B. Side cell nephropathy
C. Membranous glomerulonephritis
D. Gouty nephropathy
E. Systemic lupus erythematosus
ANSWER: E
1025.
What pathologic condition of the kidneys is caused by mercury
poisoning?
A. Renal papillary necrosis
B. Crescentic glomerulonephritis
C. Acute interstitial nephritis
D. Renal cell carcinoma
E. Acute tubular necrosis
ANSWER: E
1026.
In nonobstructive chronic pyelonephritis the most common way for
bacteria to gain entrance into the kidney is which of the following:
A. Arterial bloodstream
B. The lymphatics
C. Venous bloodstream
D. Aberrant arteriovenous shunts
E. Vesicoureteral reflux
ANSWER: E
1027.
An immunofluorescence-stained kidney specimen from a patient with
poststreptococcal glomerulonephritis is likely to show which of the following:
A. Linear deposits of Ig G
B. Granular deposits of IgA
C. Linear deposits of streptococcal antigen
D. Granular deposits of streptococcal antigen
E. Granular deposits of Ig G
ANSWER: E
1028.
All of the following clinical features are likely to be found in nephrotic
syndrome, EXCEPT:
A. Proteinuria
B. Hypoalbuminemia
C. Hyperlipidemia
D. Edema
E. Hematuria
ANSWER: E
1029.
All of the following statements regarding postinfectious
glomerulonephritis are true, EXCEPT:
A. The disease follows streptococcus infection
B. Electron microscopy shows large subendothelial immune-type deposits
C. The histologic picture is that of diffuse proliferative glomerulonephritis
D. The clinical picture is characteristic of acute nephritis
E. Most affected children develop chronic renal failure
ANSWER: E
1030.
All of the following statements regarding Goodpasture's syndrome are
true, EXCEPT:
A. Patients present with hemoptysis and hematuria
B. Death occurs due to uremia and pulmonary hemorrhage
C. Electron microscopy shows the absence of electron-dense deposits
D. Immunofluorescence reveals linear deposits of IgG in the glomeruli
E. Immunofluorescence reveals granular deposits of IgG in the glomeruli
ANSWER: E
1031.
Hydronephrosis is caused by all of the following, EXCEPT:
A. Large uterine leiomyoma
B. Renal calculi
C. Benign prostatic hypertrophy
D. Papillary transitional cell carcinoma of the ureter
E. Chronic renal vein thrombosis
ANSWER: E
1032.
Benign nephrosclerosis is characterized by all of the following,
EXCEPT:
A. Narrowing of the lumen of the arterioles and small arteries
B. Thickening and hyalinization of the vessels' walls
C. Foci of tubular atrophy
D. Deposition of collagen within the Bowman space
E. Deposition of amyloid within the Bowman space
ANSWER: E
1033.
The tubular epithelial cells in acute tubular necrosis are characterized by
all of the following pathologic features, EXCEPT:
A. Karyolysis
B. Plasmolysis
C. Plasmorrhexis
D. Plasmocoagulation
E. Tubulorrhexis
ANSWER: E
1034.
Hydronephrosis is characterized by all of the following, EXCEPT:
A. Thinning of the renal parenchyma
B. Dilatation of the renal pelvis
C. Dilatation of the renal calyces
D. Progressive atrophy of the kidney
E. Kidney infarct
ANSWER: E
1035.
In the urinary tract obstruction all pathologic processes can be found.
EXCEPT:
A. Dilatation of the pelvis and calyces
B. Interstitial inflammation
C. Interstitial fibrosis
D. Glomerular and tubular atrophy
E. Ischemic tubular necrosis
ANSWER: E
1036.
In pathogeny of glomerulonephritis a considerable role plays:
A. Reinfection
B. Toxicness of microbes
C. A presence of the inflammatory diseases of kidneys in anamnesis
D. Superinfection
E. Sensitizing of an organism
ANSWER: E
1037.
Patogenetic basis of glomerulonephritis is formed by:
A. Exsudates reactions on the basic membranes of glomerular capillaries
B. Necrotic reactions on the basic membranes of glomerular capillaries
C. Exsudative necrotic reactions on the basic membranes of glomerular
capillaries
D. Proliferative reactions on the basic membranes of glomerular capillaries
E. Immunoreactions on the basic membranes of glomerular capillaries
ANSWER: E
1038.
A 43 years old man is diagnosed clinically with the subacute rapidly
progressive| glomerulonephritis. Its morphological equivalent is:
A. Intracapillar proliferative glomerulonephritis;
B. Glomerulosclerosis ;
C. Membranous glomerulonephritis;
D. Mesangial proliferative glomerulonephritis.
E. Extracapillary proliferative glomerulonephritis
ANSWER: E
1039.
Structurally-functional unit of kidney is:
A. Nephron
ANSWER: A
1040.
What type of epithelium covers the mucous membrane of urinoexcretory
ways?
A. Monolayer pavement epithelium;
B. Multi-layered ciliated ;
C. Monolayer cylindrical;
D. Multi-layered pavement unonkeratinizing .
E. Multi-layered transitional;
ANSWER: E
1041.
In case of poisoning by mercuric chloride there is a necrosis of
A. Cardiac hystiocytes
B. Glomerular system of kidneys
C. Renal stroma
D. Wall of ureter
E. Epithelium of renal tubules
ANSWER: E
1042.
What develops in case of prolong obstruction of ureters by stone in a
kidney
A. Heart attack
B. Gangrene
C. Glomerulonephritis
D. Amiloidosis
E. Hydronephrosis
ANSWER: E
1043.
What disease can be complicated by amyloidosis:
A. Atherosclerosis
B. Heart ischemic disease
C. Croupous pneumonia
D. Hypertension
E. Fibrous cavernous tuberculosis of lungs
ANSWER: E
1044.
Symptomatic hypertension evolves most frequently from:
A. Atherosclerosis of renal arteries
B. Thyrotoxicosis
C. Aldosteronism
D. Pheochromocytoma
E. Glomerulonephritis
ANSWER: E
1045.
A gouty kidney develops as a result of abnormality of metabolism of
A. Chromoproteins
B. Lipids
C. Albumins
D. Carbonhydrates
E. Purines
ANSWER: E
1046.
What abnormality develops at shock
A. Chronic kidney insufficiency
B. Glomerulonephritis
C. Renal amiloidisis
D. Renal cirrhosis
E. Acute kidney insufficiency
ANSWER: E
1047.
Morphological form of subacute glomerulonephritis is:
A. Mesangial membranous glomerulonephritis
B. Mesangial proliferative
C. Minimum changes
D. Intracapillar proliferative
E. Extracapillar proliferative
ANSWER: E
1048.
1049.
1050.
1051.
1052.
by
A clinical syndrome is characteristic for the renal amiloidisis
A. Hepatonephric
B. Lowe's syndrome
C. Alport's syndrome
D. Nephritic syndrome
E. Nephrotic syndrome
ANSWER: E
Changes in a colon at uremia are
A. Granulomatous inflammation
B. Stricture
C. Polyposis
D. Ulcerous colitis
E. Fibrinous [diphtheritic] colitis
ANSWER: E
Proteinuria at glomerulonephritis is caused by
A. Purulent exsudate permeating into the urine
B. Hyperproteinemia
C. Purulent infiltration of stroma
D. Damage of tubular wall
E. Damage of wall of glomerular capillares
ANSWER: E
Most frequent complication of glomerulonephritis is
A. Amiloidosis
B. Increase in arterial pressure
C. Renal abscesses
D. Hematuria
E. Arteriolosclerotic kidney
ANSWER: E
At extracapillar productive glomerulonephritis half moons are formed
A. Red corpuscles
B. Neutrophilic leucocytes
C. Red corpuscles and neutrophilic leucocytes
D. Exfoliated endothelium and fibrin
E. Exfoliated renal epithelium and fibrin
ANSWER: E
1053.
The main complication of nephrolithiasis is
A. Hypertrophy of the left ventricle
B. Hypertension
C. Contracted [granular] kidney
D. Cancer of kidney
E. Hydronephrosis
ANSWER: E
1054.
The changes of epithelium of tubulis are in the shock stage of acute
kidney insufficiency
A. Metaplasia of epithelium
B. Necrosis of epithelium
C. The changes are absent
D. Removing a layer by the layer of epithelium
E. Dystrophy of epithelium
ANSWER: E
1055.
The changes of tubular epithelium in the oliguric stage of acute renal
insufficiency
A. Metaplasia of epithelium
B. Dystrophy of epythelium
C. Changes are absent
D. Exfoliation of epithelium
E. Necrosis of epithelium
ANSWER: E
1056.
What belongs to the inherited diseases from these tubular patologies
A. Pyelonephritis
B. Necrotic nephrosis
C. “Myelome” kidney
D. “Shield-shaped” kidney
E. Tubular enzimatic pathology
ANSWER: E
1057.
What illness from the listed below relates to the large mottled kidney
A. Amiloidosis of kidney
B. Chronic pyelonephritis
C. Nephrolithiasis
D. Chronic glomerulonephritis
E. Subacute glomerulonephritis
ANSWER: E
1058.
The characteristic feature of chorioepithelioma is
A. Presence of lymphatic vessels
B. Petrification
C. Absence of stroma
D. Sclerosis of stroma
E. Atrophy of chorionic epithelium
ANSWER: C
1059.
Histological type of of high quality hyperplasia of prostatic gland
A. Muscular type
B. Stromal vascular type
C. Amyloid type
D. Fibroid cystous type
E. Mixed type
ANSWER: E
1060.
The most frequent clinical display of fibroid hyperplasia of mucous
membrane of uterus is:
A. Pain syndrome
B. Masculinization
C. Enlargement of uterus determined by ultrasonography
D. The characteristic clinical displays are absent
E. Metrorrhagia
ANSWER: E
1061.
The dishormonal disease of the sexual system of men is
A. Prostatitis
B. Orchitis
C. Epididymitis
D. Cancer of testicles
E. Gynecomastia
ANSWER: E
1062.
The dishormonal disease of the sexual system of men is
A. Prostatitis
B. Orchitis
C. Cancer of prostatic gland
D. Urethritis
E. Adenoma of prostate
ANSWER: E
1063.
The dishormonal disease of the sexual system of women is
A. Cervicitis
B. Endometritis
C. Salpingitis
D. Cancer of the body of uterus
E. Mastopathy
ANSWER: E
1064.
The dishormonal disease of the sexual system of women is
A. Oophoritis
B. Cystitis
C. Sarcoma of uterus
D. Endometritis
E. Endocervicosis
ANSWER: E
1065.
What process occurs in the wall of urinary bladder at prostatopathy ?
A. Sclerosis
B. Fibrosis
C. Atrophy
D. The changes are absent
E. Compensatory hypertrophy
ANSWER: E
1066.
Name a process, which is regarded as an inflammatory disease of
mucous membrane of uterus
A. Cystophorous hyperplasia
B. Pseudo erosion
C. Polypous excrescence
D. Endometriosis
E. Endometritis
ANSWER: E
1067.
What pathological process the hypertrophy of prostatic gland is related
to?
A. Chronic prostatitis
B. Acute prostatitis
C. Hyperfunction of sexual glands
D. Disorder of the urine outflow
E. Hypofunction of sexual glands
ANSWER: E
1068.
Name the complication of dishormonal hypertrophic prostatopathy?
A. Formation of the prostatic cysts.
B. Purulent melting of prostate
C. Sclerosis of prostate
D. Appearance of polypous excrescences
E. Difficulty in the urine outflow
ANSWER: E
1069.
Give the determination of gynecomastia
A. Non-cancerous cystous dysplasia of mammary gland
B. Non-cancerous hyperplasia of mammary gland
C. Malignant dysplasia of mammary gland
D. Non-cancerous scleroyic dysplasia of mammary gland
E. Non-cancerous dysplasia of breast in men
ANSWER: E
1070.
Pneumonia (pneumonia) is the disease, which includes large group of
varied after etiology, pathogenic and morphological description of
inflammations:
A. respiratory department of lung
B. pleura
C. mediastinum
D. overhead respiratory tracts
E. pericardium
ANSWER: E
1071.
The way of penetration of exciter of pneumonia in lungs is called:
A. contageous
B. primary
C. second
D. primary or second
E. bronchogenic
ANSWER: E
1072.
Lobar pneumonia – in 95 % cases is caused by:
A. streptococcus
B. staphylococcus
C. chlamydiae
D. coli bacteria
E. Franklyn’s pneumococcus
ANSWER: E
1073.
At croupous pneumonia in the red hepatization color of sputum is:
A. black
B. red
C. yellow
D. grey
1074.
1075.
1076.
1077.
up:
E. ferruginous
ANSWER: E
Pulmonary complications of croupous pneumonia can be:
A. pneumofibrosis
B. acute bronchitis
C. fibrous alveolitis
D. meningitis
E. carnification
ANSWER: E
Extrapulmonary complication of croupous pneumonia can be:
A. abscess of the lung
B. empyema of pleura
C. gangrene of the lung
D. bronchitis
E. meningitis
ANSWER: E
By foreign bodies of respiratory tracts can arise up:
A. stagnant pneumonia
B. paravertebral pneumonia
C. hypostatic pneumonia
D. croupous pneumonia
E. atelectatic pneumonia
ANSWER: E
By the hit of extraneous bodies in respiratory tracts in lungs can arise
A. aspiration pneumonia
B. stagnant pneumonia
C. hypostatic pneumonia
D. atelectatic pneumonia
E. croupous pneumonia
ANSWER: E
1078.
The most prevalent way of penetration of exciter at bronchopneumonia
is:
A. bronchogenic
B. haematogenic
C. lymphogenic
D. mixed
E. enterogenic
ANSWER: A
1079.
Atelectasis is:
A. The active сollaps of pulmonary tissue, which can arise up at the lack of
surfactant
B. multiplying the amount of teethridge
C. in. diminishing of amount of teethridge
D. passive сollaps force-feed exsudate or air
E. passive сollaps of pulmonary fabric force-feed tumour
ANSWER: A
1080.
Collapse is:
A. The passive сollaps force-feed exsudate, air or tumour
B. The active сollaps of pulmonary fabric, which can arise up at the lack of
surfactant
C. multiplying the amount of teethridge
D. diminishing of amount of teethridge
E. the active сollaps of pulmonary fabric with diminishing of amount of
teethridge
ANSWER: A
1081.
The exception of part of teethridge from a respiratory function draws the
development of the:
A. collapse
B. atelectasis
C. Hamman-Rich syndrome
D. collapse and atelectasis
E. vicarious (compensatory) emphysema
ANSWER: E
1082.
Subject to character of exsudate pneumonia is:
A. serosal
B. primary
C. bronchopneumonia
D. interstitial
E. secondary
ANSWER: A
1083.
What is the possible complication of bronchopneumonia:
A. gangrene
B. myocardial infarction
C. pulmonary infarction
D. Hamman-Rich syndrome
E. renal infarct
ANSWER: A
1084.
To the atipical forms of pneumonias belongs:
A. intermediate
B. viral
C. croupous
D. bronchopneumonia
E. bacterial
ANSWER: A
1085.
Intermediate pneumonia is:
A. peribronchial
B. lobular
C. segmental
D. acinous
E. croupous
ANSWER: A
1086.
What is the possible complication of interstitial pneumonia:
A. pneumosclerosis
B. myocardial infarction
C. bronchial asthma
D. acute bronchitis
E. renal infarct
ANSWER: A
1087.
A chronic bronchitis with the protracted clinical course is accompanied:
A. by dystrophy of elastic, muscular and cartilaginous frameworks
B. by hypertrophy of elastic and cartilaginous frameworks
C. by atrophy of elastic framework
D. by hyperplasia of muscular type
E. by dystrophy of cartilaginous framework
ANSWER: A
1088.
Emphysema of lungs is:
A. The pathological state of pulmonary fabric, which is characterized by
enhanceable maintenance in her air
B. The active collapse of pulmonary fabric, which can arise up at the lack
of surfactant
C. multiplying the amount of teethridge
D. the passive collapse force-feed exsudate, air or tumor
E. the pathological state of pulmonary fabric, which is characterized by the
lowered maintenance in her air
ANSWER: A
1089.
Emphysema of lungs is:
A. acute
B. chronic
C. croupous
D. subacute
E. vesicular
ANSWER: E
1090.
The development of the vesicular emphysema is connected with:
A. chronic bronchitis, bronchiolitis
B. croupous pneumonia
C. delete of part of lungs
D. age-old involution of lungs
E. lobar pneumonia
ANSWER: A
1091.
Chronic bronchitis is associated with all of the following pathologic
changes, EXCEPT:
A. Inflammatory infiltration of alveolar walls
B. Goblet cell hyperplasia in bronchi
C. Goblet cell metaplasia in bronchioli
D. Hypertrophy of bronchial submucosal glands
E. Inflammatory infiltration of bronchioli
ANSWER: A
1092.
An autopsy of a 45-year-old male patient, who had double
bronchopneumonia and died under the phenomena of intoxication, revealed in
the lower lobe of the right lung some thick-walled cavity, 4 cm in diameter,
filled with liquid yellowish masses. Choose pathological process complicated
the course of pneumonia:
A. Tuberculoma
B. Gangrene
C. Abscess
D. Sequester
E. Empyema
ANSWER: A
1093.
An autopsy of 78-year-old male patient, who died from cardiopulmonary
insufficiency, revealed an enlarged right lung with massive fibrinous
superpositions on the pleura. Histologically, the alveolar lumen had
accumulations of fibrin and neutrophils. The lymph nodes in the lung roots
were pale pink and somewhat enlarged. Name the stage of croupous
pneumonia:
A. Red hepatization
B. Influx
C. Grey hepatization
D. Resolution
E. Empyema
ANSWER: C
1094.
On autopsy, 380 ml of some yellow fluid with an unpleasant odour were
found in the right pleural cavity. Microscopically, the liquid contained a lot of
neutrophilic granulocytes. Choose the diagnosis:
A. Pulmonary abscess
B. Phlegmon
C. Pulmonary gangrene
D. Pulmonary infarction
E. Pleural empyema
ANSWER: E
1095.
Chronic obstructive bronchitis is associated with all of the following
pathologic changes, EXCEPT:
A. Goblet cell hyperplasia in bronchi
B. Goblet cell metaplasia in bronchioli
C. Hypertrophy of bronchial submucosal glands
D. Inflammatory infiltration of bronchioli
E. Inflammatory infiltration of alveolar walls
ANSWER: E
1096.
An autopsy of a 43-year-old male, who died from cardiopulmonary
insufficiency, revealed an enlarged grey dense air-free lower lobe of the left
lung, the vincernl pleura of this lobe had superpositions of fibrin.
Microscopically, the lumens of alveoli revealed some fibrinous-leukocytic
exudate. Name the stage of croupous pneumonia:
A. Red hepatization
B. Influx
C. Grey hepatization
D. Resolution
E. Empyema
ANSWER: C
1097.
An autopsy of a 55-year-old male, who died from cardiopulmonary
insufficiency, revealed thickened deformed bronchi filled with some
mucopurulent exudate. Microscopically, an inflammatory infiltration, foci of
squamous metaplasia of the epithelium and an increased number of goblet cells
were found in the bronchi. Choose the diagnosis:
A. Chronic bronchitis
B. Bronchopneumonia
C. Bronchiectatic disease
D. Acute bronchitis
E. Fibrinous bronchitis
ANSWER: C
1098.
Complications of bronchopneumonia include all of the following,
EXCEPT:
A. Bronchiectasis
B. Pleural fibrosis
C. Metastatic abscess formation
D. Permanent lobar solidification
E. Lung histiocytosis
ANSWER: E
1099.
All of the following factors commonly predispose to bacterial
pneumonias, EXCEPT:
A. Viral respiratory tract infections
B. Cigarette smoking
C. Congestive heart failure
D. Anesthesia
E. Bacterial urinary tract infection
ANSWER: E
1100.
Aspiration of gastric contents produces all of the following types of
pulmonary injury, EXCEPT:
A. Adult respiratory distress syndrome
B. Lipoid pneumonia
C. Lung abscess
D. Empyema
E. Pulmonary alveolar proteinosis
ANSWER: E
1101.
Diseases with diffuse interstitial lung fibrosis development include all of
the following, EXCEPT:
A. Sarcoidosis
B. Asbestos
C. Rheumatoid arthritis
D. Bleomycin lung
E. Measles pneumonia
ANSWER: E
1102.
Eosinophilic infiltrates characterize all of the following disorders,
EXCEPT:
A. Loeffler's syndrome
B. Allergic bronchopulmonary aspergillosis
C. Bronchial asthma
D. Pigeon/bird breeder's lung
E. Pneumocystic infection
ANSWER: E
1103.
Cigarette smoke contributes to the pathogenesis of the chronic
obstructive emphysema by all of the following mechanisms, EXCEPT:
A. Attracting neutrophils into the lung
B. Stimulating release of neutrophil elastase
C. Inhibiting the ability of pulmonary leukocytes to clear bacteria
D. Inhibiting alpha 1-antitrypsin
E. Stimulating macrophage elastase activity
ANSWER: C
1104.
Noncaseating pulmonary granulomas can be found in all of the following
diseases, EXCEPT:
A. Chronic berylliosis
B. Silicosis
C. Sarcoidosis
D. Alveococcosis
E. Tuberculosis
ANSWER: E
1105.
Primary pulmonary hypertension is characterized by which of the
following:
A. Strong association with cigarette smoking
B. Association with pulmonary embolism
C. Common association with chronic obstructive lung disease
D. "Bush tea " drinking lung
E. Atherosclerosis of the pulmonary arteries
ANSWER: E
1106.
Cigarette smoking is causally related to all of the following pulmonary
diseases, EXCEPT:
A. Chronic bronchitis
B. Centrilobular emphysema
C. Small airway disease (bronchiolitis)
D. Bronchogenic carcinoma
E. Idiopathic pulmonary fibrosis (fibrosing alveolitis)
ANSWER: E
1107.
Types of emphysema, according to the anatomic nature of the lesion
include all of the following, EXCEPT:
A. Centriacinar
B. Panacinar
C. Paraceptal
D. Irregular
E. Interstitial
ANSWER: E
1108.
The abnormal fenestrations of the walls of the alveoli, destruction of
septal walls, abnormal airspaces and possibly blebs or bullae are common
histological features of which of the following diseases:
A. Chronic bronchitis
B. Bronchial asthma
C. Pheumonia
D. Lung abscess
E. Chronic obstructive emphysema
ANSWER: E
1109.
The abnormal dilation of the bronchi and bronchioli is the main
characteristic feature of which of the following diseases:
A. Emphysema
B. Lung abscess
C. Bronchial asthma
D. Pneumonia
E. Bronchiectasis
ANSWER: E
1110.
Patchy consolidation of the lung is the dominant characteristic of which
of the following:
A. Bronchopneumonia
B. Lobar pneumonia
C. Bronchiectasis
D. Emphysema
E. Bronchial asthma
ANSWER: A
1111.
Acute bacterial infection of an entire lobe is often referred to as which of
the following:
A. Bronchopneumonia
B. Lobar pneumonia
C. Emphysema
D. Tuberculosis
E. Bronchial asthma
ANSWER: B
1112.
The stages of lobar pneumonia include all of the following, EXCEPT:
A. Congestion
B. Red hepatization
C. Yellow hepatization
D. Grey hepatization
E. Resolution
ANSWER: C
1113.
Exudate in lobar pneumonia may consist of all of the following,
EXCEPT:
A. Fibrin
B. Neutrophils
C. Erythrocytes
D. Macrophages
E. Plasma cells
ANSWER: E
1114.
Complications of pneumonia include all of the following, EXCEPT:
A. Abscess formation
B. Empyema
C. Organization of the exudate
D. Thromboembolism of the pulmonary artery
E. Bacteremic dissemination
ANSWER: D
1115.
The most characteristic feature of viral pneumonia is which of the
following:
A. The interstitial nature of the inflammatory reaction
B. Presence of acute suppurative inflammation
C. Emphysema formation
D. Coalescent granulomas formation
E. Suppurative destruction of the lung parenchyma
ANSWER: A
1116.
The cardinal histologic change in the acute abscess of the lung is which
of the following:
A. Emphysema formation
B. The interstitial nature of the inflammatory reaction
C. Fibrinous exudate
D. Ghon complex formation
E. Suppurative destruction of the lung parenchyma
ANSWER: E
1117.
All of the following diseases are pneumoconioses, EXCEPT:
A. Anthracosis
B. Silicosis
C. Tuberculosis
D. Asbestosis
E. Berylliosis
ANSWER: C
1118.
The stain used to identify fibrin is which of the following:
A. Weigert's stain
B. Congo red stain
C. PAS-reaction
D. Hematoxylin and eosin stain
E. Toluidin blue stain
ANSWER: A
1119.
The term given to the group of lung diseases that are caused by the
chronic inhalation of particulate or gaseous agents as a result of occupational
exposure is which of the following:
A. Granulomatous disease
B. Pneumoconiosis
C. Mycobacteriosis
D. Pseudolymphoma
E. Bronchiectasis
ANSWER: B
1120.
Conditions that predispose to lung abscess formation are all of the
following, EXCEPT:
A. Aspiration of infective material
B. Antecedent primary bacterial infection
C. Septic embolism
D. Neoplasia
E. Injury of the mucociliary apparatus
ANSWER: E
1121.
Factors that predispose to bacterial pneumonia are all of the following,
EXCEPT:
A. Loss or supression of the cough reflex
B. Injury of the mucociliary apparatus
C. Pulmonary congestion and edema
D. Accumulation of secretions
E. Septic embolism
ANSWER: E
1122.
Type 1 hypersensitivity reaction is the most common cause of which of
the following diseases:
A. Atopic asthma
B. Nonatopic asthma
C. Drug-induced asthma
D. Bacterial pneumonia
E. Bronchogenic carcinoma
ANSWER: A
1123.
The histological features in hypersensitivity pneumonitis include all of
the following, EXCEPT:
A. Interstitial pneumonitis
B. Interstitial fibrosis
C. Obliterative bronchiolitis
D. Hemosiderosis
E. Granuloma formation
ANSWER: D
1124.
The infection causing interstitial pneumonia is which of the following:
A. Gram-positive bacterial
B. Gram-negative bacterial
C. Viral
D. Fungal
E. Parasitic
ANSWER: C
1125.
The form of chronic interstitial pneumonia characterized by marked
proliferation and desquamation of alveolar lining cells is which of the
following:
A. Usual interstitial pneumonitis
B. Idiopathic pulmonary fibrosis
C. Desquamative interstitial pneumonitis
D. Lymphoid interstitial pneumonitis
E. Hamman-Rich syndrome
ANSWER: C
1126.
Tumor associated mainly with occupational exposure to asbestos is
which of the following:
A. Bronchioloalveolar carcinoma
B. Oat cell carcinoma
C. Mesothelioma
D. Squamous cell carcinoma
E. Adenocarcinoma
ANSWER: C
1127.
Clinically, insulin-dependent diabetes mellitus is characterized by all of
the following, EXCEPT:
A. Normal weight
B. Anti-islet cell antibodies in the serum
C. Ketoacidosis
D. Decreased blood insulin level
E. Normal or increased blood insulin level
ANSWER: E
1128.
Clinically non-insulin-dependent diabetes mellitus is characterized by all
of the following, EXCEPT:
A. Obesity
B. Absence of anti-islet-cell antibodies
C. Absence of ketoacidosis
D. Normal or increased blood insulin level
E. Decreased blood insulin level
ANSWER: E
1129.
Secondary diabetes may accompany all of the following diseases,
EXCEPT:
A. Cytomegalovirus infection
B. Down syndrome
C. Pituitary tumors
D. Adrenal endocrinopathy
E. Bacterial infection
ANSWER: E
1130.
The causes of morbidity and death from diabetes are the long-term
complications developing in all of the following organs, EXCEPT:
A. Kidneys
B. Liver
C. Eyes
D. Nerves
E. Blood vessels
ANSWER: E
1131.
All of the following pathogenic factors may contribute to the
development of type I diabetes, EXCEPT:
A. HLA-linked genes and other genetic loci
B. Immune response to normal beta-cells
C. Immune response to altered beta-cells
D. Molecular mimimcry of beta-eel after viral infection
E. Peripheral tissue insulin resistance
ANSWER: E
1132.
All of the following pathogenetic mechanisms may lead to the
development of type II diabetes, EXCEPT:
A. Primary beta-cell insufficiency
B. Deranged insulin secretion
C. Inadequate glucose utilization
D. Peripheral tissue insulin resistan<
E. Immune response to altered beta-cells
ANSWER: E
1133.
Insulin is necessary for all cell metabolic processes, EXCEPT:
A. Transmembrane transport of glucose and amino acids
B. Glycogen formation
C. Protein synthesis
D. Glucose conversion to triglycerides
E. Lipid synthesis
ANSWER: E
1134.
Genetic defects of beta-cell function leading to diabetes development are
all of following, EXCEPT:
A. Mutations in the gene for hepatocyte nuclear transcription factor 4a MODY-I
B. Mutations in the glucokinase gene - MODY2
C. Mutations in the gene for HNF-la - MODY3
D. Point mutations in mitochondrial DNA
E. Point mutations in the gene for low-density lipoprotein receptorMODY4
ANSWER: E
1135.
Diabetic nephropathy can lead to all of the following pathologic
processes, EXCEPT:
A. Nephrosclerosis
B. Glomerulosclerosis
C. Tubulopathy
D. Pyelonephritis
E. Hydronephrosis
ANSWER: E
1136.
Diabetic nephropathy is microscopically characterized by all of the
following, EXCEPT:
A. Diffuse glomerulosclerosis
B. Diffuse increase of the mesangial matrix
C. Mesangial cell proliferation
D. Basement membrane thickening
E. Amyloid masses within mesangial matrix
ANSWER: E
1137.
Diabetic microangiopathy is most evident in the capillaries of all of the
following organs, EXCEPT:
A. Skin
B. Sceletal muscle
C. Retina (eye)
D. Kidney
E. Liver
ANSWER: E
1138.
Diabetic microangiopathy may be seen in all of the following internal
organs, EXCEPT:
A. Kidney
B. Brain
C. Skin
D. Lung
E. Lens
ANSWER: E
1139.
Type I diabetes is clinically characterized by all of the following
features, EXCEPT:
A. Polyphagia
B. Poliuria
C. Weight loss
D. Polydipsia
E. Obesity
ANSWER: E
1140.
The most threatening and frequent atherosclerotic events in long
standing diabetes are all of the following, EXCEPT:
A. Myocardial infarction
B. Cerebrovascular accidents
C. Gangrene of the leg
D. Renal insufficiency
E. Lung artery thromboembolism
ANSWER: E
1141.
Morphological changes in diabetic microangiopathy are all of the
following, EXCEPT:
A. Endothelium prolit'iration
B. Generalized basement membrane thickening of capillaries
C. Hyalinosis
D. Artheriolosclerosis
E. Generalized basement membrane thikenening
ANSWER: E
1142.
Morphological lesions in the pancreas in type I diabetes include all of the
following, EXCEPT:
A. Reduction in the number of islets
B. Reduction in the size of islets
C. Inconspicuous islets
D. Leukocyte infiltration of islets
E. Replacement of islets by glycogen
ANSWER: E
1143.
Leukocytic infiltration of the islets in type I diabetes is also referred to
as:
A. Insulinoma
B. Granuloma
C. Pancreatitis
D. Interstitial inflammation
E. Insulitis
ANSWER: E
1144.
Pancreatic islet lesions in Zollinger- Ellison syndrome are associated
with which of the following diseases of gastrointestinal tract:
A. Cancerous stomach ulcer
B. Pseudomembranous colitis
C. Hypertrophic gastropathy
D. Crohn disease
E. Severe peptic ulceration
ANSWER: E
1145.
Major clinical features of Cushing's syndrome are all of the following,
EXCEPT:
A. Central obesity
B. Hirsutism
C. Hypertension
D. Osteoporosis
E. Gastrointestinal bleedings
ANSWER: E
1146.
The causes of primary adrenocor-tical insufficiency are all of the
following, EXCEPT:
A. Autoimmune process in the adrenal cortex
B. Waterhouse-Friderichsen syndrome
C. Loss of adrenal cortex
D. Metabolic failure in hormone production
E. Hypothalamic pituitary disease
ANSWER: E
1147.
The causes of secondary adrenocortical insufficiency are all of the
following, EXCEPT:
A. Hypothalamic pituitary disease
B. Neoplasm of the adrenal cortex
C. Long-term steroid administration
D. Tuberculosis of the adrenal cortex
E. Hereditary atrophy of cortex
ANSWER: E
1148.
Waterhouse-Friderichsen syndrome is characterized by all of the
following, EXCEPT:
A. Overwhelming bacterial infection
B. Rapidly progressive hypertension
C. Disseminated intravascular coagulation with widespread skin purpura
D. Rapidly developing adrenocortical insufficiency associated with
massive bilateral adrenal hemorrhages
E. Hyperpigmentation of the skin, particularly of sun-exposed areas
ANSWER: E
1149.
The dominant clinical symptom in patients with pheochromocytoma is
which of the following:
A. Rapidly progressive hypotension leading to collapse
B. Hyperpigmentation of the skin
C. Disseminated intravascular coagulation with widespread skin purpura
D. Severe peptic ulceration
E. Severe hypertension
ANSWER: E
1150.
The hypertension in pheochromocytoma is associated with synthesis of
which of the following products:
A. ACTH
B. Aldosteron
C. Cortisol
D. TSH
E. Catecholamines
ANSWER: E
1151.
The pheochromocytoma may be associated with all of the following
clinical symptoms, EXCEPT:
A. Tachycardia
B. Headache
C. Sweating
D. Tremor
E. Hyperpigmentation of the skin
ANSWER: E
1152.
Pituitary adenomas include all of the following variants, EXCEPT:
A. Growth hormone cell adenoma
B. Prolactin cell adenoma
C. ACTH cell adenoma
D. TSH cell adenoma
E. Catecholamine cell adenoma
ANSWER: E
1153.
Hypothyroidism may be caused by all of the following disorders,
EXCEPT:
A. Hashimoto thyroiditis
B. Immune block of TSH receptors
C. Iodine deficiency
D. Pituitary lesions reducing TSH secretion
E. Diffuse hyperplasia of thyroid in Graves disease
ANSWER: E
1154.
Thyrotoxicosis may be caused by all of the following disorders,
EXCEPT:
A. Diffuse hyperplasia of the thyroid in Graves disease
B. Ingestion of exogenous thyroid hormone
C. Hyperfunctional multinodular goiter
D. Hyperfunctional adenoma of the thyroid
E. Pituitary lesions reducing TSH secretion
ANSWER: E
1155.
The clinical manifestations of hyperthyroidism include all of the
following symptoms, EXCEPT:
A. Tremor
B. Ocular changes
C. Tachycardia
D. Atrophy of sceletal muscles
E. Skin striae
ANSWER: E
1156.
Hypertension is found in all of the following endocrine disorders,
EXCEPT:
A. Cushing's syndrome
B. Pheochromocytoma
C. Adrenal medullary hyperplasia
D. Conn's syndrome
E. Addison's disease
ANSWER: E
1157.
The defect of interventricular septum belongs to:
A. fetopathy
B. pathologies of placenta
C. birth trauma
D. perinatal pathology
E. embryopathy
ANSWER: E
1158.
Complete transposition of pulmonary artery and aorta belongs to:
A. fetopathy
B. pathology of placenta
C. birth trauma
D. perinatal pathology
E. embryopathy
ANSWER: E
1159.
The Fallot's triad (defect of interventricular septum, stenosis of
pulmonary artery, hypertrophy of right ventricle) belongs to:
A. fetopathy
B. pathology of placenta
C. birth trauma
D. perinatal pathology
E. embryopathy
ANSWER: E
1160.
The Fallot's pentade (defect of interventricular septum, narrowing of
pulmonary artery, dextraposition aorta, hypertrophy of right ventricle, defect of
interatrial septum) belongs to:
A. fetopathy
B. birth trauma
C. perinatal pathology
D. pathology of placenta
E. embryopathy
ANSWER: E
1161.
Cystic disease of liver is the plural cysts of different sizes, meets from
cystic disease kidney and pancreas, belongs to:
A. fetopathy
B. perinatal pathology
C. birth trauma
D. pathology of placenta
E. embryopathy
ANSWER: E
1162.
Agenesis of kidneys belongs to:
A. fetopathy
B. perinatal pathology
C. birth trauma
D. pathology of placenta
E. embryopathy
ANSWER: E
1163.
Renal hypoplasia belongs to:
A. fetopathy
B. perinatal pathology
C. birth trauma
D. pathology of placenta
E. embryopathy
ANSWER: E
1164.
Renal dysplasia belongs to:
A. fetopathy
B. perinatal pathology
C. birth trauma
D. pathology of placenta
E. embryopathy
ANSWER: E
1165.
Cysts of lungs belong to:
A. perinatal pathology
B. birth trauma
C. pathology of placenta
D. embryopathy
E. fetopathy
ANSWER: E
1166.
Innate emphysema belongs to:
A. perinatal pathology
B. birth trauma
C. pathology of placenta
D. embryopathy
E. fetopathy
ANSWER: E
1167.
Dysplastic acetabulum belongs to:
A. embryopathy
B. perinatal pathology
C. birth trauma
1168.
1169.
1170.
1171.
1172.
1173.
to:
D. pathology of placenta
E. fetopathy
ANSWER: E
Polydactyly is multiplying the number of fingers, belongs to:
A. embryopathy
B. perinatal pathology
C. birth trauma
D. pathology of placenta
E. fetopathy
ANSWER: E
The system hypoplasia of muscular system belongs to:
A. embryopathy
B. perinatal pathology
C. birth trauma
D. pathology of placenta
E. fetopathy
ANSWER: E
Fetopathy has a period from:
A. from 71 for 280 day
B. from 73 for 280 day
C. from 74 for 280 day
D. from 75 for 280 day
E. from 72 for 280 day
ANSWER: E
Hemolytic illness of new-born belongs to:
A. infectious fetopathyes
B. reactive changes
C. embryopathies
D. malformation
E. uninfectious fetopathyes
ANSWER: E
Dropsy of amnion belongs to:
A. defect of removing of placenta
B. defect of localization of placenta
C. malformation of form
D. reactive changes
E. malformation of amnion
ANSWER: E
Mucoviscidosis (fibrocystic disease [cystic fibrosis] of pancreas) belongs
A. infectious fetopathyes
B. reactive changes
C. embryopathies
D. malformation
E. uninfectious fetopathyes
ANSWER: E
Endocardial fibroelastosis belongs to:
A. infectious fetopathyes
B. reactive changes
C. embryopathies
D. malformation
E. uninfectious fetopathyes
ANSWER: E
1174.
Oligo(hydr)amnios belongs to:
A. defect of removing of placenta
B. defect of localization of placenta
C. malformation of form
D. reactive changes
E. malformation of amnion
ANSWER: E
1175.
What fetus is considered viable?
A. By mass of 960 gr, by length 35 centimeter
B. By mass of 970 gr, by length 35 centimeter
C. By mass of 980 gr, by length 34 centimeter
D. By mass of 1000 gr, by length 30 centimeter
E. By mass of 1000 gr, by length 35 centimeter
ANSWER: E
1176.
Births from 28 week by mass of fetus less than 1000 me and length, less
than 35 centimeter is named:
A. late abortion
B. prematurely fetus
C. overmature fetus
D. stillborn fetus
E. miscarriage
ANSWER: E
1177.
The death-rate of children in the first 7 days after birth is named:
A. intrapartum
B. postnatal
C. neonatal
D. intranatal
E. perinatal
ANSWER: E
1178.
Child's death-rate to births is named:
A. perinatal
B. intrapartum
C. postnatal
D. neonatal
E. antenatal
ANSWER: E
1.
2.
3.
4.
SITUATIONAL TASKS
Several days following a myocardial infarction, a 51-year-old man develops
the sudden onset of a new pansystolic murmur along with a diastolic flow
murmur. Workup reveals increased left atrial pressure that develops late in
systole and extends into diastole. Which of the following is the most likely
cause of the abnormalities present in this individual?
A. Aneurysmal dilation of the left ventricle
B. Obstruction of the aortic valve
C. Rupture of the left ventricle wall
D. *Rupture of a papillary muscle
E. Thrombosis of the left atrial cavity
Three weeks following a myocardial infarction, a 54-year-old man presents
with fever, productive cough, and chest pain. The pain is worse with
inspiration, better when he is sitting up, and not relieved by nitroglycerin.
Physical examination finds a friction rub along with increased jugular venous
pressure and pulsus paradoxus (excess blood pressure drop with inspiration).
Which of the following is the most likely explanation for these findings?
A. Caplan’s syndrome
B. *Dressler’s syndrome
C. Ruptured papillary muscle
D. Ruptured ventricular wall
E. Ventricular aneurysm
A 59-year-old woman presents with increasing shortness of breath. Physical
examination reveals signs of left heart failure. She is admitted to the hospital to
workup her symptoms, but she dies suddenly. A section from her heart at the
time of autopsy reveals marked thickening of the wall of the left ventricle, but
the thickness of the right ventricle is within normal limits. Many of the nuclei
of the myocytes in the wall of the left ventricle have a “box car” appearance.
The endocardium does not appear to be increased in thickness or fibrotic, and
the cardiac valves do not appear abnormal. The left ventricular cavity is noted
to be decreased in size. What is the most likely cause of this cardiac pathology?
A. Carcinoid heart disease
B. Cor pulmonale
C. Eccentric hypertrophy
D. *Systemic hypertensive
E. Volume overload
A 71-year-old woman presents with increasing chest pain and occasional
syncopal episodes, especially with physical exertion. She has trouble breathing
at night and when she lies down. Physical examination reveals a crescendodecrescendo midsystolic ejection murmur with a paradoxically split second
heart sound (S2). Pressure studies reveal that the left ventricular pressure
during systole is markedly greater than the aortic pressure. Which of the
following is the most likely diagnosis?
5.
6.
7.
8.
A. Aortic regurgitation
B. *Aortic stenosis
C. Constrictive pericarditis
D. Mitral regurgitation
E. Mitral stenosis
A 63-year-old man presents with signs of congestive heart failure, including
shortness of breath, cough, and paroxysmal nocturnal dyspnea. Physical
examination reveals a hyperdynamic, bounding, “water-hammer” pulse and a
decrescendo diastolic murmur. His hyperdynamic pulse causes “bobbing” of
his head. Which of the following is the most frequent cause of the cardiac
valvular abnormality present in this individual?
A. Aortic dissection
B. Infective endocarditis
C. Latent syphilis
D. Marfan syndrome
E. *Rheumatic fever
Physical examination of an asymptomatic 29-year-old woman with a history of
rheumatic fever during childhood finds an early diastolic opening snap with a
rumbling late diastolic murmur. Which of the following is the most likely
diagnosis?
A. Aortic regurgitation
B. Aortic stenosis
C. Mitral regurgitation
D. *Mitral stenosis
E. Pulmonic stenosis
A 7-year-old boy presents with the acute onset of fever, pain in several joints,
and a skin rash. Physical examination finds an enlarged heart, several
subcutaneous nodules, and a skin rash on his back with a raised, erythematous
margin. Laboratory tests find an elevated erythrocyte sedimentation rate and an
elevated antistreptolysin O titers. Within the past month, this boy most likely
had which one of the following abnormalities?
A. Anitschkow cells develop in the lungs
B. Aschoff bodies develop in the skin
C. *Beta-hemolytic streptococci infection of the pharynx
D. Pseudomonas aeruginosa infection of the aorta
E. Stenosis of the mitral valve
An autopsy done on a 23-year-old man who died suddenly with no previous
medical history reveals the right ventricle to be dilated with near total
transmural replacement of the right ventricle (RV) free-wall myocardium by
fat and fibrosis. No skin or hair abnormalities are seen. What is the best
diagnosis?
A. *Arrhythmogenic RV cardiomyopathy
B. Endocardial fibrosis
C. Hyper-serotonin RV syndrome
D. Loeffler endomyocarditis
E. Naxos syndrome
9. A 31-year-old woman presents with fever, intermittent severe pain in the left
upper quadrant of her abdomen, and painful lesions involving her fingers and
nail beds. History reveals that she had acute rheumatic fever as a child and that
when she was around 20 years of age she developed a new cardiac murmur. At
the present time one of three blood cultures submitted to the hospital lab grew
a specific bacteria. Which of the following is the most likely cause of her
disease?
A. Staphylococcus aureus
B. * α-hemolytic viridans streptococci
C. Candida species
D. Group A streptococci
E. Pseudomonas species
10. A 23-year-old woman develops the sudden onset of congestive heart failure.
Her condition rapidly deteriorates and she dies in heart failure. At autopsy,
patchy interstitial infiltrates composed mainly of lymphocytes are found, some
of which surround individual myocytes. Which of the following is the most
likely cause of this patient’s heart failure?
A. Autoimmune reaction (to group A β-hemolytic streptococci)
B. Bacterial myocarditis (due to S. aureus infection)
C. Hypersensitivity myocarditis (due to an allergic reaction)
D. Nutritional deficiency (due to thiamine deficiency)
E. *Viral myocarditis (due to coxsackievirus infection)
11. At the time of autopsy of a 39-year-old woman who died of complications of
systemic lupus erythematosus, several medium-sized vegetations are found on
both sides of the mitral valve and tricuspid valve. Which of the following is the
basic abnormality that produced these cardiac vegetations?
A. Turbulent blood flow through an incompetent mitral valve
B. Excess secretion of a vasoactive amine
C. *Presence of an anticardiolipin antibody
D. Cachexia produced by a hypercoagulable state
E. Bacterial colonization of an abnormal valve
12. A 37-year-old woman presents with prolonged cramps, nausea, vomiting,
diarrhea, and episodic flushing of the skin. Additionally, she develops pearly
white, plaque-like deposits on the tricuspid valve leaflets. Which of the
following disorders is most likely to be present in this individual?
A. Rheumatic heart disease
B. Amyloidosis
C. Iron overload
D. Hypothyroidism
E. *Carcinoid heart disease
13. A 59-year-old patient receiving chemotherapy with the anthracycline
Adriamycin develops severe heart failure. Sections from an endocardial biopsy
specimen reveal vacuolization of the endoplasmic reticulum of the myocytes.
Adriamycin therapy most frequently causes what type of cardiomyopathy?
A. *Dilated cardiomyopathy
B. Hyperplastic cardiomyopathy
C. Hypertrophic cardiomyopathy
D. Obliterative cardiomyopathy
E. Restrictive cardiomyopathy
14. A 3-month-old girl is being evaluated for feeding difficulty and failure to
thrive. Physical examination finds pallor, peripheral cyanosis, tachypnea, and
fine expiratory wheezing. Chest x-ray shows cardiac enlargement. She is
admitted to the hospital, quickly develops severe cardiac failure, and dies 3
days after admission. At the time of autopsy the endocardium is found to have
a “cream cheese” gross appearance. Histologic sections from this area reveal
thickening of the endocardium due to a proliferation of fibrous and elastic
tissue. Which of the following is the most likely diagnosis?
A. Dilated cardiomyopathy
B. Hypertrophic cardiomyopathy
C. Infective endocarditis
D. Libman-Sachs endocarditis
E. * Restrictive cardiomyopathy
15. A 49-year-old man 7 days after being admitted to the hospital for an inferior
wall, transmural myocardial infarction suddenly becomes short of breath.
Physical examination reveals hypotension, elevated jugular venous pressure,
and muffled heart sounds. His systemic blood pressure drops 13 mmHg with
inspiration. Which one of the following pathologic processes produced these
clinical findings?
A. Acute inflammation of the pericardium due to an autoimmune reaction
B. Acute mitral regurgitation due to rupture of a papillary muscle
C. Acute suppurative inflammation of the pericardium due to bacterial
infection
D. *Blood accumulation in the pericardial cavity due to rupture of the
ventricular wall
E. Serous fluid accumulation in the pericardial cavity due to congestive
heart failure
16. A 35-year-old man179. A 35-year-old man presents with weight loss, fever,
and fatigue. Physical examination finds signs and symptoms of mitral valve
disease. Further workup finds a pedunculated mass in the left atrium. The
tumor is resected and histologic sections reveal stellate cells in a loose myxoid
background. Which of the following is the most likely diagnosis?
A. Chordoma
B. Fibroelastoma
C. Leiomyoma
D. *Myxoma
E. Rhabdomyoma
17. A 2-year-old girl is being evaluated for growth and developmental delay. She
has had several past episodes when she would suddenly have trouble breathing,
become blue, and then assume a squatting position to catch her breath. Workup
finds a defect in the wall of the ventricular septum, increased thickness of the
right ventricle, and dextroposition of the aorta. Which of the following
cardiovascular abnormalities is most likely to be present in this child?
A. Coarctation of the aorta
B. Incompetence of the mitral valve
C. Patency of the foramen ovale
D. Persistence of the AV canal
E. *Stenosis of the pulmonic valve
18. A 2-month-old girl is being examined for a routine checkup. She was born at
term, and there were no problems or complications during the pregnancy. The
baby appeared normal at birth and has been asymptomatic. Physical
examination at this time finds a soft systolic murmur with a systolic thrill. No
cyanosis is present, and her peripheral pulses are thought to be within normal
limits. An ECG reveals slight left ventricular hypertrophy. Which of the
following is the most likely diagnosis?
A. Coarctation of the aorta
B. Patent ductus arteriosus
C. Persistent truncus arteriosus
D. Tetralogy of Fallot
E. *Ventricular septal defect
19. Which one of the following statements correctly describes the flow of blood in
an individual with an atrial septal defect who develops Eisenmenger’s
syndrome?
A. Aorta to pulmonary artery to lungs to left atrium to left ventricle to aorta
B. Left atrium to right atrium to right ventricle to lungs to left atrium
C. Left ventricle to right ventricle to lungs to left atrium to right ventricle
D. *Right atrium to left atrium to left ventricle to aorta to right atrium
E. Right ventricle to left ventricle to aorta to right atrium to right ventricle
20. A 43 years old man is diagnosed clinically with the subacute progressive
glomerulonephritis. Its morphological equivalent is:
A. Intracapillar proliferative glomerulonephritis;
B. Glomerulosclerosis
C. Membranous glomerulonephritis;
D. * Extracapillary proliferative glomerulonephritis
E. Mesangial proliferative glomerulonephritis.
21. In the villages of saliva mumps patient the microscopic study showing giant
cells that contain large basophilic intranuclear inclusion surrounded by a zone
enlightenment. The appearance of these cells resembles SAUVIGNY eye.
What is the most likely diagnosis?
A. mumps
B. non-specific bacterial parotitis
C. candidiasis mumps
D. non-contagious mumps because of lead poisoning
E. * cytomegalovirus parotitis
22. During an ectopic pregnancy in the uterine wall of the pipe through a small
hole, which is overweighted with packages tamponirovalos blood. What is the
name of such a complication?
A. Blood drop
B. *Covered the gap
C. Cystic drop
D. Incomplete tubal abortion
E. Full Pipe abortion
23. In microscopic examination revealed endometrial villi chorion, the
convolution of blood decidual tissue. What is the process of being developed?
A. Glandular hyperplasia of endometrial
B. Iron-cystic hyperplasia of endometrial
C. *Placental polyp
D. Fibrous polyp
E. Adenomatous polyp
24. In the microscopic study revealed endometrial villi of chorion, the
convolution of blood decidual tissue. What is the process of being developed?
A. Glandular hyperplasia of endometrial tissue
B. Iron-cystic hyperplasia endometrial tissue
C. *Placental polyp
D. Fibrous polyp
E. Adenomatous polyp
25. In the microscopic examination of endometrial tissue revealed diffuse
leukocyte infiltration. What is the process of being developed?
A. Placental polyp
B. Horionepithelioma
C. *Endometritis
D. Septic endometritis
E. Glandular hyperplasia of endometrium
26. A 56 year old man was taken to the hospital with complaints of general
weakness, pain and burning in the region of tongue, extremity numbness. In the
past he had resection of cardiac part of ventricle. Blood test: Hb- 80 g/L; RBC2,0*1012/L; colour index of blood- 1,2; leukocytes - 3,5*109/L. What type of
anemy is it?
A. A. Iron-deficient
B. Hemolytic
C. Aplastic
D. * В12 folic-deficient
E. Posthemorrhagic
27. A 7 year old child had an acute onset of disease. Pediatrician stated that
mucous membrane of fauces is hyperemic and covered with a lot of mucus.
Mucous membrane of cheeks has whitish stains. Next day the child's skin of
face, neck, body was covered with coarsely-papular rash. What disease may be
presumed?
A. Allergic dermatitis
B. Diphteria
C. Scarlet fever
D. * Measles
E. Meningococcemia
28. Microscopic analysis of tissue sampling from patient's skin reveals granulomas
that consist of epithelioid cells surrounded mostly by T-lymphocytes. Among
epithelioid cells there are solitary giant multinuclear cells of Pirogov-Langhans
typ. In the centre of some granulomas there are areas of caseous necrosis.
Blood vessels are absent. What disease are the described granulomas typical
for?
A. * Tuberculosis
B. Syphilis
C. Leprosy
D. Rhinoscleroma
E. Glanders
29. Autopsy of an 8 year old boy who was ill with pharyngeal and tonsillar
diphtheria and died one week after illness begin revealed myocardial changes
in form of small-focal myocardiocyte necroses, stroma edema with slight
lymphocytic infiltration. What type of myocarditis is it:
A. Septic
B. * Alterative
C. Granulomatous
D. Focal-intermediate, exudative
E. Interstitional
30. A 7 year old child was taken to the infectious disease hospital with complaints
of acute pain during swallowing, temperature rise up to 390С, neck edem.
Objective signs: tonsills are enlarged, their mucous membrane is plethoric and
covered with a big number of whitish-yellowish films that are closely adjacent
to the mucous membrane. After removal of these films the deep bleeding
defect remains. What type of inflammation is it?
A. Purulent
B. Crupous
C. Serous
D. Hemorrhagic
E. * Diphteritic
31. Autopsy of a man who died from ethylene glycol poisoning revealed that his
kidneys are a little bit enlarged, edematic; their capsule can be easily remove.
Cortical substance is broad and light-grey. Medullary substance is dark. What
pathology had this man?
A. * Necrotic nephrosis
B. Acute tubular-interstitial nephritis
C. Lipoid nephrosis
D. Acute glomerulonephritis
E. Acute pyelonephritis
32. Recovery from an infectious disease is accompanied by neutralization of
antigens by specific antibodies. What cells produce them?
A. * Plasmocytes
B. Tissue basophils
C. Fibroblasts
D. T-lymphocytes
E. Eosinophils
33. A patient with an acute myocarditis has the clinic presentations of cardiogenic
shock. What pathogenetic mechanism plays the main part in shock
development?
A. Depositing of blood in veins
B. Decrease of diastolic flow to the heart
C. * Disorder of pumping ability of heart
D. Increase of vascular tone
E. Decrease of vascular tone
34. Histologic examination revealed in all layers of appendix a big number of
polymorthonuclear leukocytes; hyperemia, stases. What disease are these
symptoms typical for?
A. Superficial appendicitis
B. * Phlegmonous appendicitis
C. Gangrenous appendicitis
D. Chronic appendicitis
E. Simple appendicitis
35. A 20 year old patient complains of excessive thirst and urinary excretion up to
10 L a day. The level of glucose in blood is normal, there is no glucose in
urine. What hormone deficit can cause such changes?
A. Insulin
B. Cortisol
C. Oxytocin
D. * Vasopressin
E. Triiodothyronine
36. Dystrophic changes of heart are accompanied by dilatation of cardiac cavities,
decrease of heart beat force, increased volume of blood that remains in cardiac
cavity after systole; veins are overfille. What state is this presentation typical
for?
A. Cardiac tamponade
B. Tonogenic dilatation
C. Cardiosclerosis stage
D. Emergency phase of myocardial hypertrophy
E. * Myogenic dilatation
37. The symptoms of regeneration process (callus) on the place of fracture were
revealed at the histologic specimen of tubular bon. What tissue forms this
structure?
A. * Fibrous bone tissue
B. Loose connective tissue
C. Reticular tissue
D. Epithelial tissue
E. Lamellar bone tissue
38. During the experimental analysis of chondrohistogenesis a sclerotome was
damage. What cells will it make impossible to differentiate?
A. Smooth myocytes
B. Fibroblasts
C. Epidermocytes
D. * Chondroblasts
E. Myoblasts
39. After a psychoemotional stress a 48 year old patient had a sudden attack of
acute heart pain with irradiation to the left hand. Nitroglycerine suppressed
pain in 10 minutes. What pathogenetic mechanism is principal for the pain
development?
A. Increased need of myocardium in oxygen
B. Dilatation of peripheral vessels
C. Embarrassement of coronary vessels
D. * Spasm of coronary vessels
E. Coronary vessel occlusion
40. Skin samples of a patient with bronchial asthma revealed allergen sensitization
of poplar fuzz. What factor of immune system plays the main part in
development of this immunopathological state?
A. Sensitized Т-lymphocytes
B. IgM
C. IgD
D. * IgE
E. 41. A patient has the following diagnosis: renal hypertension. What is the initial
pathogenetic factor of arterial hypertension development in this case?
A. * Renal ischemia
B. Intensified renin synthesis
C. Hypernatremia
D. Intensified angiotensin synthesis
E. Hyperaldosteronism
42. Patient with pigmentary xeroderma are characterized by anamalously high
sensivity to ultraviolet rays that causes skin cancer as a result of enzyme
systems incapability to restore damages of hereditary apparatus of cells. What
process abnormality is this pathology connected with?
A. DNA recombination
B. * DNA reparation
C. Genetic complementation
D. Genetic conversion
E. DNA reduplication
43. A one year old child has enlarged head and belly, retarded cutting of teeth,
destruction of enamel structur. What hypovitaminosis causes these changes?
A. Hypovitaminosis С
B. Hypovitaminosis В2
C. Hypovitaminosis А
D. Hypovitaminosis В1
E. * Hypovitaminosis D
44. What vitamin deficit causes the simultaneous disorder of reproductive function
and dystrophy of skeletal musculature?
A. Vitamin К
B. * Vitamin Е
C. Vitamin В1
D. Vitamin D
E. Vitamin А
45. A man had an acute onset of disease, he complained of chill, temperature rise
up to 400С, headache, cough, dyspnoe. On the fifth day of illness he died.
Autopsy revealed: his lungs were enlarged, they had a look of "coal-miner's
lungs". What illness is such postmortem diagnosis typical for?
A. Respiratory syncytial infection
B. Adenovirus infection
C. * Influenza
D. Croupous pneumonia
E. Multiple bronchiectasis
46. A patient with chronic glomerulonephritis has disorder of incretory function of
kidneys. What blood elements deficit will result from it?
A. Erythrocytes and leukocytes
B. * Erythrocytes
C. Leukocytes and thrombocytes
D. Thrombocytes
E. Leukocytes
47. The lung hypertension and cardiac insufficiency of right ventricle with ascites
and edemata developed at patient with pneumosclerosis. What is the principal
pathogenetic mechanism of edemata development?
A. Decrease of osmotic blood pressure
B. Increase of oncotic pressure of intercellular fluid
C. Increase of vascular permeability
D. Decrease of oncotic blood pressure
E. * Increase of hydrostatic blood pressure in veins
48. A 46 year old patient was admitted to the hematological department. It was
found that he had disorder of granulocytopoesis and thrombocytogenesis
processes. In what organ does this pathological process take pace?
A. Lymphatic ganglion
B. Thymus
C. Spleen
D. Palatine tonsil
E. * Red bone marrow
49. Two days after myocardial infarction a patient had a sudden systolic pressure
decrease up to 60 mm, tachycardia up to 140/min, dyspnea; the patient lost
consciousness. What mechanism is principal for the shock pathogenesis?
A. * Decrease of cardiac volume
B. Decrease of circulating blood volume
C. Anaphylactic reaction
D. Paroxysmal tachycardia
E. Intoxication
50. A patient has the sudden decrease of Са2+ content in blood. What hormone
secretion will increase?
A. Thyrocalcitonin
B. * Parathormone
C. Vasopressin
D. Aldosterone
E. Somatotropin
51. The chemical burn of esophagus caused its local constriction as a result of scar
formation. What cells of loose connective tissue take part in scar formation?
A. Young fibroblasts
B. Myofibroblasts
C. * Mature specialized fibroblasts
D. Fibrocytes
E. Fibroclasts
52. What substance makes saliva viscous and mucous, has protective function,
protects mucous membrane of oral cavity from mechanical damage?
A. Glucose
B. Amylase
C. Kallikrein
D. Lysozyme
E. * Mucin
53. During the embryogenesis of oral cavity the development of dental enamel was
disturbe. What source of dental development was damaged?
A. Dental saccule
B. Mesenchyma
C. Dental papilla
D. * Epithelium
E. Mesoderma
54. A patient with kidney disease has high blood pressure, espesially the diastolic.
Hypersecretion of what biologically active substance causes blood pressure
rise?
A. Catecholamines
B. Vasopressin
C. * Renin
D. Adrenaline
E. Noradrenaline
55. There is a 9 year old boy in endocrinological department, who has already had
a few fractures of extremeties caused by fragility of bones. Malfunction of
what endocrinous glands (gland) takes place?
A. Epiphysis
B. Thymus
C. Thyroid gland
D. * Parathyroid glands
E. Adrenal glands
56. A 22 year woman has enlarged lymphatic ganglions. Histological analysis of a
ganglion revealed lymphocytes, histiocytes, reticular cells, small and great
Hodgkin's cells, multinuclear Reed-Sternberg cells, solitary foci of caseous
necrosis. What disease are these changes typical for?
A. Cancer metastasis
B. Lymphosarcoma
C. * Lymphogranulematosis
D. Acute leukemia
E. Chronic leukemia
57. After recovering from epidemic parotiditis a patient began to put off weight, he
was permanently thirsty, drank a lot of water, had frequent urination, voracious
appetit. Now he has complaints of skin itch, weakness, furunculosis. His blood
contains: glucose - 16 mmole/L, ketone bodies - 100 mcmole/L; glucosuria.
What disease has developed?
A. * Insulin-dependent diabetes
B. Diabetes insipidus
C. Insulin-independent diabetes
D. Malnutrition diabetes
E. Steroid diabetes
58. Before teeth come out first on their roots appears a solid tissue that looks like
membrane reticulated bone. What tissue is it?
A. Enamel
B. * Cement
C. Loose fibrous connective tissue
D. Dentin
E. Dense fibrous connective tissue
59. A patient with adenoma of glomeral zone of adrenal cortex (Conn's disease)
has arterial hypertension, convulsions, polyuria. What is the main link in
pathogenesis of these disorders?
A. Glucocorticoid hyposecretion
B. Glucocorticoid hypersecretion
C. Catecholamine hypersecretion
D. * Aldosterone hypersecretion
E. Aldosterone hyposecretion
60. 15 minutes after a car accident examination of a 35 year old man revealed
massive injury of lower extremities without serious external loos of blood. The
victim is in excited stat. What component of pathogenesis of traumatic shock is
basic and requires urgent correction?
A. * Pain
B. Cardiac function disorder
C. Acute renal insufficience
D. Internal loss of plasma
E. Intoxication
61. A 65 year old patient suddenly die. She suffered from thrombophlebitis of deep
veins of shin. Autopsy revealed: trunk and bifurcation of pulmonary artery
contain red loose masses with dull corrugated surface. What pathological
process did the morbid anatomist reveal in pulmonary artery?
A. Foreign body embolism
B. Thrombosis
C. Fat embolism
D. * Thromboembolism
E. Tissue embolism
62. During the electronical microscopic analysis of salivary gland the cell fragmets
were revealed which are surrounded by a membrane and contain condensed
particles of nuclear substance and solitary organelles; the inflammatory
reaction around these cells is absent. What process is meant?
A. Karyopicnosis
B. * Apoptosis
C. Karyorhexis
D. Coagulation necrosis
E. Karyolysis
63. A patient complains of having urination disorder. He is diagnosed the
hypertrophy of prostate gland. What part of gland is damaged?
A. Right lobe
B. Apex
C. Base
D. Left lobe
E. * Median lobe
64. A man permanently lives high in the mountains. What changes of blood
characteristics can be found in his organism?
A. Decrease of hemoglobin content
B. Decrease of colour index of blood
C. * Increase of erythrocytes number
D. Decrease of reticulocytes number
E. Erythroblasts in blood
65. Microspecimen analysis of child's finger skin revealed that epidermis has signs
of inadequate development. What embryonal leaf was damaged in the process
of development?
A. Entoderma
B. Mezenchyma
C. Ectomezenchyma
D. Mesoderma
E. * Ectoderma
66. Autopsy of a woman who died of tumorous dissemination of mucinosous
cystadenocarcinoma and before that had to stay in bed for a long time revealed
big necrotic areas of skin and soft subjacent tissues in sacral region. What form
of necrosis is the case?
A. Sequester
B. Infarction
C. * Pressure sore
D. Zenker's necrosis
E. Caseous necrosis
67. A man who took part in disaster-management at a nuclear power plant had
hemorrhagic syndrome at the same time with acute radiation sickness. What is
the most important thing for the pathogenesis of this syndrome?
A. Low activity of anticoagulative blood system
B. * Thrombocytopenia
C. High activity of anticoagulative blood system
D. Destructed structure of vessel walls
E. High activity of fibrinolysis factors
68. A woman after labor lost 20 kg of body weight, her hair and teeth fall out, she
has muscle atrophy (hypophysial cachexia). Synthesis of what hypophysis
hormone is disturbed?
A. Gonadotropic
B. Corticotrophic
C. * Somatotropic
D. Prolactin
E. Thyreotropic
69. A patient who suffered form syphilis took a course of antibiotic therapy and
fully recovere. Some time later he was infected again with Treponema
pallidum. What form of infection is it?
A. Superinfection
B. Recurrence
C. Secondary infection
D. * Reinfection
E. Complication
70. During the histologic lung analysis of a man who died from cardiac
insufficiency the inflammation focuses were reveale. Alveoles were full of
light-pink fluid, here and there with pinkish fibers that formed a close-meshed
reticulum with a small number of lymphocytes. What type of exudate is present
in lungs?
A. Fibrinous
B. Hemorrhagic
C. Serous
D. * Serofibrinous
E. Purulent
71. A 53 year old patient consulted a doctor about white patch on the mucous
membrane of tongue. This patch sticks out from the mucous membrane, its
surface is cracked. Microscopic analysis reveals thickening of multilayer
epithelium, parakeratosis and acanthosis. What is the most probable diagnosis?
A. Geographic tongue
B. Papilloma
C. Median rhomboid glossitis
D. Epidermoid cancer
E. * Leukoplakia
72. After consumption some tinned meat a patient had diplopia, acute headache,
deglutition disorder, hard breathing, muscle weakness. The diagnosis was
botulism. What factor of pathogenicity are the clinic presentations of this
disease connected with?
A. Fibrinolysin
B. Plasmocoagulase
C. * Exotoxin
D. Endotoxin
E. Hemolysin
73. A patient's preliminary diagnosis is toxoplasmosis. What material was used for
diagnostics of this disease?
A. * Blood
B. Sputum
C. Feces
D. Urine
E. Duodenal contents
74. A 30 year old patient who was taken to the hospital with diagnosis acute
glomerulonephritis has proteinuria. What disorder caused this occurrence?
A. * Increased permeability of renal filter
B. Decreased number of functioning nephrons
C. Delayed excretion of nitrogen metabolism products
D. Increase of hydrostatic blood pressure in capillaries
E. Decreased oncotic pressure of blood plasma
75. During morphologic analysis of pulp floor three zones can be distinctly
differentiated: the one of softened dentin, transparent dentin and replacing
dentin. What stage of caries are these changes typical for?
A. Chronic caries
B. Deep caries
C. Stain stage
D. Superficial caries
E. * Median caries
76. The impact of oxitocine on uterus wall helps to stop uterine bleeding after
labor. What membrane of this organ reacts on the effect of this hormone?
A. Perimetrium
B. Endometrium
C. Parametrium
D. * Myometrium
E. Submucous membrane
77. A 4 year old child had Mantoux test. 60 hours after tuberculin introduction a
focal skin hardening and redness 15 mm in diameter appeare. It was regarded
as positive test. What type of hypersensitivity reaction is this test based upon?
A. Immune complex-mediated hypersensitivity
B. Immediate hypersensitivity
C. Complement-mediated cytotoxic hypersensitivity
D. E. * Delayed-type hypersensitivity
78. Microscopic analysis of tissue sampling from affected area of mucous
membrane of oral cavity revealed bacillus in form of accumulations that looked
like a pack of cigarettes. Ziehl-Neelsen staining gives them red colour. What
kind of pathogenic organism was most likely revealed in tissue sampling?
A. A.bovis
B. M.avium
C. M.tuberculosis
D. A.israilii
E. * M.leprae
79. During the histologic examination of thyroid gland of a man who died of
cardiac insufficiency together with hypothyroidism there was found the
diffusive infiltration of gland by lymphocytes and plasmocytes, parenchyma
atrophy and growth of connective tissue. Formulate a diagnosis:
A. Thyroid gland adenoma
B. Thyrotoxic goiter
C. * Hashimoto's thyroiditis
D. E. Purulent thyroiditis
80. During the tooth development the enamel organ has prismatic cells with
hexagonal intersection; the nucleus is situated in the central part of the cell.
What cells are meant?
A. Preodontoblasts
B. Cambial cells
C. * Preenameloblasts
D. Enamel pulp cells
E. Exterior enameloblasts
81. Microscopic analysis of brain base vessels of a patient who died of ischemic
stroke revealed that intima of cerebral vessels is irregular, with moderate
quantity of yellow stains and yellowish-whitish patches that narrow lumen.
What is the most probable diagnosis?
A. Nodular periarteritis
B. Diabetes mellitus
C. * Atherosclerosis
D. Rheumatism
E. Primary hypertension
82. While the examination of patient's oral cavity the dentist found xerostomia,
numerous erosions. What vitamin deficit caused this effect?
A. * Vitamin А
B. Vitamin Р
C. Vitamin К
D. Vitamin Н
E. Vitamin РР
83. A 10 year old child lives in the region where fluorine content in water is above
the mark. A dentist examined the child and found teeth damage in form of
chalky and also pigmentary stains and stripes. What is the most probable
diagnosis?
A. Wedge defects
B. * Fluorosis
C. Median caries
D. Tooth erosion
E. Acidic necrosis of hard tooth tissues
84. Histologic analysis of uterus mucous membrane revealed twisting glands,
serrated and spinned, they were extended by stroma growth with proliferation
of its cells. Formulate a diagnosis:
A. Cystic mole
B. Placental polyp
C. Leiomyoma
D. * Glandular hyperplasia of endometrium
E. Acute endometritis
85. The microscopic analysis of bronch biopsy revealed a tumor that consisted of
circumscribed accumulations of atypical cells of multylayer plane epithelium,
here and there with typical "pearls". What is the most likely diagnosis?
A. Solid carcinoma
B. * Epidermoid cancer with keratinization
C. Epidermoid cancer without keratinization
D. Scirrhus
E. Mucous carcinoma
86. Autopsy of a man who died of typhoid fever revealed ulcers along the ileum.
These ulcers have even sides, clean fundus formed by muscle layer or even by
serous tunic of an intestine. What stage of disease does the described
presentation correspond with?
A. Stage of medullary swelling
B. * Stage of "clean" ulcers
C. Stage of necrosis
D. Stage of "dirty" ulcers
E. Stage of ulcer healing
87. Tissue sample of soft palate arches that was taken because a tumor was
suspected (microscopic analysis revealed an ulcer with dense fundus) revealed
mucous membrane necrosis, submucous layer was infiltrated by lymphocytes,
epithelioid cells, plasmocytes, solitary neutrophils. There was also evident
endovasculitis and perivasculitis. What disease are these changes typical for?
A. * Primary syphilis
B. Vensan's ulcerative-necrotic stomatitis
C. Aphthous stomatitis
D. Faucial diphteria
E. ulcerative stomatitis
88. Autopsy of a 5 year old child revealed that pia maters of brain are extremely
plethoric, nebulous, have a look of yellowish-green "bonnet". Microscopic
analysis: pia mater of brain is very thickened, plethoric, impregnated with
purulent exudate containing fibrin. What disease is meant?
A. Anthrax
B. Measles
C. * Meningococcosis
D. Influenza
E. Tuberculosis
89. After a long-lasting and grave illness the blood pressure of a patient fell up to
60/40 mm; he has tachicardia, dyspnea, black-out. How can this state be
defined?
A. * Preagony
B. C. Apparent death
D. Shock
E. Agony
90. A woman has ovary hyperemia, increase of hematofollicular barrier
permeability with edema development, infiltration of follicle wall by
segmentonuclear leukocytes. The volume of follicle is big, its wall is thinned.
What period of sex cycle does the described picture correspond with?
A. * Preovulatory stage
B. Ovulation
C. Relative rest period
D. Menstrual period
E. Postmenstrual period
91. A 50 year old woman had her tooth extracte. The tissue regenerated. Which of
the following organelle are the most active during tissue regeneration?
A. Lysosomes
B. * Ribosomes
C. Agranular endoplasmic reticulum
D. Postlysosomes
E. Centrosomes
92.A dead man who suffered from a stomach ulcer for a long time died of massive
stomach hemorrhage (in the stomach and intestine there is more than 2,5 l
(0,66 gallon) of blood). What type of anaemia is represented?
A. acute post-hemorrhagic anaemia
B. chronic post-hemorrhagic anaemia
C. megaloblastic anaemia
D. *acute post-hemorrhagic anaemia.
E. hypochromic anaemia
93.A dead man who suffered from a stomach ulcer for a long time died of massive
stomach hemorrhage (in the stomach and intestine there is more than 2,5 l
(0,66 gallon) of blood). What is the colour of the skin and tissue of the
deceased?
A. yellow coloured and swelling
B. * pale coloured, anaemic.
C. . rose coloured and hot
D. . blue coloured and cold
E. . dark coloured with ecchymoses
94.A dead man who suffered from a stomach ulcer for a long time died of massive
stomach hemorrhage (in the stomach and intestine there is more than 2,5 l
(0,66 gallon) of blood). What microscopic changes will there be in his internal
organs?
A. swelling, dystrophic changes.
B. hypertrophied, serous exudation
C. amyloid accumulation, hypertrophy
D. * anaemic, dystrophic changes.
E. hyalinosis
95.A dead man who suffered from a stomach ulcer for a long time died of massive
stomach= hemorrhage (in the stomach and intestine there is more than 2,5 l
(0,66 gallon) of blood). What changes are there in the hemopoietic organs?
A. * hyperplastic changes
B. hypotrophy changes
C. atrophy changes
D. dystrophy changes
E. no changes
96.An ill woman aged 22 has icteric sclera and skin, and enlarged spleen. Blood
test, reveals decreased and falciform erythrocytes. What type of anaemia is
represented?
A. megaloblastic anaemia
B. anaemia by Adisson-Birmer
C. * hemolytic anaemia
D. hypochromic anaemia
E. posthaemorrhagic anaemia
97.An ill woman aged 22 has icteric sclera and skin, and enlarged spleen. Blood
test, reveals decreased and falciform erythrocytes. What is the name of the
disease?
A. В12 vitamin-deficit anaemia
B. Addison-Biermer anaemia
C. B12 achrestic anemia
D. neoplastic anaemia
E. * falciformcell anaemia
98.An ill woman aged 22 has icteric sclera and skin, and enlarged spleen. Blood
test, reveals decreased and falciform erythrocytes. What is the pathogenesis of
the disease?
A. * defective erythrocytes can't live along time
B. neoplastic processes lead to erythrocytes destroyed
C. defective erythrocytes are attacked by antibodies
D. defective erythrocytes are is phagocytized by macrophages
E. there are agglutination of erythrocytes
99.An ill woman aged 22 has icteric sclera and skin, and enlarged spleen. Blood
test, reveals decreased and falciform erythrocytes. What macroscopic changes
are there in the spleen?
A. hypoplastic & atrophy processes
B. neopplastic processes, necrosis & hemorrhages
C. * hyperplastic processes & hemosiderosis
D. splenomegaly and stasis of erythrocytes
E. blood & B-leucocytes accumulation
100.
An ill woman has metrorrhagia of more than two months and has not
sought medical help. There is increased weakness, paleness, tachycardia, and
dyspnoea. What type of anaemia is represented?
A. hemolytic
B. acute posthemorrhagic
C. * chronic posthemorrhagic
D. hypoplastic
E. deficiency
101.
An ill woman has metrorrhagia of more than two months and has not
sought medical help. There is increased weakness, paleness, tachycardia, and
dyspnoea. What changes are there in the circulating blood?
A. * decrease of erythrocytes, hemoglobin, nucleic (young) erythrocytes are
possible
B. decrease of erythrocytes, increase of hemoglobin and bilirubin
C. agglutination of erythrocytes, stasis in the microcirculatory vessels
D. hemolysis, hemoglobinosis
E. bilirubineamia, anisocytosis and poikilocytosis of erythrocytes are
possible
102.
A 70-year-old male patient with an expressed hepatosplenomegaly and
cachexia underwent a diagnostic puncture biopsy of his liver. A histological
examination revealed that along the portal tracts there were numerous
infiltrates of monomorphous round cells verified as prolymphocytes and Blymphocytes. What disease are the above changes characteristic of?
A.
B.
C.
D.
E.
Lymphosarcoma
Acute lymphoplastic leukaemia
Lymphogranulomatosis
*Chronic lymphatic leukaemia
Cesari's disease
103.
A death of a 7-year-old boy resulted from acute posthaemorrhagic
anaemia caused by a profuse bleeding from the gastrointestinal tract. A
postmortem examination revealed: macroscopically - an anaemia of the
internal organs, an enlargement of lymph nodes in different groups,
thymomegaly, a moderately manifested hepatosplenomegaly, a bright red bone
marrow; microscopically - a hypercellular bone marrow with some
monomorphous infiltrate of blast cells, diffuse-focal tumour infiltrates in the
liver, spleen, lymph nodes, meninges and substance of the brain. Make a
diagnosis for this form of leukaemia.
A. *Acute lymphoblastic
B. Acute myeloblastic
C. Acute stem cell
D. Acute monoblastic
E. Acute plasmablastic
104.
A histological examination of an enlarged cervical lymph node revealed
the following microscopic signs: proliferation of the lymphoid cells with
various degrees of maturity, presence of giant Hodgkin's and Reed-Sternberg
cells, as well as eosinophils, plasma cells and neutrophilic leukocytes, among
which there were foci of necrosis and fibrosis. Which of the variants of
lymphogranulomatosis listed below was the most probable?
A. With prevalence of the lymphoid tissue
B. Nodular sclerosis
C. *Mixed-cell variant
D. With suppression of the lymphoid tissue
E. Hodgkin's sarcoma
105.
A histological examination of an enlarged lymph node revealed a
proliferation of lymphocytes, histiocytes, reticular cells, acidophilic leukocytes,
small and large Hodgkin's cells, multinuclear Reed-Sternberg cells. Which of
the diseases listed below do the described morphological data correspond to?
A. Lymphosarcoma
B. Metastasis of carcinoma
C. Chronic leukaemia
D. Acute leukaemia
E. *Lymphogranulomatosis
106.
A male patient, who worked for a long period of time with petrol,
develops progressing anaemia and the haemorrhagic syndrome. A biopsy of his
breastbone reveals prevalence of a fatty tissue, and there are some small islets
of haemopoiesis with solitary cells of myelopoiesis. What is your diagnosis?
A. Chronic myeloleukosis
B. Pernicious anaemia
C. Haemolytic anaemia
D. *Hypoplastic anaemia
E. Aplastic anaemia
107.
A tumour was found in the locus of a pathological fracture of a rib in a
male patient. The case history contained information about persistent
proteinuria with presence of abnormal proteins of Bence-Jones type, as well as
presence of osteolytic foci in the bones of the spine, skull and pelvis.
Histologically, the tumour cells were represented by plasmablasts and
plasmacytes. What is your diagnosis?
A. Primary macroglobulinaemia
B. Heavy-chain disease
C. Osteosarcoma
D. *Multiple myeloma
E. Fibrosarcoma
108.
An autopsy of a female, who suffered from some blood disease (in her
clinical blood analysis leukocytosis achieved 100x109), revealed a pyoid bone
marrow (microscopically, it had myelocytes, promyelocytes and blast cells), an
enlarged spleen weighing up to 7 kg (microscopically, it had ischaemic infarcts
and an infiltration of the pulp by myelocytes), an enlarged liver weighing up to
6 kg (microscopically, it had a diffuse leukaemic infiltration of the sinusoids).
Name the diagnosis which was the most probable one of those listed below.
A. Acute granulocytic leukaemia
B. *Chronic granulocytic leukaemia
C. Acute stem cell leukaemia
D. Acute lymphocytic leukaemia
E. Multiple myeloma
109.
An autopsy of a male, who died from chronic renal insufficiency,
revealed numerous nodes with soft elastic consistency in the ribs, bones of the
vault of the skull and the breastbone. The osseous substance was decalcified
according to the nodes. The kidneys were enlarged, light grey, dense, their
section had some greasy lustre. What is your diagnosis?
A. Primary amyloid nephropathy
B. Parathyroid osteodystrophy
C. *Multiple myeloma
D. Osteoma
E. Osteosarcoma
110.
An autopsy of a male, who suffered from frequent fractures of his bones
and died from uraemia, revealed phenomena of osteoporosis and multiple
smooth-walled defects (as if produced by punching) in the bones of the skull,
ribs and spine. A microscopic examination of the bone marrow revealed its
diffuse infiltration by tumour cells of the lymphoplasmacytic line. Which of
the diagnoses listed below was the most probable?
A. *Multiple myeloma
B. Primary macroglobulinaemia
C. Heavy-chain disease
D. Paget's disease
E. Recklinghausen's disease
111.
An autopsy of a person, who died at a haematological department,
revealed numerous haemorrhages in the skin, mucous and serous membranes,
enlarged dark-grey tonsils. The lymph nodes of different localization were up
to 1.5 cm in diameter, soft, and grey-pink on section. The bone marrow of the
femur was dark red. Microscopically, the lymph nodes, liver, spleen and
kidneys contained leukaemic infiltrates consisting of lymphoblasts. It was
known that during the life-time the clinical blood analysis showed up to 3 x
10l2 of erythrocytes, 100x109 of leukocytes, a lot of lymphoblasts with
presence of mature forms and absence of prolymphocytes. Which of the
diagnoses listed below was the most probable?
A. Lymphogranulomatosis
B. Chronic lymphocytic leukaemia
C. *Acute lymphocytic leukaemia
D. Lymphosarcoma
E. Chronic granulocytic leukaemia
112.
An X-ray examination of a male patient revealed numerous foci of
osteoporosis and osteolysis in his flat bones. A high content of tumour plasma
cells was found in a trepanobiopsy. What is your diagnosis?
A. Acute monocytic leukaemia
B. Chronic myeloleukaemia
C. Osteosarcoma
D. *Multiple myeloma
E. Fibrosarcoma
113.
At autopsy of a man who suffered from frequent fractures and died of
uremia, in the bones of the skull, ribs and spine - the phenomenon of
osteoporosis and multiple "punched" defects. Microscopic examination of bone
marrow revealed its diffuse infiltration of tumor cells limfoplazmotsitarnogo
series. Which of the following diagnoses most reliable?
A. *Multiple myeloma
B. Primary makroglobulinemiya
C. Heavy chain disease
D. Paget's disease
E. Recklinghausen's disease
114.
At autopsy of the deceased in the hematological unit revealed multiple
hemorrhages in the skin, mucous and serous membranes, enlarged tonsils dark
gray color. The lymph nodes of various sites to 1,5 cm in diameter, soft,
grayish-pink in cross-section. The bone marrow of the femur dark red.
Microscopic examination of lymph nodes, liver, spleen and kidneys revealed
leukemic infiltration of lymphoblasts. Which of the following diagnoses most
reliable?
A. Lymphogranulomatosis
B. Chronic lymphocytic leukemia
C. *Acute lymphocytic leukemia
D. Lymphosarcoma
E. Chronic myeloid leukemia
115.
At autopsy the body of the deceased, who suffers from blood (complete
blood leukocytosis reached 100 ×109, found: pioidny bone marrow, increased
to 7 kg spleen (microscopically in it - ischemic heart attacks and infiltration of
the pulp myelocytes) increased to 6 kg of liver (microscopically in it - diffuse
leukemic infiltration of sinusoids). Which of the following diagnoses most
reliable?
A. Acute myeloid leukemia
B. *Chronic myelogenous leukemia
C. Acute undifferentiated leukemia
D. Acute lymphocytic leukemia
E. Multiple myeloma
116.
At necropsy males 78 years revealed patchy replacement of red bone
marrow is yellow. Lymph nodes, spleen reduced in size with marked atrophy
of the parenchyma. Such changes are characteristic:
A. Hemolytic anemia
B. Fanconi anemia
C. Anemia Ehrlich
D. *Ageing
E. Iron-deficiency anemia
117.
Autopsy of a woman, who died from renal insufficiency, revealed in her
spinal column, cranial bones and ribs some defects of the osseous tissue with
tumour nodes on their margins. The kidneys were enlarged, dense and "greasy"
on section. Microscopically, the tumour nodes and bone marrow were
characterized by a proliferation of tumour cells of the plasmacytic line. Which
of the diseases listed below corresponds to the description?
A. Metastasis of pulmonary carcinoma into bones
B. Osteosarcoma
C. *Multiple myeloma
D. Osteoporosis
E. Osteomyelitis
118.
During the microscopic examination of an enlarged cervical lymph node
14 years old girl was found: lymphoid follicles are missing, there are areas of
sclerosis and focal necrosis, cellular composition of the polymorphic site, there
are lymphocytes, eosinophils, atypical cells are large (Berezovsky-Sternberg
cells) and mononuclear cells, as large. What is the most likely diagnosis?
A. Burkitt's lymphoma
B. Acute lymphatic leukemia
C. Chronic lymphocytic leukemia
D. *Lymphogranulomatosis
E. Granulosarcoid
119.
Histological examination of enlarged lymph node revealed proliferation
of lymphocytes, histiocytes, reticulum cells, eosinophils, small and large cells
of Hodgkin's and Reed-Sternberg cells, whom of these diseases are responsible
described morphological data?
A. Lymphosarcoma
B. Cancer Metastasis
C. Chronic leukemia
D. Acute leukemia
E. *Lymphogranulomatosis
120.
In a patient with X-ray examination in the flat bones revealed multiple
cells of osteoporosis and osteolyzis. In trepanobioptate found a high content of
cancer of plasma cells. Your diagnosis.
A. Acute monocytic leukemia
B. Chronic myelogenous leukemia
C. *Multiple myeloma
D. Lymphogranulomatosis
E. Histiocytosis
121.
Patients in both jaws radiologically detected numerous defects in the
form of round holes with smooth walls. Histologically: the phenomenon
osteolizisa and osteoporosis in the background of weak bone formation. In the
urine - protein Bence-Jones. Name the disease.
A. Chronic myeloid leukemia.
B. *Myeloma disease.
C. Chronic erythroleukemia.
D. Acute undifferentiated leukemia.
E. Acute myeloid leukemia.
122.
The patient with the dental examination revealed atrophy of the mucous
membrane of the tongue with red spots (Gunter’s glossitis). The sclera is
yellowish. Color index greater than unity. For what anemia is characterized by
these changes?
A. Pernicious
B. Hemolytic
C. Chronic posthaemorrhagic
D. Iron.
E. 123.
The patient, who long worked with petrol, progressing anemia and
hemorrhagic syndrome. In the biopsy of sternum predominant adipose tissue,
revealed a few small islands with isolated blood cells myelopoiesis. Your
diagnosis.
A. Chronic myeloid leukemia
B. Pernicious anemia
C. Hemolytic anemia
D. *Hypoplastic
E. Iron deficiency anemia
124.
Thoracotomy in a 55-year-old male patient revealed a packet of lymph
nodes in the anterior mediastinum; a biopsy was taken from one of them.
Microscopically, there were infiltrates consisting of lymphocytes, histiocytes,
eosinophils and Reed-Stemberg multinuclear cells which were surrounded by
vegetations of a fibrous connective tissue. Name the clinical-morphological
form of lymphogranulomatosis.
A. Lymphogranulomatosis with suppression of the lymphoid tissue
B. Mixed-cell variant of lymphogranulomatosis
C. * Lymphogranulomatosis, nodular sclerosis
D. Lymphogranulomatosis with prevalence of the lymphoid tissue
E. -
125.
When imaging in a patient in the mediastinum were found enlarged
lymph nodes. Histological examination of the lymph nodes detected circular
growths of connective tissue that surrounded granulemopodobnye formation of
lymphocytes, plasma cells and large Berezovsky-Sternberg cells. What is the
most likely diagnosis?
A. Tuberculosis
B. Lymphosarcoma
C. *Lymphogranulomatosis
D. Sarcoidosis
E. Lymphoid
126.
A 40-year-old female patient underwent an operation of thyroidectomy.
A histological examination of the thyroid tissue revealed that its follicles
differed in size, contained some foamy colloid, the follicular epithelium was
high and in some places it formed papillae. The stroma of the gland had
clusters of lymphocytes which formed follicles with light centres. Make a
diagnosis of the disease of the thyroid gland.
A. *Toxic goiter
B. Hashimoto's disease
C. Ligneous thyroiditis
D. Acute nonsuppurative thyroiditis
E. Nodular goiter
127.
A histological examination of the thyroid gland revealed a significant
infiltration of its tissue by lymphocytes, formation of lymphoid follicles, an
atrophy of parenchymatous elements and a significant vegetation of the
connective tissue. What disease is characterized by this picture?
A. Colloid goiter
B. Endemic goiter
C. *Hashimoto's disease
D. Diffuse toxic goiter
E. Parenchymatous goiter
128.
An autopsy of a young female, who died from adrenal insufficiency,
revealed diffuse hypermelanosis of the skin, hyperplasia of the cells in islets of
Langerhans in the pancreas, the adrenal glands were sharply reduced in size
and their thinned cortical substance had foci of necrosis, haemorrhages and
sclerosis. What is your diagnosis?
A. Waterhouse-Friderichsen syndrome
B. Primary aldosteronism
C. Cushing's disease
D. *Addison's disease
E. Pheochromocytoma
129.
In a young male, an abundant quantity of the somatotropic hormone and
enlargement of the nose, lips, ears, lower jaw, hands and feet were revealed.
What is your diagnosis?
A. Pituitary dwarfism
B. Cushing's disease
C. Addison's disease
D. Adiposogenital dystrophy
E. *Acromegaly
130.
In a male patient with an increased level of the parathormone, a
histological examination in the area of a pathological fracture of his femur
revealed foci of a lacunar resolution of the osteoid beams and new formation of
a fibrous tissue. What is your diagnosis?
A. Multiple myeloma
B. *Parathyroid osteodystrophy
C. Osteoblastoclastoma
D. Paget's disease
E. Osteopetrosis
131.
A male patient with phenomena of hypothyroidism died from heart
failure. On histological examination, his thyroid gland revealed a diffuse
infiltration of the gland by lymphocytes and plasmacytes, an atrophy of the
parenchyma and a vegetation of the connective tissue. What disease was it?
A. Acute nonsuppurative thyroiditis
B. Ligneous thyroiditis
C. *Hashimoto's disease
D. Nodular goiter
E. Toxic goiter
132.
An autopsy of a 45-year-old female, who was suffering from arterial
hypertension, diabetes mellitus and ovarian dysfunction during past 15 years,
revealed obesity by the upper type, a pituitary basophil adenoma in the anterior
lobe of the hypophysis, hyperplasia of the adrenal cortex. Which of the
diagnoses listed below was the most probable?
A. *Cushing's disease
B. Cushing's syndrome
C. Hypertensive disease
D. Addison's disease
E. Adiposogenital dystrophy
133.
A 50-year-old female took medical advice complaining of excretion of a
large amount of urine and excessive thirst. On examination, her nourishment
was reduced, the skin was dry, density of the urine ranged from 1001 to 1010,
data of an ultrasound examination and computed tomography of the brain
revealed a tumour in the posterior lobe of the hypophysis. Indicate the most
probable disease.
A. Acromegaly
B. Babinsky-Frelich disease
C. *Diabetes insipidus
D. Simmonds disease
E. Cushing's disease
134.
A 46-year-old male patient, who suffered from bulimia, polydipsia,
polyuria with glucosuria and albuminuria, died of renal insufficiency. On
autopsy, the kidneys were reduced in size, dense and had a fine-grained
surface. The pancreas was reduced and partially substituted for a fatty tissue.
Microscopically, islets of Langerhans were fine, in some places they were
substituted for a connective tissue, solitary ones were hypertrophic. The
kidneys reveal intracapillary glomerulosclerosis. Which of the diagnoses was
the most probable?
A. Subacute glomerulonephritis
B. *Diabetes mellitus
C. Chronic indurative pancreatitis
D. Diabetes insipidus
E. Chronic glomerulonephritis
135.
A 36-year-old female patient underwent resection of the both lobes of
her thyroid gland; each of them was 5 x 6 cm in size, pink-yellow, moderately
dense and had a tuberous surface. A microscopic examination revealed follicles
of various size, some of them were dilated like cysts and filled with some
colloid; the follicular walls were lined with the smoothed cuboidal epithelium;
the stroma of the gland was redundantly developed owing to the connective
tissue, there were foci of calcinosis. Which of the diseases listed below
corresponded most to the changes found?
A. Parenchymatous goiter
B. *Colloid goiter
C. Toxic goiter
D. Hashimoto's disease
E. Ligneous thyroiditis
136.
An autopsy of a 24-year-old female (from her case history it is known
that a year before the woman had given birth to a child) revealed a sharp
decrease of the body weight down to 38 kg, the skin was dry and thin, the
weight of the internal organs was lowered. Also, there was a sharp decrease in
the weight of the adenohypophysis, the latter had cicatrices; there were foci of
dystrophy, necrobiosis and hyalinosis in the diencephalon. The ovaries, thyroid
and adrenal glands had phenomena of hypotrophy, the mucous membrane of
the intestines was atrophied. Which of the diagnoses was the most probable?
A. Nutritional dystrophy
B. Suprarenal cachexia
C. Cachexia associated with chronic amoebiasis
D. Cachexia associated with pellagra
E. *Cerebrohypophysial cachexia
137.
An autopsy of a 45-year-old female patient, who suffered from obesity
by the upper type, steroid diabetes mellitus, arterial hypertension and
secondary ovarian dysfunction, revealed hypertrichosis, hirsutism, striae on the
skin of the thighs and abdomen. The anterior lobe of the hypophysis contained
a white-pink encapsulated tumour, 2.5 cm in diameter (microscopically, it was
a pituitary basophil adenoma); the adrenal glands were characterized by
bilateral hyperplasia of the fascicular layer. Which of the diagnoses was the
most probable?
A. Cushing's syndrome
B. Adiposogenital dystrophy
C. *Cushing's disease
D. Simmonds disease
E. Pituitary dwarfism
138.
A 52-year-old male died from renal insufficiency. On microscopic
examination of his organs, the pancreas revealed lipomatosis and sclerosis with
an atrophy of islets of Langerhans, the kidneys had hyalinosis of the
mesangium and glomeruli (Kimrnelstiel-Wilson syndrome) and a glycogenic
infiltration of the epithelium of the tubules, the liver was characterized by fatty
degeneration. Which of the diagnoses listed below was the most probable?
A. *Diabetic glomerulosclerosis
B. Arterial nephrosclerosis
C. Amyloid shrunk kidneys
D. Chronic glomerulonephritis
E. Goodpasture's syndrome
139.
An autopsy of a male, who died from chronic renal insufficiency,
revealed atherosclerosis of the aorta and large arteries, small and dense kidneys
with a finegrained surface, an enlarged yellow-brown and flaccid liver, the
pancreas was reduced in size. Microscopically, there was atherocalcinosis of
the aorta and arteries, an atrophy of the parenchyma, sclerosis and lipomatosis
of the pancreas; the kidneys were characterized by hyalinosis of the
mesangium and glomeruli, a glycogenic infiltration of the epithelium of the
tubules, with large-drop adiposis in the hepatocytes. What pathological process
took place in the kidneys?
A. *Diabetic nephrosclerosis
B. Arterial nephrosclerosis
C. Chronic pancreatitis
D. Chronic glomerulonephritis
E. Steatosis
140.
On autopsy of a male, who died from uraemia, it was found that the
pancreas was reduced in size, his contracted kidneys had a fine-grained
surface, the liver was enlarged, yellow and flaccid. Microscopically, the
pancreatic tissue revealed an atrophy of the parenchyma, including islets of
Langerhans, the atrophied parenchyma was substituted for hyperplastic
connective and fatty tissues. The kidneys were characterized by sclerosis and
hyalinosis of the glomeruli, as well as by a glycogenic infiltration of the
tubules; there was a fatty degeneration in the liver and a fibrinous
inflammation in the mucous coats of the trachea, bronchi and stomach. What
disease did the died person suffer from?
A. Chronic indurative pancreatitis
B. Chronic glomerulonephritis
C. Hypertensive disease
D. *Diabetes mellitus
E. Steatosis
141.
An autopsy of a male revealed a tumour in the anterior lobe of the
hypophysis, enlarged adrenal glands, a reduction of the gonads in size, a
hypertrophy of the left cardiac ventricle, the pancreas was reduced in size and
thickened. Histologically, there was a pituitary basophil adenoma and a
hyperplasia of the cortical layer in the adrenal glands. The pancreas was
characterized by a moderately expressed atrophy of the parenchyma, including
islets of Langerhans. What disease did the patient suffer from?
A.
B.
C.
D.
E.
Diabetes mellitus
Adiposogenital dystrophy
*Cushing's disease
Cushing's syndrome
Simmonds disease
142.
For a histological examination, a lobe and a part of the isthmus of the
thyroid gland were received. The tissue of the gland was dense and tuberous,
on section it was pale brown and had grey-whitish foci. Microscopically,
against a background of an atrophy of the follicles of the gland, there was some
diffuse lymphoplasmacytic infiltration of the stroma with formation of
lymphoid follicles. What pathological process were these changes typical for?
A. Toxic goiter
B. *Allergic thyroiditis
C. Thyroid adenoma
D. Colloid goiter
E. Sporadic goiter
143.
A histological examination of a thyroid gland revealed follicles of
various size and shape which were lined with the columnar epithelium; the
latter proliferated and formed papillae of various size. The follicular lumens
contained some liquid and vacuolized colloid. The stroma of the gland was
characterized by a lymphoplasmacytic infiltration, in some places with
formation of lymphatic follicles having light centres. Which of the diagnoses
was the most probable?
A. Colloid goiter
B. Nodular goiter
C. Hashimoto's disease
D. Ligneous thyroiditis
E. *Toxic goiter
144.
An autopsy of a 48-year-old male, who died from vascular collapse,
revealed an increased pigmentation of the skin, the adrenal glands were
reduced in size, the brown-yellow liver was enlarged. On histological
examination, foci of necrosis with a tuberculous granulation tissue were found
in the adrenal glands. The liver was characterized by phenomena of fatty
degeneration. Which of the diagnoses was the most probable?
A. *Addison's disease
B. Steatosis
C. Primary aldosteronism
D. Cushing's syndrome
E. Lipofuscinosis
145.
At autopsy in the lungs is determined by chronic bronchitis. The cavity is
lined with a flat focal bronchial epithelium. A manifestation of a pathological
process are changes in the bronchial mucosa?
A. *Metaplasia
B. Dispalasia
C. Hypoplasia
D. Hypertrophy
E. Hyperplasia
146.
At autopsy in the lungs is determined by the expansion of the bronchi.
The cavity is lined with enhanced bronchial columnar epithelium from the pus
in the lumen. A manifestation of a pathological process are changes in the
bronchial mucosa.
A. *Bronchiectasis
B. Acute bronchitis
C. Asthma
D. Bronchopneumonia
E. Hot abscess
147.
At autopsy men 40 years, who died from complications of influenza, the
signs of bronchopneumonia. Enter the localization of pathological changes in
this disease:
A. *Respiratory division lung
B. Pleura
C. Mediastinum
D. Pericardium
E. Upper respiratory
148.
At autopsy of the body men aged 35, who had abused alcohol, showed
signs of lobar pneumonia. Which organism is more often the cause of this type
of pneumonia?
A. Aeruginosa
B. Streptococcus
C. Staphylococcus
D. *Pneumococcus Frenkel
E. Chlamydia
149.
At autopsy the lungs revealed bronchiectasis. The cavity is lined with
enhanced bronchial columnar epithelium from the pus in the lumen. Changes
in the bronchial mucosa - an expression:
A. *Hypertrophy
B. Hyperplasia
C. Regeneration
D. Metaplasia
E. Organization
150.
An autopsy of a 45-year-old male patient, who had had double
bronchopneumonia and died under the phenomena of intoxication, revealed in
the lower lobe of the right lung some thick-walled cavity, 4 cm in diameter,
filled with liquid yellowish masses. What pathological process complicated the
course of pneumonia?
A. Tuberculoma
B. Gangrene
C. *Abscess
D. Sequester
E. Empyema
151.
At autopsy the woman's body 35 years, who died from complications of
influenza, the signs of bronchopneumonia. What goes through pathogen in
respiratory department of the lungs in this disease?
A. Contact
B.
C.
D.
E.
*Bronchogenic
Primary
Secondary
Enteral
152.
At autopsy the woman's body 48 years, who died from complications of
SARS, showed signs of bronchopneumonia. Specify the most common way of
penetration of the agent in this disease:
A. Hematogenous or lymphogenous
B. Lymphogenous
C. *Bronchogenic
D. Mixed
E. Enterogenny
153.
At bronchoscopy in men 40 years revealed hyperemia and edema of the
mucous membrane with the presence of small hemorrhages and mucus in the
lumen. Enter the diagnosis:
A. Lobar pneumonia
B. *Bronchitis
C. Bronchopneumonia
D. Lobar pneumonia
E. Intermediate pneumonia
154.
At necropsy men 50 years old, who suffered from chronic bronchitis,
revealed tumor-like formations around the right main bronchus. Histological
examination - clumps of cells laminated, flat epithelium with signs of
polymorphism and abundance of mitosis. What process has evolved in the
lungs?
A. Bronchiectasis
B. Emphysema
C. Chronic pneumonia
D. *Cancer
E. Pneumosclerosis
155.
At necropsy men who died of chronic pulmonary heart, were found
morphological signs hypertension pulmonary circulation. Which of these
symptoms most characteristic?
A. *Expansion of the right ventricle
B. Increased left ventricular
C. Expansion of the left and right ventricular
D. Expansion of the right atrium
E. Expansion of the left atrium
156.
At necropsy men who died of chronic pulmonary heart, were found
morphological signs of "pulmonary heart." Which of these symptoms most
characteristic?
A. *Expansion of the right ventricle
B. Increased left ventricular
C. Expansion of the left and right ventricular
D. Expansion of the right atrium
E. Expansion of the left atrium
157.
At the opening of men 50 years old, suffers from chronic bronchitis,
revealed tumor-like formations around the right main bronchus. Histological
examination - clumps of cells laminated, flat epithelium with signs of
polymorphism and abundance of mitosis. What is the etiological factor for this
disease?
A. *Smoking
B. Supercooling
C. Excessive insolation
D. Fluctuations in atmospheric pressure and temperature
E. Alcoholism
158.
Female 38 years old for several years suffered from atopic bronchial
asthma. What are the morphological changes were observed in the microscopic
examination of bronchial wall in the period between the attacks?
A. *Thickening of the basement membrane of epithelium
B. Thinning of the basement membrane of the epithelium
C. Thinning and sclerosis of the basement membrane of the epithelium
D. Pathological changes are absent
E. The changes only under the basement membrane
159.
Female 38 years old for several years, suffering from atopic bronchial
asthma. What pathogenetic mechanism is most likely?
A. Autoimmune reaction and hypersensitivity reaction with Ig A
B. Hypersensitivity reaction with Ig A
C. Hypersensitivity reaction with Ig M
D. *Hypersensitivity reaction with Ig E
E. Autoimmune process
160.
In patients with final stage chronic renal failure during bronchoscopy
revealed hyperemia and edema of the mucosa with the presence of small
hemorrhages and mucus in the lumen of the bronchi. Name of these changes?
A. Bronchiectasia
B. Precancerous condition
C. Cancer
D. Bronchiolitis
E. *Bronchitis
161.
In the pulmonary section of a patient admitted with a diagnosis of
chronic nonspecific lung disease. Which of the following diseases the patient
suffered:
A. Lung cancer
B. Purulent pneumonia
C. *Bronchiectasis
D. Measles pneumonia
E. Respiratory distress syndrome
162.
Male 50 years died of chronic heart failure, which developed as a result
of bronchiectasis. During the autopsy of the lungs revealed multiple saccular
bronchiectasis. Specify the mechanism of their formation:
A. When coughing and decreases sharply when intrabronchial pressure in
areas of least resistance to change bronchial wall expands
B. Coughing when intrabronchial pressure rises sharply in most parts of the
bronchial wall of resistance to change extends
C. When coughing and decreases sharply when intrabronchial pressure in
areas of greatest resistance to change bronchial wall expands
D. Coughing when intrabronchial pressure rises sharply in areas of least
resistance decreases bronchomotor
E. *Coughing when intrabronchial pressure rises sharply in areas of least
resistance to change bronchial wall expands
163.
Male 65 years died from chronic heart failure. “Pulmonary heart”
developed as a result of occupational disease - silicosis. What could cause the
development of this disease:
A. *Prolonged inhalation of dust containing free silica SiO2.
B. Prolonged inhalation of coal dust
C. Prolonged inhalation of ambient dust
D. Prolonged inhalation of decomposition products of cellulose from cotton
E. Prolonged inhalation room and coal dust
164.
Male 65 years died of congestive heart failure, which developed as a
result chronic nonspecific lung diseases. During the autopsy revealed signs of
chronic bronchitis. Give frequent cause of this disease:
A. Hypothermia
B. *Smoking
C. Fluctuations in atmospheric pressure
D. Temperature drops
E. Excessive insolation
165.
Male 65 years died of congestive heart failure, which developed as a
result of bronchiectasis. During the autopsy the lungs showed signs of chronic
diffuse obstructive pulmonary emphysema. Give frequent cause of this disease:
A. Gangrene of the lung
B. Mediastenitis
C. Hamman-Rich syndrome
D. Acute bronchitis
E. *Chronic diffuse bronchitis
166.
Male 70 years died from chronic heart failure, “lung heart”, which
developed as a result of occupational disease - anthracosis. What could cause
the development of this disease:
A. Prolonged inhalation of dust containing free silica SiO2.
B. Prolonged inhalation of ambient dust
C. Prolonged inhalation of decomposition products of cellulose from cotton
D. Long-term smoking
E. *Prolonged inhalation of coal dust
167.
Patient age 67 died from chronic heart failure due to chronic nonspecific
lung diseases. At autopsy of the body revealed signs of chronic bronchitis.
Which of the chemical factors could contribute to its development:
A. *A pair of chlorine, nitric oxide
B. Carbon dust
C. Room dust
D. Ultraviolet irradiation
E. Fluctuations in atmospheric pressure
168.
Patient age 67 died from chronic heart failure due to chronic nonspecific
lung diseases. At autopsy of the body revealed signs of focal ill
pneumosclerosis. After a pathological process it developed?
A. Acute bronchitis
B. Fibrinous pleurisy
C. Emphysema
D. Lobar pneumonia
E. *Abscesses
169.
Patient age 67 died from chronic heart failure due to chronic nonspecific
lung diseases. At autopsy of the body revealed signs of diffuse reticular
pneumosclerosis. After a pathological process it developed?
A. *Unfinished bronchopneumonia
B. Acute bronchitis
C. Fibrinous pleurisy
D. Emphysema
E. Abscess
170.
Patients suffering from chronic bronchitis, X-ray revealed darkening
around the right main bronchus. Histological examination revealed the
accumulation of cells multilayered, squamous epithelium with the phenomena
of polymorphism and a large number of mitoses. What process has evolved in
the lungs?
A. Bronchiectasis
B. Emphysema
C. Chronic pneumonia
D. *Cancer
E. Pneumosclerosis
171.
The patient admitted to hospital with high fever, cough, with lots of
smelly sputum. X-rays revealed a cavity with fluid level. Performed
lobectomy. Macroscopically revealed pus filled cavity, which communicates
with the bronchus. Enter the diagnosis:
A. *Abscess
B. Lung
C. Chronic bronchitis
D. Acute bronchitis
E. Empyema
172.
The patient 60 old years, died of chronic heart failure due to chronic
nonspecific lung diseases. At autopsy the body showed signs of chronic
pulmonary emphysema. As can be seen after opening the chest?
A. *Lungs unabated
B. Lung collapses
C. Slightly reduced in volume
D. Lungs partially reduced in volume
E. Collapses only one lung
173.
The patient was admitted to hospital with high fever, cough, an
abundance of foul-smelling sputum. X-rays revealed a cavity with fluid level.
Performed lobectomy. Macroscopically revealed pus filled cavity, which
communicates with the bronchus. What kind of disease a patient?
A. Bronchiectasis
B. Emphysema
C. Chronic pneumonia
D. *Lung abscess
E. Pneumosclerosis
174.
The patient X-ray revealed darkening around the right main bronchus.
Histological examination of biopsy revealed accumulation of cells laminated,
squamous epithelium with the phenomena of polymorphism and a large
number of mitoses. As a result, diagnosed with lung cancer. What was the
cause of this pathology?
A. Acute bronchitis
B. Emphysema
C. Atypical form of pneumonia
D. *Chronic bronchitis
E. Lobar pneumonia
175.
X-rays revealed a cavity with fluid level. Performed lobectomy.
Macroscopically vychvlyaetsya pus filled cavity, which communicates with the
bronchus. What a complication often develops in this case?
A. *Amyloidosis
B. Cancer
C. chronic bronchitis
D. Acute bronchitis
E. Empyema
176.
A 39-year-old female with a clinical picture of acute abdomen
underwent surgical removal of an enlarged uterine tube. On examination, the
serous coat of the uterine tube was dark purple, the lumen contained some
blood clots. A histological examination of the wall of the tube revealed that the
mucous membrane had layers of the decidual cells, and there were villi of the
chorion among the blood clots. What is the most probable diagnosis?
A. *Tubal pregnancy
B. Placental polyp
C. Choriocarcinoma
D. Haematosalpinx
E. Salpingitis
177.
A histological examination of a scrape from the uterine cavity of a 45year-old woman with clinical manifestations of a disordered ovariomenstrual
cycle revealed that the number of the endometrial glands was increased, the
lumens in some of them were significantly dilated and rounded, the epithelium
of the glands had morphological signs of the phase of proliferation. What is
your diagnosis?
A. Glandular hyperplasia of endometrium
B. Atypical hyperplasia of endometrium
C. Glandular polyp of endometrium
D. *Glandulocystic hyperplasia of endometrium
E. Endometrial adenocarcinoma
178.
On gynaecological examination, a red area, 0.7 cm in diameter, was
found in the vaginal portion of the uterine cervix. A histological examination
of a biopsy from the uterine cervix revealed that the squamous epithelium was
substituted for the columnar one, as well as accumulation and new formation of
glands under the integmentary epithelium were observed. What is your
diagnosis?
A. Adenosis
B. Simple endocervicosis
C. *Proliferative endocervicosis
D. Papillary endocervicosis
E. Healing endocervicosis
179.
Microscopically, a scrape from the uterine cavity, taken in a 36-year-old
female against a background of uterine bleeding, revealed a neoplasm which
consisted of a large number of light epithelial cells of Langhans and
multinuclear symplasts, the number of figures of mitosis was increased. The
stroma was absent, the vascular cavities were lined with the above cells. Make
a diagnosis of the uterine tumour.
A. *Choriocarcinoma
B. Endometrial polyp
C. Endometrial adenocarcinoma
D. Simple hydatidiform mole
E. Invasive hydatidiform mole
180.
A histological examination of the vaginal portion of the uterine cervix
revealed substitution of the stratified squamous epithelium for the simple
columnar one with numerous underlying glands without any signs of their new
formation. Which of the diagnoses listed below was the most probable?
A. Papillary endocervicosis
B. *Simple endocervicosis
C. Progressing endocervicosis
D. Healing endocervicosis
E. Adenosis
181.
A scrape from the uterine cavity revealed in the blood some particles of a
neoplasm which had an organoid structure and consisted of the columnar
epithelial cells which formed glandular structures; the stroma of the neoplasm
was significantly developed, the area of its "pedicle" had glomi of thick-walled
vessels. Which of the tumours was the most probable?
A. Uterine adenocarcinoma
B. Glandular hyperplasia of endometrium
C. Uterine fibroadenoma
D. Endometriosis
E. *Endometrial polyp
182.
A 34-year-old female took medical advice of a gynaecologist
complaining of a uterine haemorrhage for 10 days after her menses. A scrape
from the uterine cavity revealed a large number of the endometrial glands
which were spirally convoluted, dichotomically divided and ramified in a treelike manner. The stroma of the endometrium was hyperplastic and had a great
number of predecidual cells. Which of the diagnoses was the most probable?
A.
B.
C.
D.
E.
Atypical hyperplasia of endometrium
Mixed hyperplasia of endometrium
*Glandular hyperplasia of endometrium
Glandulocystic hyperplasia of endometrium
Remnants of abortion
183.
A mammary gland of a 25-year-old female revealed a slightly tender
node, 3 cm in diameter; a sectorial resection of the mammary gland was made.
Macroscopically, the node was whitish and had round hollows 0.1-0.3 cm in
diameter. Microscopically, there was a vegetation of the connective tissue with
foci of hyalinosis, an atrophy of the glandular lobules and dilation of the
lumens in some places. What disease are the above morphological changes
typical for?
A. Fibrous mastopathy
B. *Fibrocystic mastopathy
C. Sclerosing adenosis
D. Fibroadenoma of mammary gland
E. Carcinoma
184.
Within the first stage of labour, the blood pressure in a female elevated
(220/110 mm Hg), she developed convulsions and a loss of consciousness. Her
death resulted from a sharp impairment of cerebral circulation. An autopsy
revealed jaundice, an intracerebral haemorrhage, a pulmonary oedema with
microfocal haemorrhages into the pulmonary parenchyma, an enlarged yellow
liver with numerous haemorrhages, enlarged flaccid kidneys with a swollen
cortical layer, where on microscopic examination symmetrical necroses were
found. Which of the diagnoses listed below was the most probable?
A. Haemorrhagic insult
B. Viral hepatitis
C. *Eclampsia
D. Necrotic nephrosis
E. Haemorrhagic pneumonia
185.
On gynaecological examination of a 36-year-old female, a red focus,
which was 0.8 x 0.5 cm in size and had uneven contours, was found in the
mucous coat of the vaginal portion of the uterine cervix on its border with the
mouth of the womb. A microscopically examination revealed substitution of
the stratified squamous epithelium for the columnar one, as well as
accumulation and new formation of glands from the cambial elements of the
columnar epithelium in the cervical channel were observed under the
integmentary epithelium. Which of the diagnoses listed below was the most
probable?
A. Simple endocervicosis
B. Adenomatosis of uterine cervix
C. Healingendocervicosis
D. *Proliferative endocervicosis
E. Dysplasia of epithelium of uterine cervix
186.
Several days following a myocardial infarction, a 51-year-old man
develops the sudden onset of a new pansystolic murmur along with a diastolic
flow murmur. Workup reveals increased left atrial pressure that develops late
in systole and extends into diastole. Which of the following is the most likely
cause of the abnormalities present in this individual?
A. Aneurysmal dilation of the left ventricle
B. Obstruction of the aortic valve
C. Rupture of the left ventricle wall
D. *Rupture of a papillary muscle
E. Thrombosis of the left atrial cavity
187.
Three weeks following a myocardial infarction, a 54-year-old man
presents with fever, productive cough, and chest pain. The pain is worse with
inspiration, better when he is sitting up, and not relieved by nitroglycerin.
Physical examination finds a friction rub along with increased jugular venous
pressure and pulsus paradoxus (excess blood pressure drop with inspiration).
Which of the following is the most likely explanation for these findings?
A. Caplan’s syndrome
B. *Dressler’s syndrome
C. Ruptured papillary muscle
D. Ruptured ventricular wall
E. Ventricular aneurysm
188.
A 59-year-old woman presents with increasing shortness of breath.
Physical examination reveals signs of left heart failure. She is admitted to the
hospital to workup her symptoms, but she dies suddenly. A section from her
heart at the time of autopsy reveals marked thickening of the wall of the left
ventricle, but the thickness of the right ventricle is within normal limits. Many
of the nuclei of the myocytes in the wall of the left ventricle have a “box car”
appearance. The endocardium does not appear to be increased in thickness or
fibrotic, and the cardiac valves do not appear abnormal. The left ventricular
cavity is noted to be decreased in size. What is the most likely cause of this
cardiac pathology?
A. Carcinoid heart disease
B. Cor pulmonale
C. Eccentric hypertrophy
D. *Systemic hypertensive
E. Volume overload
189.
A 71-year-old woman presents with increasing chest pain and occasional
syncopal episodes, especially with physical exertion. She has trouble breathing
at night and when she lies down. Physical examination reveals a crescendodecrescendo midsystolic ejection murmur with a paradoxically split second
heart sound (S2). Pressure studies reveal that the left ventricular pressure
during systole is markedly greater than the aortic pressure. Which of the
following is the most likely diagnosis?
A. Aortic regurgitation
B. *Aortic stenosis
C. Constrictive pericarditis
D. Mitral regurgitation
E. Mitral stenosis
190.
A 63-year-old man presents with signs of congestive heart failure,
including shortness of breath, cough, and paroxysmal nocturnal dyspnea.
Physical examination reveals a hyperdynamic, bounding, “waterhammer”pulse and a decrescendo diastolic murmur. His hyperdynamic pulse
causes “bobbing” of his head. Which of the following is the most frequent
cause of the cardiac valvular abnormality present in this individual?
A. Aortic dissection
B. Infective endocarditis
C. Latent syphilis
D. Marfan syndrome
E. *Rheumatic fever
191.
Physical examination of an asymptomatic 29-year-old woman with a
history of rheumatic fever during childhood finds an early diastolic opening
snap with a rumbling late diastolic murmur. Which of the following is the most
likely diagnosis?
A. Aortic regurgitation
B. Aortic stenosis
C. Mitral regurgitation
D. *Mitral stenosis
E. Pulmonic stenosis
192.
A 7-year-old boy presents with the acute onset of fever, pain in several
joints, and a skin rash. Physical examination finds an enlarged heart, several
subcutaneous nodules, and a skin rash on his back with a raised, erythematous
margin. Laboratory tests find an elevated erythrocyte sedimentation rate and an
elevated antistreptolysin O titers. Within the past month, this boy most likely
had which one of the following abnormalities?
A. Anitschkow cells develop in the lungs
B. Aschoff bodies develop in the skin
C. *Beta-hemolytic streptococci infection of the pharynx
D. Pseudomonas aeruginosa infection of the aorta
E. Stenosis of the mitral valve
193.
An autopsy done on a 23-year-old man who died suddenly with no
previous medical history reveals the right ventricle to be dilated with near total
transmural replacement of the right ventricle (RV) free-wall myocardium by
fat and fibrosis. No skin or hair abnormalities are seen. What is the best
diagnosis?
A. *Arrhythmogenic RV cardiomyopathy
B. Endocardial fibrosis
C. Hyper-serotonin RV syndrome
D. Loeffler endomyocarditis
E. Naxos syndrome
194.
A 31-year-old woman presents with fever, intermittent severe pain in the
left upper quadrant of her abdomen, and painful lesions involving her fingers
and nail beds. History reveals that she had acute rheumatic fever as a child and
that when she was around 20 years of age she developed a new cardiac
murmur. At the present time one of three blood cultures submitted to the
hospital lab grew a specific bacteria. Which of the following is the most likely
cause of her disease?
A. Staphylococcus aureus
* α-hemolytic viridans streptococci
Candida species
Group A streptococci
Pseudomonas species
195.
A 23-year-old woman develops the sudden onset of congestive heart
failure. Her condition rapidly deteriorates and she dies in heart failure. At
autopsy, patchy interstitial infiltrates composed mainly of lymphocytes are
found, some of which surround individual myocytes. Which of the following is
the most likely cause of this patient’s heart failure?
A. Autoimmune reaction (to group A β-hemolytic streptococci)
B. Bacterial myocarditis (due to S. aureus infection)
C. Hypersensitivity myocarditis (due to an allergic reaction)
D. Nutritional deficiency (due to thiamine deficiency)
E. *Viral myocarditis (due to coxsackievirus infection)
196.
At the time of autopsy of a 39-year-old woman who died of
complications of systemic lupus erythematosus, several medium-sized
vegetations are found on both sides of the mitral valve and tricuspid valve.
Which of the following is the basic abnormality that produced these cardiac
vegetations?
A. Turbulent blood flow through an incompetent mitral valve
B. Excess secretion of a vasoactive amine
C. *Presence of an anticardiolipin antibody
D. Cachexia produced by a hypercoagulable state
E. Bacterial colonization of an abnormal valve
197.
A 37-year-old woman presents with prolonged cramps, nausea,
vomiting, diarrhea, and episodic flushing of the skin. Additionally, she
develops pearly white, plaque-like deposits on the tricuspid valve leaflets.
Which of the following disorders is most likely to be present in this individual?
A. Rheumatic heart disease
B. Amyloidosis
C. Iron overload
D. Hypothyroidism
E. *Carcinoid heart disease
198.
A 59-year-old patient receiving chemotherapy with the anthracycline
Adriamycin develops severe heart failure. Sections from an endocardial biopsy
specimen reveal vacuolization of the endoplasmic reticulum of the myocytes.
Adriamycin therapy most frequently causes what type of cardiomyopathy?
A. *Dilated cardiomyopathy
B. Hyperplastic cardiomyopathy
C. Hypertrophic cardiomyopathy
D. Obliterative cardiomyopathy
E. Restrictive cardiomyopathy
199.
A 3-month-old girl is being evaluated for feeding difficulty and failure to
thrive. Physical examination finds pallor, peripheral cyanosis, tachypnea, and
fine expiratory wheezing. Chest x-ray shows cardiac enlargement. She is
admitted to the hospital, quickly develops severe cardiac failure, and dies 3
days after admission. At the time of autopsy the endocardium is found to have
B.
C.
D.
E.
a “cream cheese” gross appearance. Histologic sections from this area reveal
thickening of the endocardium due to a proliferation of fibrous and elastic
tissue. Which of the following is the most likely diagnosis?
A. Dilated cardiomyopathy
B. Hypertrophic cardiomyopathy
C. Infective endocarditis
D. Libman-Sachs endocarditis
E. * Restrictive cardiomyopathy
200.
A 49-year-old man 7 days after being admitted to the hospital for an
inferior wall, transmural myocardial infarction suddenly becomes short of
breath. Physical examination reveals hypotension, elevated jugular venous
pressure, and muffled heart sounds. His systemic blood pressure drops 13
mmHg with inspiration. Which one of the following pathologic processes
produced these clinical findings?
A. Acute inflammation of the pericardium due to an autoimmune reaction
B. Acute mitral regurgitation due to rupture of a papillary muscle
C. Acute suppurative inflammation of the pericardium due to bacterial
infection
D. *Blood accumulation in the pericardial cavity due to rupture of the
ventricular wall
E. Serous fluid accumulation in the pericardial cavity due to congestive
heart failure
201.
A 35-year-old man179. A 35-year-old man presents with weight loss,
fever, and fatigue. Physical examination finds signs and symptoms of mitral
valve disease. Further workup finds a pedunculated mass in the left atrium. The
tumor is resected and histologic sections reveal stellate cells in a loose myxoid
background. Which of the following is the most likely diagnosis?
A. Chordoma
B. Fibroelastoma
C. Leiomyoma
D. *Myxoma
E. Rhabdomyoma
202.
A 2-year-old girl is being evaluated for growth and developmental delay.
She has had several past episodes when she would suddenly have trouble
breathing, become blue, and then assume a squatting position to catch her
breath. Workup finds a defect in the wall of the ventricular septum, increased
thickness of the right ventricle, and dextroposition of the aorta. Which of the
following cardiovascular abnormalities is most likely to be present in this
child?
A. Coarctation of the aorta
B. Incompetence of the mitral valve
C. Patency of the foramen ovale
D. Persistence of the AV canal
E. *Stenosis of the pulmonic valve
203.
A 2-month-old girl is being examined for a routine checkup. She was
born at term, and there were no problems or complications during the
pregnancy. The baby appeared normal at birth and has been asymptomatic.
Physical examination at this time finds a soft systolic murmur with a systolic
thrill. No cyanosis is present, and her peripheral pulses are thought to be within
normal limits. An ECG reveals slight left ventricular hypertrophy. Which of
the following is the most likely diagnosis?
A. Coarctation of the aorta
B. Patent ductus arteriosus
C. Persistent truncus arteriosus
D. Tetralogy of Fallot
E. *Ventricular septal defect
204.
Which one of the following statements correctly describes the flow of
blood in an individual with an atrial septal defect who develops Eisenmenger’s
syndrome?
A. Aorta to pulmonary artery to lungs to left atrium to left ventricle to aorta
B. Left atrium to right atrium to right ventricle to lungs to left atrium
C. Left ventricle to right ventricle to lungs to left atrium to right ventricle
D. *Right atrium to left atrium to left ventricle to aorta to right atrium
E. Right ventricle to left ventricle to aorta to right atrium to right ventricle
205.
Prior to surgery, which of the following is the best medical therapy for a
newborn infant with transposition of the pulmonary artery and aorta?
A. *Give prostaglandin E2 to keep the ductus arteriosus open
B. Give prostaglandin F2 to close the ductus arteriosus
C. Give oxygen to keep the ductus arteriosus open
D. Give indomethacin to keep the ductus arteriosus open
E. Give indomethacin to close the ductus arteriosus
206.
A 55-year-old man presents with prolonged epigastric pain and severe
vomiting. Laboratory evaluation finds that his blood pH is increased to 7.46,
while his serum bicarbonate is increased to 30 mM. Blood gases also reveal the
arterial carbon dioxide to be increased. Physical examination finds the man to
be afebrile with dry mucous membranes and decreased skin turgor. His heart
rate is increased, but his respiratory rate is decreased in frequency. Which of
the following is the most likely diagnosis?
A. *Metabolic alkalosis with respiratory compensation
B. Mixed metabolic acidosis and metabolic alkalosis
C. Respiratory acidosis with renal compensation
D. Respiratory alkalosis with no compensation
E. Respiratory alkalosis with renal compensation
207.
An anxious 19-year-old woman presents with perioral numbness and
carpopedal spasm. Laboratory examination reveals decreased PCO2 and
decreased bicarbonate. Which of the following is the most likely diagnosis?
A. Metabolic acidosis due to ketoacidosis
B. Metabolic acidosis due to renal tubular acidosis
C. Metabolic alkalosis due to thiazide diuretic
D. Respiratory acidosis due to hypoventilation
E. *Respiratory alkalosis due to hyperventilation
208.
A 35-year-old woman during her first pregnancy develops
oligohydramnios. At 34 weeks of gestation she delivers a stillborn infant with
abnormal facial features consisting of wide-set eyes, low-set floppy ears, and a
broad-flat nose. Which of the following abnormalities is most likely to be
present in this still-born infant?
A. Absence of the thymus
B. *Bilateral renal agenesis
C. Congenital biliary atresia
D. Cystic renal dysplasia
E. Urinary bladder exstrophy
209.
An 8-month-old male infant presents with progressive renal and hepatic
failure. Despite intensive medical therapy, the infant dies. At the time of
autopsy, the external surfaces of his kidneys are found to be smooth, but cut
section reveals numerous cysts that are lined up in a row. Which of the
following is the mode of inheritance of this renal abnormality?
A. Autosomal dominant
B. *Autosomal recessive
C. X-linked dominant
D. X-linked recessive
E. Mitochondrial
210.
Which of the following is the most likely cause of the clinical
combination of generalized edema, hypoalbuminemia, and
hypercholesterolemia in an adult whose urinalysis demonstrated marked
proteinuria, with fatty casts and oval fat bodies?
A. Nephritic syndrome
B. *Nephrotic syndrome
C. Acute renal failure
D. Renal tubular defect
E. Urinary tract infection
211.
A 35-year-old woman recovering from hepatitis B develops hematuria,
proteinuria, and red cell casts in the urine. Which one of the following
statements best describes the expected renal changes in this patient?
A. Plasma cell interstitial nephritis
B. IgG linear fluorescence along the glomerular basement membrane
C. *Granular deposits of antibodies in the glomerular basement membrane
D. Diffuse thickening of the glomerular basement membrane by
subepithelial immune deposits
E. Nodular hyaline glomerulosclerosis
212.
A 2-year-old boy is being evaluated for the development of progressive
peripheral edema. Physical examination finds that he is afebrile, and his blood
pressure is within normal limits. Laboratory examination finds decreased
serum albumin, increased serum cholesterol, and normal BUN and creatinine
levels. Examination of his urine finds massive proteinuria and lipiduria, but no
red blood cells are seen. The loss of albumin in the urine is much greater than
the loss of globulins. A histologic section from a renal biopsy examined with a
routine H
flattening and fusion of the foot processes of the podocytes. The basement
membrane is not fragmented and electron dense deposits are not found. What is
the best diagnosis?
A. Diffuse proliferative glomerulonephritis (DPGN)
B.
C.
D.
E.
Heymann glomerulonephritis (HGN)
Membranoproliferative glomerulonephritis (MPGN)
Membranous glomerulopathy (MGN)
*Minimal change disease (MCD)
213.
A 28-year-old man presents with moderate proteinuria and hypertension.
Histologic sections of a kidney biopsy reveal the combination of
normalappearing glomeruli and occasional glomeruli that have deposits of
hyaline material. No increased cellularity or necrosis is noted in the abnormal
glomeruli. Additionally, there is cystic dilation of the renal tubules, some of
which are filled with proteinaceous material. Electron microscopy reveals focal
fusion of podocytes, and immunofluorescence examination finds granular
IgM/C3 deposits. Further workup finds a mutation involving the NPHS2 gene,
the product of which is found within the slit diaphragm of the glomerulus.
What is the normal protein product of this gene?
A. Cubilin
B. Megalin
C. Nephrin
D. *Podocin
E. Polycystin
214.
A 6-year-old boy presents with bilateral swelling around his eyes. His
parents state that the child’s eyes have become “puffy” over the past several
weeks, and his urine has become smoky-colored. Physical examination reveals
mild bilateral periorbital edema, but peripheral edema is not found. The boy is
afebrile and his blood pressure is slightly elevated. A urinary dipstick reveals
mild proteinuria, while microscopic examination of the boy’s urine reveals
hematuria with red blood cell casts. Laboratory tests reveal increased ASO
titers and decreased serum C3 levels, but C2 and C4 levels are normal. A
microscopic section from the kidney reveals increased numbers of cells within
the glomeruli. An electron microscopic section of the kidney reveals large
electron-dense deposits in the glomeruli that are located between the basement
membrane and the podocytes. The foot processes of the podocytes are
otherwise unremarkable. Which one of the listed infections did this child most
like recently have that precipitated this renal disease?
A. An E. coli infection of the small or large intestines
B. A fungal infection of the urethra or urinary bladder
C. A staphylococcal infection of the skin or mouth
D. *A streptococcal infection of the pharynx or skin
E. A viral infection of the upper or lower respiratory tract
215.
A 47-year-old man presents with increasing peripheral edema and dark,
tea-colored urine. Laboratory examination finds decreased serum albumin,
while examination of a 24-h urine specimen reveals marked proteinuria.
Microscopic examination of this patient’s urine reveals numerous red cells
along with rare red cell casts. Electron microscopic examination of a renal
biopsy from this patient reveals dense, ribbon-like deposits in the lamina densa
of the glomerular basement membrane. Which of the following is the most
likely diagnosis?
A. Acute glomerulonephritis
B.
C.
D.
E.
IgA nephropathy
Lipoid nephrosis
*Membranoproliferative glomerulonephritis
Membranous glomerulopathy
216.
A 21-year-old woman presents because her urine has turned a brown
color. She states that about 2 months ago her urine turned brown 2 days after a
cold and stayed brown for about 3 days. At the current time a urinalysis reveals
complete blood count (CBC), serum electrolytes, BUN, creatinine, glucose,
antinuclear antibodies (ANAs), and serum complement levels (C3 and C4). All
of these tests are within normal limits. Immunofluorescence examination of a
renal biopsy from this patient reveals the presence of large, irregular deposits
of IgA/C3 in the mesangium. A linear staining pattern is not found. Which of
the following is the most likely diagnosis?
A. *Berger’s disease
B. Focal segmental glomerulosclerosis
C. Goodpasture’s syndrome
D. Lipoid nephrosis
E. Membranoproliferative glomerulonephritis
217.
A 43-year-old man with a history of microscopic polyarteritis acutely
develops renal failure with oliguria and hematuria. Laboratory examination
reveals the presence of serum p-ANCA (antineutrophil cytoplasmic
antibodies). A renal biopsy is diagnostic of type III rapidly progressive
glomerulonephritis. Which of the following histologic changes is most likely to
have been present in this biopsy specimen?
A. Eosinophilic masses were seen attached to the capsule of Bowman’s
space
B. Fibrinoid necrosis was present in many of the afferent arterioles
C. Large numbers of neutrophils were seen in the interstitium and tubules
D. *Numerous crescents were present in the glomeruli
E. The basement membrane was seen to be split by mesangial cells
218.
A 28-year-old man with a history of malaise and hemoptysis presents
with the acute onset of renal failure. Laboratory examination reveals increased
serum creatinine and BUN, but no antineutrophil cytoplasmic antibodies
(ANCA) nor antinuclear (ANA) antibodies are present. Urinalysis reveals the
microscopic presence of red blood cells and red blood cell casts, while a renal
biopsy reveals crescents within Bowman’s space of many glomeruli.
Immunofluorescence reveals linear deposits of IgG and C3 along the
glomerular basement membrane. Which of the following is the most likely
diagnosis?
A. Alport’s syndrome
B. Diabetic glomerulopathy
C. *Goodpasture’s syndrome
D. Henoch-Schonlein purpura
E. Wegener’s granulomatosis
219.
A 26-year-old woman presents with increasing fatigue and malaise. She
states that recently she develops a red facial rash whenever she goes outside on
a sunny day. Physical examination finds that she is afebrile, but her blood
pressure is slightly increased and slight peripheral edema is found. Laboratory
evaluation finds slightly elevated BUN and creatinine, while dipstick
examination of her urine reveals slight proteinuria with microscopic hematuria.
Very rare granular and red cell casts are seen. Laboratory examination is also
positive for serum antinuclear antibodies, one of which is anti–double-stranded
DNA. A renal biopsy reveals changes of diffuse proliferative
glomerulonephritis, and the diagnosis of class IV lupus nephritis is made.
Which of the following histologic changes is most characteristic of class IV
lupus nephritis?
A. Mesangial deposits form a “holly leaf” pattern
B. Positive immunofluorescence staining forms a “string of popcorn”
pattern
C. Splitting of the basement membrane forms a “tram-track” pattern
D. Thickening of the basement membrane forms a “spike and dome”
appearance
E. *Thickening of the glomerular capillaries forms a “wire-loop”
appearance
220.
An asymptomatic 24-year-old woman is found to have microscopic
hematuria with a routine urinalysis. Her blood pressure and kidney function are
within normal limits, but it is discovered that several members of her family
also have asymptomatic microscopic hematuria. Which of the following
abnormalities is most likely to be present in this woman?
A. A hereditary defect in the renal transport of neutral amino acids
B. A lack of the globular domain of type IV collagen
C. A mutation involving the cytoplasmic btk gene
D. * Diffuse thinning of the glomerular basement membrane
E. The presence of C3 nephritic factor in the serum
221.
Histologic sections of a kidney reveal patchy necrosis of epithelial cells
of both the proximal and distal tubules with flattening of the epithelial cells,
rupture of the basement membrane (tubulorrhexis), and marked interstitial
edema. Acute inflammatory cells are not seen. Which of the following is the
most likely diagnosis?
A. Acute pyelonephritis
B. *Acute tubular necrosis
C. Chronic glomerulonephritis
D. Chronic pyelonephritis
E. Diffuse cortical necrosis
222.
During a routine physical examination, a 42-year-old woman is found to
have an elevated blood pressure of 150/100 mmHg. Workup reveals a small
left kidney and a normal-sized right kidney. Laboratory examination reveals
elevated serum renin levels. Further workup reveals that renal vein renin levels
are increased on the left but decreased on the right. Which of the following is
the most likely cause of this patient’s hypertension?
A. Atherosclerotic narrowing of the left renal artery
B. Atherosclerotic narrowing of the right renal artery
C. *Fibromuscular hyperplasia of the left renal artery
D. Fibromuscular hyperplasia of the right renal artery
E. Hyaline arteriolosclerosis
223.
A 53-year-old man presents with severe headaches, nausea, and
vomiting. He also relates seeing spots before his eyes and is found to have a
diastolic blood pressure of 160 mmHg. Microscopic examination of a renal
biopsy demonstrates hyperplastic arteriolitis. Gross examination of his kidneys
is most likely to reveal which one of the following changes?
A. A finely granular appearance to the surface
B. * Multiple small petechial hemorrhages on the surface
C. Diffuse, irregular cortical scars overlying dilated calyces
D. Cortical scars overlying dilated calyces in renal poles
E. Depressed cortical areas overlying necrotic papillae of varying stages
224.
A 35-year-old woman presents with the sudden onset of severe, colicky
pain on the right side of her abdomen. She does not relate the pain to food, but
says that she cannot find a pain-free position. Physical examination finds
marked tenderness over the right costovertebral angle, but rebound tenderness
is not present. A pelvic examination is unremarkable. Microscopic
examination of her urine reveals the presence of numerous red blood cells. The
urine is negative for esterase and nitrite, and no bacteria are seen. Which of the
following is the most likely cause of her signs and symptoms?
A. Bilirubin gallstones
B. *Calcium oxalate kidney stones
C. Cholesterol gallstones
D. Magnesium ammonium phosphate kidney stones
E. Acute uric acid nephropathy
225.
An 8-month-old infant boy presents with an enlarging abdominal mass.
Laboratory evaluation finds normal urinary levels of vanillylmandelic acid
(VMA). The mass is removed surgically and microscopic sections reveal
undifferentiated mesenchymal cells, immature tubules, and abortive glomerular
formation. This tumor is most closely associated with abnormalities involving
which one of the listed genes?
A. MET gene
B. PRCC gene
C. p16INK4a gene
D. VHL gene
E. * WT-1 gene
226.
Physical examination of a 3-day-old male infant reveals urine leaking
from the area of the umbilicus. Which of the following is the most likely
diagnosis?
A. Balanoposthitis
B. Meckel’s cyst
C. Meckel’s diverticulum
D. Omphalocele
E. *Urachal fistula
227.
A 19-year-old man presents with dysuria and a mucoid or watery
urethral discharge. No prostatic pain is present. Microscopic examination of
the discharge reveals numerous neutrophils, but no organisms are seen. Which
of the following organisms is the most likely cause of this patient’s signs and
symptoms?
A. * Chlamydia trachomatis
B. Escherchia coli
C. Mycoplasma genitalium
D. Mycoplasma hominis
E. Trichomonas vaginalis
228.
Which of the following histologic changes is most likely to be seen when
examining a mucosal biopsy of the urinary bladder from an individual with
acute cystitis due to infection with Escherichia coli?
A. An infiltrate of lymphocytes and plasma cells
B. *An infiltrate of neutrophils
C. Inflammation with eosinophils
D. Noncaseating granulomas
E. Sheets of macrophages with granular cytoplasm
229.
A 49-year-old man who is a long-term smoker presents with frequency
and hematuria. Histologic examination of sections taken from an exophytic
lesion of the urinary bladder reveals groups of atypical cells with frequent
mitoses forming finger-like projections that have thin, fibrovascular cores.
These groups of atypical cells do not extend into the lamina propria and
muscularis. No glands or keratin production are found. Which of the following
is the most likely diagnosis?
A. Adenocarcinoma, noninvasive
B. Inverted papilloma, noninvasive
C. Transitional cell carcinoma in situ
D. * Papillary transitional cell carcinoma, noninvasive
E. Squamous cell carcinoma in situ
TESTS FOR FIGURES
1. Name the diagnosis of disease of the heart layers, which is represented in Figure
2.57.
A. Hemorrhagic pericarditis
B. purulent pericarditis
C. serous pericarditis
D. fibrinous pancarditis
E. fibrinous pericarditis*
2. At the heart of dermatomyositis observed dystrophy, cardiomyocytes mediate
myocarditis, and the pathological process is completed (Figure 2.34):
A. The partial regeneration of cardiomyocytes
B. Complete regeneration of cardiomyocytes
C. Pathological cardiomyocyte regeneration
3.
4.
5.
6.
7.
D. Focal elastofibrosis
E. Diffuse cardiosclerosis atrophy of cardiomyocytes*
A characteristic feature of lupus glomerulonephritis (Fig. 2.31-2) is the deposition:
A. Non-immune complexes and thickening of the capillaries in the form of " wire
sticks"
B. Immune complexes and thickening of the capillaries in the form of " wire
Rings
C. Non-immune complexes and thickening of the capillaries in the form of " wire
Rings
D. Non-immune complexes and utonsheniem capillaries in the form of " wire
loops"
E. Immune complexes and thickening of capillaries in the form of " wire loops"*
What are the characteristic feature of lupus glomerulonephritis (Fig. 2.31-2):
A. Non-immune complexes and thickening of the capillaries in the form of " wire
sticks"
B. Immune complexes and thickening of the capillaries in the form of " wire
Rings
C. Non-immune complexes and thickening of the capillaries in the form of " wire
Rings
D. Non-immune complexes and utonsheniem capillaries in the form of " wire
loops"
E. Immune complexes and thickening of capillaries in the form of " wire loops"*
What is a characteristic feature of lupus glomerulonephritis (Fig. 2.31-2):
A. Non-immune complexes and thickening of the capillaries in the form of " wire
sticks"
B. Immune complexes and thickening of the capillaries in the form of " wire
Rings
C. Non-immune complexes and thickening of the capillaries in the form of " wire
Rings
D. Non-immune complexes and utonsheniem capillaries in the form of " wire
loops"
E. Immune complexes and thickening of capillaries in the form of " wire loops"*
The consequence of lupus glomerulonephritis (Fig. 2.31-2) is the development of:
A. Wrinkled kidneys
B. Cystic expansion of renal
C. Hydronephrosis Kidney
D. Motley kidney
E. Utonshenie kidney*
What are the characteristic feature of lupus glomerulonephritis (Fig. 2.31-2):
A. Non-immune complexes and thickening of the capillaries in the form of " wire
sticks"
B. Immune complexes and thickening of the capillaries in the form of " wire
Rings
C. Non-immune complexes and thickening of the capillaries in the form of " wire
Rings
D. Non-immune complexes and utonsheniem capillaries in the form of " wire
loops"
E. Immune complexes and thickening of capillaries in the form of " wire loops"*
8. In the heart (Fig. 2.32) patients on systemic lupus erythematosus develops:
A. Sepsis
B. Bacterial valvulitis
C. Bacterial endocarditis
D. Nebakterial endocarditis
E. Nebakterial Libman-Sachs endocarditis*
9. What is characterized by a diffuse pleural interstitial myocarditis (Fig. 2.35):
A. No significant infiltration of lymphocytes, histiocytes, neutrophils and
eosinophils in the presence of single granulomas Ashof-body
B. Significant infiltration of lymphocytes in the presence of single granulomas
Ashof-body
C. Significant infiltration of lymphocytes
D. No significant interstitial infiltration of neutrophils in the presence of single
granulomas Ashof-body
E. Significant infiltration of lymphocytes, histiocytes, neutrophils and eosinophils
in the presence of solitary Ashof-body*
10.Focal exudative interstitial myocarditis manifested (Fig. 2.35):
A. To a large focal interstitial infiltration by lymphocytes
B. To a large focal interstitial infiltration eosinophils
C. To a large focal interstitial infiltration of plasma cells
D. Ashof- body
E. The small focal interstitial infiltration of lymphocytes, histiocytes and
neutrophils*
11.What is manifested focal interstitial myocarditis, exudative (Fig. 2.35):
A. To a large focal interstitial infiltration by lymphocytes
B. To a large focal interstitial infiltration eosinophils
C. To a large focal interstitial infiltration of plasma cells
D. Ashof- body
E. The small focal interstitial infiltration of lymphocytes, histiocytes and
neutrophils*
12.The development of sclerosis myocarditis completed (Fig. 2.34) at:
A. Not a favorable flow of rheumatism
B. The occurrence of rheumatic disease with frequent relapses
C. The occurrence of rheumatic fever with high activity indices of blood
D. Predominance of endocarditis
E. Favorable course of rheumatism*
13.Cardiosclerosis a result of myocarditis (Fig. 2.34) at:
A. Predominance of endocarditis
B. Not the favorable development of rheumatic
C. The occurrence of rheumatic disease with frequent relapses
D. The occurrence of rheumatic fever with high activity indices of blood
E. Favorable development of rheumatic*
14.Pericarditis rheumatism has character(Fig. 2.57):
A. Mixed inflammation
B. Purulent exudative inflammation
C. Hemorrhagic exudative inflammation
D. Catarrhal exudative inflammation
E. Serous exudative inflammation*
15.What are the nature of inflammation in the pericardium(Fig. 2.57) :
A. Mixed inflammation
B. Purulent exudative inflammation
C. Hemorrhagic exudative inflammation
D. Catarrhal exudative inflammation
E. Serous exudative inflammation*
16.What is the nature of inflammation in the pericardium (Fig. 2.57):
A. Mixed inflammation
B. Purulent exudative inflammation
C. Hemorrhagic exudative inflammation
D. Catarrhal exudative inflammation
E. Serous exudative inflammation*
17.Pericarditis rheumatism has character(Fig. 2.57):
A. Mixed inflammation
B. Purulent exudative inflammation
C. Hemorrhagic exudative inflammation
D. Catarrhal exudative inflammation
E. Serous exudative inflammation*
18.Frequently rheumatic pericarditis completed education(Fig. 2.57):
A. Suppuration pericardial cavity
B. Papillary proliferation of the pericardium
C. Inflammation
D. Рapillary transformation of the epicardium
E. Adhesions pericardial cavity*
19.The result of pericarditis(Fi.2.57) is education:
A. Suppuration pericardial cavity
B. Papillary proliferation of the pericardium
C. Inflammation
D. Рapillary transformation of the epicardium
E. Adhesions pericardial cavity*
20.Most rheumatic pericarditis(Fig. 2.57) concludes obliteration of the cavity of the
heart bags with calcification, formation of connective tissue in this case leads to
the development:
A. Cor Pulmonale
B. Tiger Heart
C. Hairy heart
D. Bovine heart
E. Stone heart*
21.Histological examination of kidney tissue revealed focal and diffuse thickening of
the basal membrane with the formation of " wire loops " Fig. 2.31. Occurs
appearance gematoksiliny body. What clinical and anatomical form of the disease.
A. Cardiac
B. Arthritic
C. Mixed
D. Cerebral
E. Visceral*
22.Histological examination of kidney tissue revealed focal and diffuse thickening of
the basal membrane with the formation of " wire loops " Fig. 2.31. Occurs
appearance gematoksiliny body. What clinical and anatomical form of the disease.
A. Cardiac
B. Arthritic
C. Mixed
D. Cerebral
E. Visceral*
23.What clinical and anatomical form of the disease on Fig. 2.31. Histologically in
the study of kidney tissue revealed focal and diffuse thickening of the basal
membrane with the formation of " wire loops”. Appear gematoksiliny calf
A. Cardiac
B. Arthritic
C. Mixed
D. Cerebral
E. Visceral*
24.What pathological processes caused the name of spleen ("porphyry") in
lymphogranulomatosis (Fig. 2.30)?
A. Amyloidosis
B. Hyalinosis
C. Necrosis and amyloidosis
D. Sclerosis and hyalinosis
E. Necrosis and sclerosis*
25.The specific pathognomonic sign of this type of anaemia is atrophy of fundal
mucus cells in stomach. Name disease. (Fig. 2.38)
A. Acute posthemorrhagic
B. Chronic posthemorrhagic
C. Hemolytic
D. Aplastic
E. Pernicious (vit. B 12-deficiency)*
26.The sign of what pathological process is decrease of glands in the mucus layer of
stomach (a number 1 on Fig. 2.38) at pernicious (vit. B 12-deficiency) anemia?
A. Hyperplasia
B. Hypertrophy
C. Aplasia
D. Sclerosis
E. Atrophy*
27.At histological research of stomach (Fig. 2.38) patient with anaemia founded
atrophy of glands and sclerosis (1), focal lymphocytes infiltration (2). Name the
type of anaemia.
A. Acute posthemorrhagic
B. Chronic posthemorrhagic
C. Hemolytic
D. Aplastic
E. Pernicious (vit. B 12-deficiency)*
28.At histological examination of lymph node revealed of Hodgkin's giant single nucleus cells (Fig. 2.29). What pathological process shown in the lymph node
slide?
A. Acute leukemia
B. Ekstramedullar hemopoiesis
C. Chronic erythromyelosis
D. Chronic lympholeukosis
E. Lymphogranulomatosis*
29.At histological examination of lymph node diagnosed lymphogranulomatosis (Fig.
2.29). Determine group of hemopoietic and lymphatic tissue diseases?
A. Pretumor disease
B. Immune inflammation
C. Systemic disease hemopoietic tissue
D. Malignant tumor of mesenhyma
E. Regional tumor - lymphomas*
30.At histological examination of lymph node in patient with lymphogranulomatosis
detected giant single - nucleus cells (Fig. 2.29). Name the cells marked by arrow
on the lymph node slide.
A. Pirogov-Langhans' giant cells
B. Mykulich' cells
C. Eosinophil
D. Berezovsky-Sternberg' cells
E. Hodgkin' cells*
31.In patients diagnosed lymphogranulomatosis. What specific cells founded in the
lymph nodes (Fig. 2.29)?
A. Pirogov-Langhans' giant cells
B. Mykulich' cells
C. Anichkov' cells
D. Virchow' cells
E. Hodgkin' cells*
32.Autopsy of a woman, who died from lymphogranulomatosis revealed enlarged
spleen. On the section - multiple white-yellow areas of necrosis and sclerosis (Fig.
2.30). What is named of such spleen?.
A. Sebaceous
B. Sago-like spleen
C. Glazed
D. Hyperplastic
E. Porphyry*
33.Autopsy of a woman, who died from lymphogranulomatosis revealed enlarged
spleen. On the section - multiple white-yellow areas of necrosis and sclerosis
(porphyry) (Fig. 2.30). Name the diseases which characterizing such spleen.
A. Acute leukemia
B. Ekstramedullar hemopoiesis
C. Chronic miyeloleykosis
D. Chronic lymphocytic leukemia
E. Lymphogranulomatosis*
34.At histological examination of the stomach (Fig. 2.38) patients with anemia
founded glands atrophy and sclerosis (1), focal lymphocytic infiltration (2). Name
the cause of this phenomen.
A. Iron deficiency
B. Excess vit. B12
C. Deficiency of folic acid
D. Bone marrow aplasia
E. Deficiency of vit. B12*
35.Hodgkin's disease (lymphoma) (Fig. 2.29) – is
A. acute lymphoma
B. Acute limphotcytoma
C. Acute limphocytoblastoma
D. chronic lymphocytic leukemia
E. Chronic recurrent lymphoma*
36.Hodgkin's disease (lymphoma) (Fig. 2.29) is:
A. episodic
B. endemic
C. pandemic
D. sporadic
E. isolated (local) and generalized*
37.In Hodgkin's disease often affects the spleen (Fig. 2.30) (necrotic foci whiteyellow color, multiple sclerosis, lymphocytic infiltration), so is called:
A. sugar-coated spleen
B. hyaline spleen
C. fibrous spleen
D. sago-like spleen
E. porphyric spleen*
38.In the lymph nodes in lymphogranulomatosis revealed proliferation of
lymphocytes, histiocytes, atypical small and large mononuclear cells (Fig. 2.29):
A. Ponce
B. Gaucher
C. Pirogov
D. Berezovsky
E. Hodgkin's*
39.In the patient histologically diagnosed limphogranulomatous (Hodgkin's disease)
(Fig. 29). Distinguish the following clinical and morphological form of the disease
A. variant with monocytosis
B. variant polycytosis
C. variant geterocytosis
D. version of the anemia
E. variant with a predominance of lymphoid tissue*
40.In the patient histologically diagnosed limphogranulomatous (Hodgkin's disease)
(Fig. 2.29). Distinguish the following clinical and morphological form of the
disease:
A. diffuse fibroblastocytosis
B. knotty fibrosis
C. multinodular cirrhosis
D. focal fibrosis
E. nodular sclerosis*
41.In the patient was histologically diagnosed limphogranulomatous (Hodgkin's
disease) (Fig. 2.29). Distinguish the following clinical and morphological form of
the disease:
A. multinodular cirrhosis
B. knotty fibrosis
C. diffuse fibroblastocytosis
D. single cell variant
E. mixed-cell variant*
42.Hodgkin's giant cells (Fig. 2.29) contain:
A. giant vacuoles
B. giant fat inclusion
C. giant phagosomes
D. giant lysosomes
E. large hyperchromic nucleus*
43.Pathomorphological manifestations of pernicious anemia in the gastric mucosa
(Fig. 2.38) are:
A. polyposis
B. ulceration
C. exacerbation of the inflammatory process
D. development of chronic colitis
E. atrophic and sclerotic changes*
44.The patient revealed atrophy and sclerosis of the gastric mucosa (Fig. 2.38) and
hyperchromic anemia. Among the following hyperchromic anemia is
A. Iron deficiency:
B. Metaplastic
C. Hemolytic
D. Posthemorrhagic
E. B12-deficient*
45.During the endoscopy the patient revealed atrophy and sclerosis of the gastric
mucosa (Fig. 2.38), the laboratory - megaloblastic hyperplasia of the bone
marrow, which is caused by hypovitaminosis:
A. A
B. B1
C. C
D. PP
E. B12*
46.The patient endoscopy revealed atrophy and sclerosis of the gastric mucosa (Fig.
2.38) and hyperchromic anemia. More typical blood cells and bone marrow with
B12-deficiency anemia are:
A. Anisocytes
B. Microcytes
C. Normoblasts
D. Hypochromic red blood cells
E. Megaloblasts*
47.What stage of Caries do you see on Fig. 2.56?
A. superfacial
B. middle
C. Fig. shows traumatic tooth defect
D. Fig. shows the erosion of tooth
E. deep*
48.What is the pathology that is depicted on Fig. 2.56?
A. tooth erosion
B. fluorosis
C. secondary caries
D. pulpitisis
E. deep caries*
49.What does the marker 1 (Fig. 2.56) show us?
A. damaged tooth enamel
B. caverns in carious dentin
C. dental pulp
D. destroyed pulp in the tooth cavity
E. carious caverns in dentin and pulp*
50.Which stage of caries do you see on Fig. 2.15?
A. stage spots (enamel structure is not changed)
B. surface (enamel damaged)
C. average (beginning of the dentin destruction)
D. there is not tooth tissue
E. medium or deep (dentine is damaged)*
51.What is marked by number 3 on Fig. 2.15?
A. transparent dentin
B. softening and destroying of dentin
C. typical dentine
D. reparative dentine
E. extended irregular dentine tubules*
52.What is marked by number 1 on Fig. 2.15?
A. transparent dentin
B. extended irregular dentine tubules
C. typical dentine
D. reparative dentine
E. softening and destroying of dentin*
53.What is marked by number 2 on Fig. 2.15?
A. Extended irregular dentine tubules
B. softening and destroying of dentin
C. typical dentine
D. reparative dentine
E. transparent dentin*
54.What is marked by number 1 on Fig. 2.5 (preparation of tooth with deep caries
and complication by pulpitis)?
A. lymphocytes and histiocytes
B. abscess formation in pulp
C. sclerosis and fibrosis of pulp
D. hydropic dystrophy of pulp
E. hyperaemia and stasis in the capillaries of pulp*
55.What type of dystrophy marked by number 2 on Fig. 2.5 (preparation of tooth
with deep caries and complication by pulpitis)?
A. hyaline dystrophy of pulp
B. leukoplakia of pulp
C. hyalinosis of pulp
D. sclerosis and fibrosis of pulp
E. hydropic (vakuol) pulp dystrophy*
56.In which part of the tooth is placed number 3 on Fig. 2.5 (preparation of tooth
with deep caries and complication by pulpitis)?
A. superficial of the tooth
B. enamel-dentine border
C. deep divisions of a tooth cavity
D. tooth enamel
E. marginal portion of the tooth dentine*
57.What is marked by number 4 on Fig. 2.5 (preparation of tooth with deep caries
and complication by pulpitis)?
A. hyperaemia and stasis in the capillaries of pulp
B. lymphocytes and histiocytes
C. abscess formation in pulp
D. hydropic dystrophy of pulp
E. sclerosis and fibrosis of pulp*
58.What is marked by number 5 on Fig. 2.5 (preparation of tooth with deep caries
and complication by pulpitis)?
A. periodontal of tooth
B. transparent dentine of the tooth
C. tooth enamel
D. tooth cement
E. dentine with elements of destruction*
59.What is marked by number 6 on Fig. 2.5 (preparation of tooth with deep caries
and complication by pulpitis)?
A. central department of tooth crown
B. transparent dentine of the tooth
C. typical dentine
D. tooth cement
E. tooth enamel*
60.About what do the changes show us (indicated by number 1 on Fig. 2.8
(preparation of tooth with deep caries, chronic pulpitis))?
A. formation of reparative dentin
B. hydropic dystrophy of pulp
C. formation of adipose tissue
D. forming callus
E. sclerotic and atrophic changes in pulp*
61.About what do the changes show us (indicated by number 3 on Fig. 2.8
(preparation of tooth with deep caries, chronic pulpitis))?
A. sclerotic and atrophic changes in pulp
B. hydropic dystrophy of pulp
C. adipose tissue formation
D. callus forming
E. denticles and calcificates inside of the altered pulp*
62.What is markered by number 1 on Fig. 2.13 (the tooth at acute purulent pulpitis)?
A. formed dentycles
B. sclerotic and atrophic changes in pulp
C. necrosis of pulp
D. forming of callus
E. irregular dentine with extended tubules*
63.What is markered by number 2 on Fig. 2.13 (the tooth at acute purulent pulpitis)?
A. macrophages around the abscess
B. pulp necrosis
C. substitution dentine in the tooth cavity
D. forming callus
E. connective tissue of pulp with fibroblasts*
64.What part of tooth is markered by number 3 on Fig. 2.13 (the tooth at acute
purulent pulpitis)?
A. external surface of the
B. tooth enamel
C. dentine of tooth
D. cementum of tooth
E. tooth cavity*
65.What is markered by number 4 on Fig. 2.13 (the tooth at acute purulent pulpitis)?
A. grown up of pulp connective tissue with fibroblasts
B. macrophages around the abscess
C. granulemma formation
D. forming of callus
E. purulent melting pulp*
66.What pathology is the on Fig. 2.14?
A. granulema
B. fibromatosis
C. papiloma
D. sharp end condiloma
E. dens-alveolar portion of jaw*
67.What is markered by number 1 on Fig. 2.14 (preparation of alveolar processus of
jaw)?
A. periodontal damage
B. caries of dens neck
C. circular caries
D. bone resorption
E. mucosal defect and formation of periodontal gum niche*
68.What is markered by number 2 on Fig. 2.14 (preparation of alveolar processus of
jaw)?
A. mucosal defect and formation of periodontal gum niche
B. periodontal damage
C. circular caries
D. bone resorption
E. dens stone formation*
69.At endoscopical research a doctor found the changes of mucus membrane of
stomach (Fig. 2.28.B). What previous diagnosis did a doctor put?
A. Acute gastritis
B. Chronic gastritis with acuteening
C. Chronic hyperplastycal gastritis (Menetrie)
D. There are no visible changes mucus
E. Chronic atrophy gastritis*
70.At pathoanatomical research a doctor noticed a pied picture of stomach mucus
membrane (Fig. 2.28.C). What pathological process did a doctor see?
A. Pigmental spots of stomach mucus.
B. Catarrhal inflammation of stomach mucus.
C. Perforation of stomach mucus.
D. Abscesses of stomach.
E. Haemorrhagic erosion.*
71.Pathoanatomical macroscopic research found pathology of stomach (Fig. 2.28.C).
This pathology was confirmed by a microscopic method (Fig. 2.28.D). What
pathology do you see?
A. Pigmental spots of stomach mucus.
B. Catarrhal inflammation of stomach mucus.
C. Perforation of stomach mucus.
D. Abscesses of stomach.
E. Haemorrhagic erosion of stomach mucus.*
72.At posthumous research of 40-years-old man a pathologist found pathology of
mucus membrane of stomach (Fig. 2.37.A). What did a doctor see?
A. Acute fibrinous gastritis
B. Chronic atrophic gastritis
C. Chronic polypus gastritis
D. Chronic gastric ulcer
E. Acute gastric ulcer*
73.Pathoanatomical research of stomach found the pathology of mucus membrane in
dying man (Fig. 2.37.B). Your diagnosis is..
A. Acute fibrinous gastritis
B. Chronic atrophic gastritis
C. Chronic polypus gastritis
D. Acute gastric ulcer
E. Chronic gastric ulcer*
74.At macroscopic research a pathologist found the pathology of stomach mucus
(Fig. 2.37.B). What pathology do you see?
A. Fibrinous inflammation of mucus membrane
B. The acid burn of mucus membrane on a small curvature
C. Hyperplastycal gastritis (Menetrie)
D. Metaplasy of mucus membrane
E. Atrophy and deformation of mucus membrane*
75.Pathoanatomical research of gastroenteric tract found the pathology of duodenum..
(Fig. 2.36.A)
A. Acute gastric ulcer
B. Chronic gastric ulcer
C. Acute duodenitis
D. Acute ulcer of duodenum
E. chronic ulcer of duodenum*
76.In the pathoanatomical department was delivered a man which died in the
cardiologic department. In the history of diseases was written that the man
complained on pain in a stomach a long time. At research of internalss a
pathologist discovered (Fig. 36.B) ..
A. Acute bleeding from a gastric ulcer
B. Erosion of mucus stomach
C. traumatic destroing of duodenum
D. Perforated ulcer of duodenum
E. erosive vessels on the bottom of duodenum chronic ulcer*
77.The pathoanatomical macroscopic research of gastroenteric tract discovered (Fig.
2.36)
A. Acute ulcer of duodenum
B. Signs of repair of acute ulcer of duodenum
C. Gastric ulcer with complication by bleeding
D. Chronic gastric ulcer with arrosive vessels on a bottom
E. chronic ulcer of duodenum with arrosive vessels on a bottom*
78.Macroscopic research of gastroenteric tract found pathology of mucus stomach
(Fig. 2.26.A). It is ..
A. Acute gastritis
B. Chronic gastritis in the acute condition
C. Gastric ulcer
D. Haemorragic gastritis
E. Scar of mucus*
79.Macroscopic research of gastroenteric tract of man, which died from acute
peritonitis, discovered .. (Fig. 2.26.B)
A. inflammation of stomach
B. Pancreanecrosis
C. Fatty hepatosis of liver («ancerine» liver)
D. Muscat liver
E. Perforated ulcer of duodenum*
80.Pathoanatomical research of stomach mucus discovered cicatrical tissue in the
pylorus area. It became reason of .. (Fig. 2.26.B)
A. Gastric ulcers
B. Ulcers of a duodenum
C. Perforations of stomach
D. Perforations of a duodenum
E. Dilatations of stomach*
81.After surgical operation in the abdominal cavity the ectomied organ was presented
to the pathologist for research (Fig. 2.27.A.). Your diagnosis ..
A. Acute phlegmonous appendicitis
B. Acute fibrinous-festering appendicitis
C. Acute destructively-necrotizing appendicitis
D. Chronic appendicitis
E. Acute catarrhal appendicitis*
82.After appendectomy the appendix was given to the pathologist for research (Fig.
2.27.B.). Your diagnosis ..
A. Abscess of appendix
B. Fibrinous appendicitis
C. Phlegmonous appendicitis
D. Chronic appendicitis
E. Mucocele (mucocyst)*
83.The removed at the operation appendix became the object of pathoanatomical
research. Name the diagnose using the original view of organ (Fig. 6.A.).
A. Acute fibrinous-festering appendicitis
B. Acute phlegmonous appendicitis
C. Acute destructively-necrotizing appendicitis
D. Chronic appendicitis
E. Acute catarrhal fibrinous appendicitis*
84.Specify (Fig. 2.16) fibrinous exudate (lobar pneumonia):
A. 2
B. 3
C. 4
D. 5
E. 1*
85.Specify leukocyte infiltration in lobar pneumonia (Fig. 2.16):
A. 1
B. 3
C. 4
D. 5
E. 2*
86.Specify (Fig. 2.16) the thickened walls of the alveoli (lobar pneumonia):
A. 1
B. 2
C. 4
D. 5
E. 3*
87.Specify leukocyte infiltration in lobar pneumonia (Fig. 2.16):
A. 1
B. 3
C. 4
D. 5
E. 2*
88.Name the pathological process (Fig. 2.16):
A. bronchopneumonia
B. bronchitis
C. pneumonia
D. larengit
E. lobar pneumonia*
89.Select the pus in the abscess cavity (Fig. 2.19):
A. 2
B. 3
C. 4
D. 5
E. 1*
90.Select the wall abscess (Fig. 19):
A. 1
B. 3
C. 4
D. 5
E. 2*
91.Select the dilated vessels in the lung abscess (Fig. 2.19):
A. 1
B. 2
C. 4
D. 5
E. 3*
92.Select the neutrophilic infiltration in the lung abscess (Fig. 2.19):
A. 1
B. 2
C. 3
D. 5
E. 4*
93.What are the pathological process (Fig. 2.19):
A. lobar pneumonia
B. bronchitis
C. cancer
D. larengit
E. abscess*
94.Specify (Fig. 2.20) the leucocyte infiltration (bronchopneumonia):
A. 2
B. 3
C. 4
D. 5
E. 1*
95.Specify (Fig. 2.20) the exudate in the bronchial lumen (bronchopneumonia):
A. 1
B. 3
C. 4
D. 5
E. 2*
96.Specify (Fig. 2.20) the advanced vessels (bronchopneumonia):
A. 1
B. 2
C. 4
D. 5
E. 3*
97.Specify (Fig. 2.20) the thickened walls of the alveoli (bronchopneumonia):
A. 1
B. 2
C. 3
D. 5
E. 4*
98.What is the pathological process in this Figure (Fig. 2.20):
A. lobar pneumonia
B. bronchitis
C. pneumonia
D. larengit
E. bronchopneumonia*
99.What are the pathological process (Fig. 2.21):
A. bronchopneumonia
B. bronchitis
C. pneumonia
D. larengit
E. carnification*
100. Specify (Fig. 2.22) the leukocyte infiltration (intermediate pneumonia):
A. 2
B. 3
C. 4
D. 5
E. 1*
101. Specify (Fig. 2.22) the extended alveolar walls (intermediate pneumonia):
A. 1
B. 3
C. 4
D. 5
E. 2*
102. Specify (Fig. 2.22) the advanced vessels (intermediate pneumonia):
A. 1
B. 2
C. 4
D. 5
E. 3*
103. What are the pathological process (Fig. 2.22):
A. pneumonia in influenza
B. viral bronchitis
C. pneumonia
D. Viral larengit
E. intermediate pneumonia*
104. Specify (Fig. 2.23) the leukocyte infiltration (chronic bronchitis):
A. 2
B. 3
C. 4
D. 5
E. 1*
105. Specify (Fig. 2.23) the advanced vessels (chronic bronchitis):
A. 1
B. 2
C. 4
D. 5
E. 3*
106. Specify (Fig. 2.23) the exudate in the lumen of the bronchus (chronic
bronchitis):
A. 1
B. 3
C. 4
D. 5
E. 2*
107. Name (Fig. 2.23) the pathological process:
A. bronchopneumonia
B. intermediate pneumonia
C. pneumonia
D. larengit
E. bronchitis*
108. Specify (Fig. 2.24) advanced lumen alveoli (emphysema):
A. 2
B. 3
C. 4
D. 2 i 4
E. 1*
109. Specify (Fig. 2.24) thinning alveolar septum:
A. 1
B. 3
C. 1 i 4
D. 1 i 2
E. 2 i 4*
110. Name the pathological process (Fig. 2.24):
A. bronchopneumonia
B. intermediate pneumonia
C. pneumonia
D. larengit
E. emphysema*
111. Specify hyperchromic nuclei (Fig. 17):
A. 2 and 3
B. 3 and 1
C. 3
D. 2
E. 1*
112. Specify the cancer cells (Fig. 2.17):
A. 1
B. 2 and 3
C. 3
D. 3 and 1
E. 1 and 2*
113. What type of cell atypizm can be seen in the picture (Fig. 2.17):
A. biochemical
B. fabric
C. intercellular
D. interstices
E. cell and tissue*
114. Name the pathological process (Fig. 2.17):
A. bronchopneumonia
B. intermediate pneumonia
C. pneumonia
D. larengit
E. cancer*
115. What type of cell atypizm is numbered 1 (Fig. 2.25):
A. hipohromii nuclei
B. different size fibers
C. atypical location of glands
D. irregular shape of glands
E. hyperchromic nuclei*
116. What type of cell atypizm can be seen (Fig. 2.17):
A. cancer than in normal
B. different size fibers
C. Atypical location of glands
D. irregular shape of glands
E. polymorphism nuclei*
117. What type of cell atypizm can be seen (Fig. 2.17):
A. cancer than in normal
B. different size fibers
C. Atypical location of glands
D. irregular shape of glands
E. different size nuclei*
118. Specify (Fig. 2.25) the cancer cells:
A. 1
B. 2 and 3
C. 3
D. 3 and 1
E. 2*
119. What types of atypizm can be seen (Fig. 2.25):
A. biochemical
B. fabric
C. intercellular
D. interstices
E. cell and tissue*
120. Name the pathological process (Fig. 2.25):
A. bronchopneumonia
B. intermediate pneumonia
C. pneumonia
D. larengit
E. lung*
121. What type of cell atypizm can be seen (Fig. 2.25):
A. cancer than in normal
B. different size fibers
C. Atypical location of glands
D. irregular shape of glands
E. polymorphism nuclei*
122. What type of cell atypizm can be seen (Fig. 2.25):
A. cancer than in normal
B. different size fibers
C. Atypical location of glands
D. irregular shape of glands
E. different size nuclei*
123. Name the pathological process (Fig. 2.20):
A. lobar pneumonia
B. tracheitis
C. larengit
D. lung
E. bronchopneumonia*
124. Name the pathological process (Fig. 2.24):
A. lobar pneumonia
B. tracheitis
C. larengit
D. lung
E. emphysema*
125. Specify (Fig. 2.25) the diagnosis:
A. bronchitis
B. tracheitis
C. laryngitis
D. pericarditis
E. cancer of the lung*
126. Histological examination of lung tissue (Fig. 2.6) revealed dense protein
masses, which are colored intensely with eosin, and are on the respiratory regions
of the lungs. Hyaline membranes composed of fibrin. They block the exchange of
gases in the lungs. What kind of pathology:
A. Pathology of the placenta
B. Fetopathy
C. Blastopathy
D. Antenatal Pathology
E. Perinathal pathology*
127. Histological examination of lung tissue (Fig. 2.6) revealed dense protein
masses, which are colored intensely with eosin, and are on the respiratory regions
of the lungs. Hyaline membranes composed of fibrin. They block the exchange of
gases in the lungs. What kind of pathology:
A. Pathology of the placenta
B. Fetopathy
C. Blastopathy
D. Antenatal Pathology
E. Pneumopathy*
128. What is the pathology of lung tissue is on Fig. 2.6. Histology revealed dense
protein masses, which are colored intensely with eosin, and firmly stick to the
walls of respiratory regions of the lungs. Hyaline membranes are composed of
fibrin and blocks gas exchange in the lungs.
A. Pathology of the placenta
B. Fetopathy
C. Blastopathy
D. Antenatal Pathology
E. Pneumopathy*
129. What is the pathology of lung tissue in Fig. 2.6. Histology revealed dense
protein masses, which are colored intensely with eosin, and firmly stick to the
walls of respiratory regions of the lungs. Hyaline membranes are composed of
fibrin and blocks gas exchange in the lungs.
A. Pathology of the placenta
B. Fetopathy
C. Blastopathy
D. Perinatal pathology
E. Congenital malformations of the respiratory system*
130. Grossly the brain tissue preterm neonates revealed a hematoma in the cavity of
the ventricles with the development of tamponade (Fig. 2.7). What are the causes
of birth injury, embodied in the state of the fetus:
A. Embriopathy - malformations, which are accompanied by venous congestion in
the tissues of the fetus
B. Fetopathy, which are accompanied by haemorrhagic syndrome
C. Rigidity tissue birth canal
D. All of the options
E. Premature birth or fetal postmaturity*
131. Grossly the brain tissue preterm neonates revealed a hematoma in the cavity of
the ventricles with the development of tamponade (Fig. 2.7). What are the causes
of birth trauma rooted in the mother's birth canal:
A. Distortion of the pelvis
B. Tumors of the birth canal
C. oligohydramnios and premature rupture of membranes
D. Rigidity tissues birth canal
E. All of these options correct*
132. What are the causes of birth injury, embodied in the mother's birth canal, which
lead to the development of pathologic changes, shown in Fig. 2.7. Grossly the
brain tissue in a premature newborn revealed a hematoma in the cavity of the
ventricles with the development of tamponade.
A. Distortion of the pelvis
B. Tumors of the birth canal
C. Oligohydramnios and premature rupture of membranes
D. Rigidity tissues birth canal
E. All of these options correct*
133. Grossly the brain tissue preterm neonates revealed a hematoma in the cavity of
the ventricles with the development of tamponade (Fig. 2.7). What are the possible
causes of birth trauma rooted in the dynamics of birth:
A. fetal hypoxia
B. oligohydramnios and premature rupture of membranes
C. Rigidity tissues birth canal
D. All of these options correct
E. accelerated labor*
134. Grossly tissue of the abdominal cavity in a child who died at 2 days was found
additional spleen( Fig. 2.58). What it is:
A. Aplasia
B. Hypoxia
C. Ectopia
D. All of the options
E. Congenital hypertrophy*
135. Grossly tissue of the abdominal cavity in a child who died at 2 days was found
additional spleen (Fig. 2.58). Specify in what time period are formed such changes
A. Period blastogenesis
B. Period fetogeneza
C. Early fetal period
D. postural fetal period
E. Period embryogenesis*
136. Grossly tissue of the abdominal cavity of the child, who died at 2 days, found
additional spleen (Fig. 2.58). Indicate to what pathology include these changes.
A. Blastopathy
B. Fetopathy
C. Early fetal period
D. Postural fetal period
E. Embriopathy*
137. At Fig. 2.9 shows kefalogematoma newborn baby. What the reason for this
pathological process:
A. Polyhydramnios
B. Blastopathy
C. Embriopathy
D. Fetopathy
E. Overlay forceps*
138. At Fig. 2.9 shows kefalogematoma newborn baby. What the reason for this
pathological process:
A. Polyhydramnios
B. Blastopathy
C. Embriopathy
D. Fetopathy
E. Accelerated labor*
139. At Fig. 2.9 shows kefalogematoma newborn baby. What the reason for this
pathological process:
A. Polyhydramnios
B. Blastopathy
C. Embriopathy
D. Fetopathy
E. Prolonged labor*
140. At Fig. 2.9 shows kefalogematoma newborn baby. What the reason for this
pathological process:
A. Polyhydramnios
B. Blastopathy
C. Embriopathy
D. Fetopathy
E. State of prematurity or newborn postmaturity*
141. At Fig. 2.9 shows kefalogematoma newborn baby. What the reason for this
pathological process:
A. Polyhydramnios
B. Blastopathy
C. Embriopathy
D. Fetopathy
E. Oligohydramnios*
142. Women 32 years removed uterine tube. Histologically, it revealed chorionic
villi, which are depicted on Fig. 2.11. Name the cause of intervention:
A. Abdominal Pregnancy
B. Ovarian Pregnancy
C. Uterine pregnancy
D. Vesicular mole
E. Tubal pregnancy*
143. After removal of uterine tubes of women 32 years old, histologically revealed
chorionic villi, which are depicted on Fig. 2.11. Name the cause of intervention:
A. Abdominal Pregnancy
B. Ovarian Pregnancy
C. Uterine pregnancy
D. Vesicular mole
E. Tubal pregnancy*
144. The presence of decidual reaction and chorionic villi shown in Fig. 2.11 shows:
A. Specific granulomatous inflammation
B. Nonspecific granulomatous inflammation
C. Аcute inflammation
D. Chronic inflammation
E. Pregnancy*
145. A young woman was operated on urgently rupture uterine tube . Gistological
reaction revealed decidual and chorionic villi (Fig. 2.11).This is typical:
A. Specific granulomatous inflammation
B. Nonspecific granulomatous inflammation
C. Acute inflammation
D. Chronic inflammation
E. Pregnancy*
146. What pathology is shown in Fig. 2.41 the number 3:
A. uterine pregnancy
B. tubal pregnancy
C. teratoma
D. Corpus luteum
E. Ovarian Pregnancy*
147. Name the structure shown in Fig. 2.10.
A. embryoblast
B. trophoblast
C. Embryon
D. Chorion
E. Fetus*
148. In what period of developing pathology shown on Fig. 2.12.
A. Gametopathy
B. Embriopathy
C. Congenital
D. bicuspid aortic
E. Blastopathy*
149. Aentsefaly - agenesis of the brain, shown in Fig. 2.39 refers to:
A. Gametopathy
B. Blastopathy
C. Afterbirth pathology
D. Hypoplasia
E. Embriopathy*
150. What pathology is shown in Fig. 2.2
A. Hypoplasia of muscle tissue
B. Dysplasia
C. chondrodysplasia
D. Perverted Osteogenesis
E. Congenital face-heyloshiz*